Skip to main content

ORTHOPEDIC MCQS 20 OB TRAUMA 1B

165 views
375 min read

ORTHOPEDIC MCQS ONLINE 20 OB TRAUMA 1B

 

 

  1. Deep peroneal nerve, sural nerve

  2. Deep peroneal nerve, tibial nerve

  3. Superficial and deep peroneal nerves

  4. Superficial peroneal nerve, tibial nerve

  5. There is no true internervous plane Corrent answer: 4

The posterolateral approach is most appropriate, through which direct anatomic reduction and fixation of both the distal fibular and posterior malleolar fractures can be achieved. The interval is between the peroneal tendons, innervated by the superficial peroneal nerve, and the flexor halluces longus, innervated by the tibial nerve.

 

Posterior malleolar fractures most often occur in the setting of rotational ankle injuries and almost always with concomitant fracture of the lateral and/or medial malleoli. The posterior malleolus is not only an important buttress to posterior translation of the talus, but serves as the origin of the posterior inferior tibiofibular ligament (PITFL) and is therefore essential to syndesmotic stability. For this reason, most authors recommend anatomic reduction and fixation or large posterolateral fragments. The ideal surgical approach is controversial, with some advocating for indirect reduction with anterior-to-posterior screw fixation and others for direct visualization through a posterolateral approach. The posterolateral approach is advantageous in that it provides for both anatomic reduction and fixation of the posterior malleolus as well as the often associated lateral malleolar fractures through the interval between the flexor halluces longus and peroneal tendons.

Irwin et al. review the presentation and management of posterior malleolar fractures. They discuss the various surgical techniques which have been described, noting that the posterolateral approach provides the most direct visualization through which anatomic reduction and fixation can be achieved of both the posterior malleolar and distal fibular fractures. There is debate over which fracture should be fixed first, as fibular fixation can restore length and facilitate reduction of the posterior malleolus, but the fibular hardware often interferes with fluoroscopic visualization.

 

Forberger et al. evaluated the functional outcomes and morbidity following fixation of posterior malleolar fractures via a posterolateral approach performed in 45 consecutive patients. The authors found that the approach not only permitted excellent surgical exposure and stable fixation of the posterior malleolar fragment, but complications were minimal. Functional outcomes were furthermore excellent in the majority of patients.

 

Figures and Illustrations:

Figures A through C are AP, oblique, and lateral radiographic views of a right ankle demonstrating displaced fractures of the lateral and posterior malleoli. Figure D is an axial CT scan of the right ankle following closed reduction and splint application.

 

Illustration A is demonstrates the plane of the posterolateral approach. A cadaveric specimen is shown with a pin in the lateral malleolus retracting the peroneal tendons (2) laterally. The Achilles (5), FHL (1) and PITFL (3) are shown. The posterior malleolus (4) is visualized in the interval between the distal fibula and FHL.

 

Incorrect Answers:

Answer 1: This is not considered an internervous plane, given that the sural nerve is strictly sensory.

Answer 2: This interval describes medially-based approaches to the foot. Answer 3: This interval describes the anterolateral approach to the ankle. Answer 5: A true internervous plane between the tibial and superficial peroneal nerves exists.

 

 

 

 

 

OrthoCash 2020

 

  1. Figures A-C are the radiographs of a 26-year-old male who presents to the emergency department following a motocross accident. Two attempts at a closed reduction by the on-call orthopedic resident were unsuccessful. Figures D and E are the pre-operative axial CT-images that were obtained. The patient undergoes surgical fixation seen in Figure F. Limitations in post-operative dorsiflexion is likely influenced by which of the following?

     

     

     

     

     

     

     

     

     

     

     

     

     

    1. Fracture extension to the posteromedial rim

    2. Initial displacement of the fibula fracture

    3. Posterolateral approach to address both the fibula and posterior malleolus

    4. Initial talar subluxation

    5. Inability to reduce the ankle Corrent answer: 3

    This patient sustained a Bosworth fracture-dislocation and a posterolateral approach to the ankle is used to reduce the fibula back into the incisura and stabilize both the fibula and posterior malleolus. Loss of dorsiflexion is the most common arc of motion that is limited following this fracture pattern with posterior fixation.

     

    The rare Bosworth fracture-dislocation is a posterior dislocation of the fibula which becomes entrapped behind the tibia. As demonstrated in this vignette, these injuries are extremely difficult to close reduce secondary to the ridge of the posterolateral distal tibia. The irreducible nature of this injury is a known risk factor for the development of compartment syndrome. The CT images further demonstrate fracture extension to the posteromedial rim (“posterior pilon variant”). In this situation, the only effective method to reduce the fracture is through an open posterolateral approach with the interval between the flexor hallucis longus and the peroneal tendons. This is the same approach that is utilized for fixation of the posterolateral fragment and fibula. Loss of dorsiflexion has been demonstrated following this fracture pattern with posterior fixation. The positioning of the plates in Figure F suggests the

    utilization of a posterolateral approach.

     

    Switaj et al. retrospectively reviewed 270 patients with posterior malleolus and pilon variant fractures that were operatively treated. They reported that female gender, increasing age, and diabetes may be associated with both fractures of the posterior malleolus as well as the posterior pilon variant. They concluded that the relative frequency of the posterior malleolar fracture and posterior pilon variant in operatively treated ankle fractures was 50% and 20%, respectively.

     

    Gardner et al. reviewed closed pronation-external rotation 4 (PER4) ankle fractures that involved the posterior malleolus without a complete tear of the posterior-inferior tibiofibular ligament (PITFL) treated with either posterior malleolus ORIF or syndesmotic stabilization. They reported that stiffness was restored to 70% after fixation of the posterior malleolus, and to 40% after syndesmosis stabilization. They concluded that syndesmotic stability may be obtained more effectively by the fixation of the posterior malleolus rather than by using a syndesmotic screw.

     

    Berkes et al. reviewed 108 supination-external rotation 4 (SER4) fractures to examine the impact of articular surface congruity on the functional outcomes. They reported that 33% of patients had elements of articular surface incongruity on postoperative CT scanning. They found that the group with articular incongruity had significantly worse foot and ankle outcome scores (FAOS) with regards to symptoms, pain, and activities of daily living. They concluded that the presence of postoperative articular incongruity correlated with inferior early clinical outcomes.

     

    Kalem et al. compared the results of AP screws, PA screws and posterior buttress plate use for posterior malleolar fixation of trimalleolar fractures. They report better AOFAS scores in the PA group and the plate group compared to the AP screw group. However, they concluded that utilization of a posterolateral approach was found to have a loss of ankle dorsiflexion of 5 degrees or greater for both the PA screws and plate groups; 39% and 24%, respectively.

     

    Chen et al. addressed the term “posterior pilon” in their letter to the Editor. He reports that this term was first introduced to describe severe trimalleolar fractures with the presence of a fourth fragment located deeper than the avulsed posterior fragment. The mechanism for this injury results from the impaction of the posterior fragments secondary from axial loading, which is the main causative factor of a classic pilon fracture.

    Figures A-C: The initial radiographs reveal the posterior subluxation of the talus with associated posterior subluxation of the fibula without significant coronal plane deformity. This deformity should raise the suspicion of a Bosworth fracture-dislocation, especially if closed reduction is not successful. Figures D and E: Axial CT images demonstrating Bosworth fracture-dislocation of the fibula entrapped behind the tibia. Also, note the fracture extension to the posteromedial rim in this posterior pilon variant.

    Figure F: Positioning of the plate suggests a posterolateral approach to address both the fibula and posterior malleolus fractures.

     

    Incorrect Answers:

    Answer 1: Fracture extension to the posteromedial rim makes this a "posterior pilon variant," but in itself is not the reason for a decrease in postoperative dorsiflexion

    Answer 2: Initial displacement of the fibula fracture may be significant with some ankle injuries. However, the amount of initial displacement has not been correlated with a loss of postoperative dorsiflexion.

    Answer 4: The presence of talar subluxation is often seen with ankle injuries. While this may be a result of a large posterior malleolus fragment requiring surgery through a posterolateral approach, the subluxation itself has not been shown to limit postoperative dorsiflexion.

    Answer 5: The inability to reduce the ankle may suggest a Bosworth fracture-dislocation. Though reduction helps with soft tissue swelling and chondrolysis, it has not been shown to correlate with postoperative dorsiflexion limitations.

     

     

     

    OrthoCash 2020

     

  2. A 69-year-old man presents with the injury depicted in Figures A and B after a fall from his attic. On initial presentation, his lower extremity strength is grade 3/5 but has full sensation in bilateral lower extremities. When his indwelling urinary catheter is pulled, his anal sphincter tightens. He remains bed-bound for the next 4 days and unfortunately develops aspiration pneumonia, requiring urgent intubation and increased positive end-expiratory pressures and a high ventilatory rate to keep him oxygenated. He remains febrile to 39°C, has a pulse rate of 130 bpm, and requires high dose pressors to keep his mean arterial pressure above 65 mmHg. Laboratory markers include WBC 18,000 with 95% PMNs, ESR 120 mm/h, and CRP 200 mg/L. Which of the following findings would most likely be observed in the type of shock this patient is currently in?

     

     

     

    1. Decreased mixed venous oxygen saturation

    2. Increased pulmonary capillary wedge pressure

    3. Bradycardia

    4. Loss of rectal tone

    5. Decrease in systemic vascular resistance Corrent answer: 5

    This patient sustained an L3 burst fracture with no evidence of spinal shock on presentation, however, suffered an aspiration event causing pneumonia and is now in septic shock. Decreased systemic vascular resistance is observed during septic shock.

     

    Septic shock has a mortality rate estimated at 25%-50%. It is a medical emergency resulting from decreased tissue perfusion and oxygen delivery as a result of severe infection and sepsis. Septic shock is associated with decreased cardiac output, decreased pulmonary capillary wedge pressure, decreased central venous pressure, and increased mixed venous oxygen saturation. It is differentiated from hypovolemic shock in that the systemic vascular resistance is decreased in septic shock, whereas it is increased in hypovolemic shock.

    Although this patient has a burst fracture, it is important to distinguish this injury from neurogenic shock, which is usually a consequence of traumatic spinal cord injuries to the upper thoracic or cervical levels that results in autonomic dysregulation (loss of sympathetic tone and unopposed parasympathetic response, leading to bradycardia and decreased vascular resistance).

     

    Schouten et al. reviewed the initial assessment and emergency treatment in

    the spine-injured patient. They reported that hypotension should always be assumed to be of hemorrhagic etiology until proven otherwise, with initial treatment involving fluid resuscitation with a focus on raising systemic vascular resistance in cases of septic shock. They recommended that all aspects of emergent care, including resuscitation and choice of imaging modalities, be systematically reviewed, and practice guidelines are adopted institutionally.

     

    Hadley et al. reviewed blood pressure management after acute spinal cord injury. They reported that hemodynamic alterations during acute spinal cord injury include hypotension, cardiac dysrhythmias, decreased peripheral vascular resistance, and reduced cardiac output. They recommended that hypotension (systolic blood pressure <90 mm Hg) be avoided if possible or corrected as soon as possible after acute spinal cord injury.

     

    Sekhon et al. reviewed the epidemiology, demographics, and pathophysiology of acute spinal cord injury. They reported that total peripheral resistance and cardiac output may remain depressed for a prolonged period of time in both neurogenic and septic shock. They concluded that survival has improved along with a greater understanding of patterns of presentation, survival, and complications.

     

    Figures A and B depict an L3 burst fracture with retropulsion of fracture fragments into the spinal canal. Illustration A is a table demonstrating the differences between hypovolemic and septic shock.

     

    Incorrect Answers:

    Answer 1: Septic shock is associated with INCREASED mixed venous oxygen saturation.

    Answer 2: Septic shock is associated with DECREASED pulmonary capillary wedge pressure.

    Answer 3: Septic shock is often associated with tachycardia, not bradycardia. Answer 4: Patients with cauda equina syndrome and spinal shock may present with loss of rectal tone, but this is not typical for patients in septic shock in isolation.

     

     

     

     

     

     

    OrthoCash 2020

     

  3. A 45-year-old female barista from Portland fell off her skateboard and sustained a closed distal radius fracture. The patient undergoes

    open reduction internal fixation (ORIF). Post-operatively she is given a prescription with the goal of mitigating a potential adverse outcome.

    Which of the following has evidence to support its utility in this clinical situationr?

    1. Alendronate 700mg once per week for 3 months

    2. Vitamin C 500mg once daily for 50days

    3. Alendronate 70mg once per week for 3 months

    4. Vitamin C 200mg once daily for 50days

    5. Vitamin C 1500mg once daily for 100days Corrent answer: 2

    There is some evidence to support that Vitamin C 500mg PO taken daily for 50 days may decrease the chances of developing complex region pain syndrome (CRPS) following ORIF of distal radius fractures.

     

    CRPS is generally classified as type I, in which symptoms develop in an absence of specific nerve injury, or type II, in which there is the presence of specific identifiable nerve injury. The diagnostic criteria include pain disproportionate to the inciting event and a combination of sensory, vasomotor, sudomotor, and trophic signs. Treatment often involves a combination of physical therapy and pharmacologic treatment, nerve blocks, or chemical or surgical sympathectomy. However success is mixed, therefore prevention is paramount. Given that the etiology is thought to involve a traumatic incident that elicits an overwhelming systemic inflammatory reactions, causing disproportion pain and increased vascular permeability, it is hypothesized that the administration of Vitamin C as a free radical scavenger may protect the vascular endothelium and reduce the incidence of CRPS. Prophylactic administration after management of distal radius fractures is supported by moderate evidence as per the 2013 AAOS guidelines on the Treatment of Distal Radius Fractures.

     

    Zollinger et al. performed a double-blinded randomized controlled trial of 427 wrist fractures, allocating patients to receive placebo or 200, 500, or 1500mg of vitamin C for 50 days. The authors found that in the vitamin C group, the incidence of CRPS was 2.4%, while it was 10.1% in the placebo group. They showed no difference in relative risk between the 500mg and 1500mg groups. The authors concluded that Vitamin C reduces the prevalence of CRPS and a daily dose of 500mg for 50 days is recommended.

     

    Koval et al. comprehensively reviewed controversies in the management of distal radius fractures. The authors highlight the evidence supporting vitamin C use in treating disproportionate pain. However, because there is no objective

    measure for the diagnosis CRPS, the evidence remains limited.

     

    Incorrect Answers:

    Answer 1: A prospectiv randomized controlled trial was conducted to evaluate healing rates in patients following administration of 70mg alendronate weekly for three months. There was no difference in clinical outcomes or healing between control and experimental groups.

    Answer 3: Alendroate has not shown to be beneficial in this setting.

    Answer 4: The recommended length and dose of treatment is Vitamin C 500mg for 50 days.

    Answer 5: Studies have shown no difference between 500mg and 1500mg. A daily 500mg dose is recommended by the AAOS.

     

     

     

    OrthoCash 2020

     

  4. A 30-year-old plastic surgery resident fell from a height of 12' and sustained a right both-bone forearm fracture. The patient underwent open reduction and internal fixation with 3.5mm Limited Contact Dynamic Compression Plating. Three months after fixation the patient has no forearm tenderness and has full active range of motion of his fingers, hand, and wrist. However, pronation and supination are severely limited. This affects his ability to suture during surgery and knit, although he has returned to his other recreational activities. His most recent radiographs are shown in Figures A and B. What is the most likely cause for his lack motion?

     

     

     

    1. Fracture nonunion

    2. Posterior interosseous nerve (PIN) palsy

    3. ECU interposition at the DRUJ

    4. Heterotopic ossification

    5. Loss of radial bow Corrent answer: 5

    This patient sustained a both-bone forearm fracture that was fixed with a long, straight plate which did not account for the native radial bow. This is the most likely reason for his limited pronosupination.

     

    Open reduction with internal fixation of both-bone forearm fractures has generally demonstrated a high rate of union and excellent functional results. However, restoration of the radial bow is a critical. A lack of radial bow will limit pronosupination. To avoid this problem, the surgeon could have used a shorter plate or a plate with an anatomic radial bow, which many modern forearm plating systems now account for. The assessment and calculation for radial bow is highlighted in Illustration A. Other factors which may contribute to limited pronosupination include heterotopic ossification and scarring, though the former is not seen on radiographs.

    Matthews et al. evaluated the impact of radial bow on pronosupination in a cadaveric both-bone forearm fracture model. The authors found that with a 10-degree deviation from the anatomic radial contour, there was no loss of pronosupination. However when this was increased to a 20-degree angulation, there was a significant loss of forearm pronosupination. The authors underscored the importance of restoration of radial bow in fixation of both-bone forearm fractures.

     

    Schemitsch and Richards evaluated the outcomes in 55 adult patients following both-bone forearm fracture fixation. They reported good-excellent outcomes in 84% of cases. The authors found that bone grafting did not affect union (54/55 achieved union), but noted that restoration of the radial bow affected functional outcome and grip strength. The authors stressed the importance of restoration of radial bow.

     

    Figure A is an AP radiograph of the forearm following ORIF of a both-bone forearm fracture with a long straight plate which fails to restore the anatomic radial bow. Figure B is a lateral radiograph of the same patient demonstrating fracture fixation with a long straight plate. Illustration A demonstrates the method for determining radial bow. A line from the sigmoid notch to the bicipital tuberosity is drawn (Y). A perpendicular line is drawn from Y to the point on the radial with maximal bow (a) (This number is usually 7%). The point of this intersection to the bicipital tuberosity is marked (x). The location of maximal bow is x/y (This number is usually 60%).

     

    Incorrect Answers:

    Answer 1: The patient is non-tender and has returned to activities. He does not have any clinical or radiographic evidence of fracture nonunion.

    Answer 2: The patient has full active range of motion of his digits and wrist, which would indicate that the PIN is functional.

    Answer 3: The ECU can be interposed at the DRUJ, particularly in radial shaft fractures in which the DRUJ is unstable after fracture fixation. However, this patient does not have DRUJ instability. Furthermore, there is no evidence of DRUJ diastasis on X-Ray.

    Answer 4: The radiographs are negative for any evidence of heterotopic ossification, though this is another potential source of limited motion following ORIF of both-bone forearm fractures.

     

     

     

     

     

    OrthoCash 2020

     

  5. A 46-year-old male presents with the radiographs in Figure A following a skiing accident. You decide to treat this injury with an intramedullary nail. During insertion of your nail, it's decided that blocking screws are needed. In which position should they be placed to prevent the most common malunion?

     

     

     

     

    1. A and D

    2. A and C

    3. B and C

    4. B and D

    5. A and B Corrent answer: 3

    Proximal third tibial shaft fractures have a propensity to be malunited in valgus and procurvatum. To prevent this, blocking screws can be used in the

    concavity of the expected deformity, locations B and C.

     

    Malunion and soft tissue compromise are two complications with a high association to proximal third tibial shaft fractures. The most common deformity seen in a malunited fracture is valgus and procurvatum due to the pull of the pes anserine and the patellar tendon, respectively. Numerous techniques exist to help prevent this deformity including the use of blocking screws, unicortical plating, and the universal distractor. If blocking screws are employed they should be placed on the concavity of the expected deformity, which in this case would be posterior in the sagittal plane and laterally in the coronal plane of the proximal fragment. Once reduction is obtained, if the blocking screws are left in place they have the added benefit of creating a stronger construct.

     

    Kulkarni et al review the use of blocking screws for intramedullary nailing of proximal third tibial shaft fractures. They report an average time to union of

    4.2 months with 5/66 cases going on to non-union, which were all successfully treated with bone grafting, and 1/66 resulting in a malunion. They conclude that blocking screws help maintain fixation and alignment of proximal third tibia fractures treated with intramedullary nails.

     

    Stinner et al review techniques for intramedullary nailing of proximal third tibial shaft fractures. They report on several techniques including blocking screws, unicortical plating, and clamp reduction to assist the surgeon in obtaining an adequate reduction. They conclude that while implant design and surgical techniques have improved, the surgeon must still be cognizant of the valgus and apex anterior deformity and know how to neutralize these forces.

     

    Franke et al review the indications and techniques for suprapatellar nailing of tibia fractures. They report that placing the knee in 20 degrees of flexion for suprapatellar nailing acts to neutralize the pull from the quadriceps muscle, thus decreasing the apex anterior deformity. They conclude that this technique can be used for Gustilo-Anderson Grade I-II tibial shaft fractures to help reduce the incidence of valgus and apex anterior malunions.

     

    Figure A is the AP and lateral radiographs of a proximal third tibial shaft fracture. Blocking screws should be placed in positions B and C to help reduce the incidence of valgus and apex anterior malalignment.

     

    Incorrect Answers:

    Answers 1-2,4-5: Blocking screws should be placed in the concavity of the deformity to be prevented, thus positions B and C in this example.

     

     

    OrthoCash 2020

     

  6. Figures 1 and 2 are the radiographs of a 41-year-old diabetic male presenting with right lower extremity pain after cutting his leg it on a piece of rusty metal yesterday. Serial physical exam demonstrates rapid progression of the overlying erythema and worsening pain. In the emergency department, labs are significant for a C-reactive protein (CRP) of 180 mg/L, white blood cell (WBC) count of 19,000/mm3, glucose of 11 mmol/L, creatinine of 150 umol/L, and sodium of 120 mmol/L. He has a temperature of 102°F and a heart rate of 110 bpm. What additional laboratory value is needed to calculate this patient’s LRINEC (Laboratory Risk Indicator for Necrotizing Fasciitis) score?

     

     

     

     

    1. ESR

    2. Hemoglobin

    3. Potassium

    4. Bicarbonate

    5. Calcium

     

    Corrent answer: 2

     

    The LRINEC score is based on the patient's serum CRP, WBC count, hemoglobin, sodium, creatinine, and glucose.

     

    The LRINEC score is a clinical tool designed to help distinguish between necrotizing fasciitis and other soft tissue infections using these 6 key laboratory values. The most important of these is CRP, as a value >150 mg/L

    contributes 4 of a possible 13 points. Illustration A demonstrates the scoring system for each laboratory value used in the calculation of the LRINEC score. A score >6 has been shown to have a PPV of 92% for having necrotizing fasciitis. The patient in this vignette already has a score of 10 without knowledge of the hemoglobin level, which is very concerning for necrotizing fasciitis. As a result, emergent surgical debridement is indicated.

     

    Stoneback et al. reviews the presentation and management of necrotizing fasciitis. The authors note that the incidence of necrotizing fasciitis is between 500 and 1500 cases per year in the United States, and that the mortality rate averages 21.9%. They emphasize that necrotizing fasciitis may be difficult to distinguish from more common skin infections such as cellulitis or soft tissue abscess in its early stages but will often rapidly deteriorate, as illustrated in this vignette.

     

    Wong et al. developed the LRINEC scoring system. In a retrospective review of 145 patients with necrotizing fasciitis and 309 patients with severe cellulitis or abscesses, they used their scoring system to demonstrate that a LRINEC score of 6 points or greater had a PPV of 92.0% and an NPV of 96.0%.

     

    Tsai et al. challenged the utility of the LRINEC score in aiding in the diagnosis of Vibrio Necrotizing Fasciitis. In their retrospective review of 70 patients with Vibrio Necrotizing Fasciitis, they showed that only 11% of patients had a LRINEC score >6. The authors concluded that the LRINEC scoring system is of more limited utility for this subset of the disease and propose that severe hypoalbuminemia, thrombocytopenia, and increased banded forms of leukocytes may have greater utility.

     

    Figures A and B are AP and lateral radiographs of the lower leg demonstrating diffuse subcutaneous gas. This is concerning for necrotizing fasciitis.

    Illustration A is a table showing the LRINEC scoring breakdown. A maximum total score of 13 could be achieved.

     

    Incorrect Answers:

    Answer 1: ESR is a non-specific measure of inflammation. While likely elevated with necrotizing fasciitis, it is not used to help calculate LRINEC score.

    Answer 3: Hyperkalemia may occur with myonecrosis but potassium is not included in the LRINEC score calculation.

    Answer 4: Acidosis may be seen in the setting of necrotizing fasciitis, but bicarbonate is not included in the LRINEC score calculation.

    Answer 5: Calcium is not a component of the LRINEC scoring system.

     

     

     

     

     

    OrthoCash 2020

     

  7. A 35-year-old male presents to the emergency department following a motorcycle accident. On initial examination in the emergency department, he is found to have a left flail arm and multiple other injuries. There is a concern for a brachial plexus injury. What examination findings would support a diagnosis of a pre-ganglionic injury?

    1. Abnormal histamine response

    2. Flail arm

    3. Intact sensory nerve action potentials (SNAPs)

    4. Lateral scapular winging

    5. Normal cervical paraspinal signal on EMG/NCV Corrent answer: 3

    Brachial plexus injuries are classified as either pre-ganglionic or post-ganglionic. A pre-ganglionic injury would be supported by intact sensory nerve action potentials (SNAPs).

     

    Diagnosing brachial plexus injuries as either pre- or post-ganglionic is multi-faceted and begins with a thorough physical examination. Findings suggestive of a pre-ganglionic injury would include Horner Syndrome (due to disruption of the sympathetic chain), MEDIAL scapular winging resulting from rhomboid and serratus anterior paralysis, cervical paraspinal muscle weakness, absent sensation, and hemidiaphragmatic paralysis. CT myelography may demonstrate pseudomeningocele, and NCS/EMG would show intact sensory nerve action potentials (SNAPs) with denervation of the cervical paraspinal musculature. Finally, a histamine test would show a triple response, with redness, wheal, AND flare. Conversely, a post-ganglionic injury would be

    supported by BOTH motor and sensory deficits of the flail arm, maintained innervation to cervical paraspinal muscles, and an abnormal histamine response test (redness and wheal with NO flare).

     

    Limthongthang et al. reviews the common clinical findings associated with pre-and post-ganglionic injuries. The authors discuss the diagnostic algorithm, including a thorough history, physical examination, electrodiagnostic studies, and CT myelogram or MRI. They emphasize that an angiogram should additionally be considered given that 20% of brachial plexus injuries have associated major vascular injuries. The authors conclude with appropriate surgical timing and suggest potential treatments depending on injury location.

     

    Giuffree et al. reviews management of brachial plexus injuries. The authors discuss appropriate timing of treatment based on the injury mechanism. They emphasize the priorities of functional restoration, beginning with elbow flexion, followed by a stable shoulder, and lastly intrinsic hand function. The authors present various surgical options to attain these goals and discuss their outcomes.

     

    O'Shea et al. discusses the utilization of advanced imaging and electrodiagnostic testing for evaluation of brachial plexus injuries. The authors note the utility of CT myelogram toward identifying root avulsions. They also suggest that electrodiagnostic studies be obtained no sooner than 4 weeks following injury and subsequently at 6-week intervals to monitor progression. They note that SNAPs are the most important criteria in distinguishing between pre- and post-ganglionic injuries, and conclude that electromyographic studies are crucial in determining not only the level of injury but also donor nerves for transfers.

     

    Incorrect Answer:

    Answer 1: A normal “triple” response to intradermal injection of histamine involves erythema due to capillary dilation, wheal due to fluid extravasation from increased permeability, and flare due to arteriolar dilatation AND to axon reflex in sensory nerve. A response with a flare in an insensate area of skin suggest that the reflex arc is intact and that the lesion has to be proximal to the dorsal root ganglion. Conversely, when the lesion is post-ganglionic, the test will be “abnormal” (i.e. without the flare) as the continuity between the skin and dorsal root ganglion will have been interrupted.

    Answer 2: A flail arm is characteristic of both pre- and post-ganglionic brachial plexus injuries, given that both involve absence of motor function.

    Answer 4: Lateral scapular winging is not associated with a pre-ganglionic injury. Lateral winging characteristically results from trapezial pathology, which is associated with CN XI injury. Instead, MEDIAL scapular winging would be

    indicative of a pre-ganglionic injury due to involvement of the dorsal scapular nerve (branching from the C5 root) and the long thoracic nerve (branching from the C5-7 roots).

    Answer 5: Normal EMG/NCV results in the cervical paraspinal muscles would be indicative of a post-ganglionic injury.

     

     

     

    OrthoCash 2020

     

  8. A 47-year-old male diabetic has developed a calcaneal ulcer, as shown in figure A, that has been managed with wound care and a total contact cast. He presents to the emergency department with worsening erythema progressing up his leg with new draining wounds. On admission, despite antibiotics and intravenous fluids, he develops worsening pain and new bullae shown in Figure B. His laboratory studies indicate CRP >200, WBC 25k, Sodium 127, glucose 233, and hemoglobin 12. What is the best next step in the management of his condition?

     

     

     

     

     

     

    1. Emergent vascular exploration

    2. Upgrade level of care ICU with antibiotic escalation

    3. MRI foot for osteomyelitis evaluation

    4. Emergent surgical debridement including possible amputation

    5. CT with contrast of lower extremity Corrent answer: 4

    This patient meets the clinical criteria for the diagnosis of necrotizing fascitis, therefore the next step in management is surgical debridement with possible amputation.

     

    Necrotizing fasciitis is an aggressive infection with rapid spread along fascial planes. Initial presentation may be consistent with cellulitis; however, rapid progression along fascial planes may result in skin necrosis, muscular invasion, and subsequent myonecrosis. The rapidity of diagnosis and emergent surgical debridement is essential. Risk factors including intravenous drug abuse, alcohol abuse, and diabetes should be assessed, and if necrotizing fascitis is on the differential, the LRINEC score may be employed for diagnosis in equivocal cases. This scoring system can be found on our associated Orthobullets page.

     

    Wong et. al. developed the Laboratory Risk Indicator for Necrotizing Fasciitis (LRINEC) score to assist in early diagnosis of necrotizing soft tissue infections. The authors used logistic regression to identify significant predictors of necrotizing fasciitis. The score utilizes a total white cell count, hemoglobin, sodium, glucose, serum creatinine, and CRP. A LRINEC score of 6 points or higher has a positive predictive value of 92%, and below 6 points a negative predictive value of 96%. Importantly, ESR is not a component in the scoring system as it correlated poorly with risk. The patient presented already has two positive serum findings with an LRINEC score of 6.

     

    Stoneback et al. review the diagnosis and management of necrotizing fasciitis. They note that Group A streptococcus is one of the most prevalent organisms, but infections are typically polymicrobial. Because of the need for prompt initiation of treatment, adjunctive diagnostic tests should not impede the timing of surgical exploration and debridement. Necrotizing fasciitis is a clinical diagnosis where only emergent surgical debridement and appropriate antibiotic treatment can prevent progression and death.

     

    Tsai et al. retrospectively reviewed 70 patients with 71 episodes of Vibrio necrotizing fasciitis and sepsis. Of the 70 patients, 68 had a history of contact with seawater or raw seafood; 66 had underlying chronic diseases. They noted that severe hypoalbuminemia, severe thrombocytopenia, and increased banded forms of leukocytes are laboratory risk indicators of necrotizing fasciitis

    that aid in pointing toward the initiation of early surgery and predict a higher risk of death.

     

    Figure A is a clinical photograph of a calcaneal ulcer. Figure B shows ascending erythema and bullae consistent with a necrotizing skin infection. Illustration A demonstrates the LRINEC scoring system. The scoring system is not largely utilized as a screening tool due to its poor sensitivity in studies that attempted to validate it.

     

    Incorrect answers:

    Answer 1: There is no indication in this scenario that this patient's clinical picture is due to a vascular issue.

    Answer 2: This patient will likely require ICU admission, but surgical debridement is most prudent.

    Answers 3 and 5: Imaging studies can assist with the diagnosis, but advanced imaging takes valuable time and should not delay emergent surgical debridement. The incidence of gas in the subcutaneous tissues is only 13%, but its presence should trigger urgent debridement.

     

     

     

     

     

     

    OrthoCash 2020

  9. A 36-year-old man with HIV, chronic kidney failure, and chronic IV drug use presents with worsening fevers, chills, and purulent drainage from his leg. His infection progresses rapidly and he becomes acutely septic. He is therefore taken urgently to the OR for radical debridement of the fascia and surrounding tissues, with a plan for delayed closure. His CRP is 90 mg/dL and he is hyponatremic. Intraoperative cultures are obtained. To cover the most common organism(s) associated with this condition, what antibiotic would you initially recommend?

     

     

     

     

    1. Intravenous vancomycin

    2. Intravenous vancomycin and gentamicin

    3. Intravenous gentamicin

    4. Intravenous linezolid and meropenem

    5. Intravenous micafungin

     

    Corrent answer: 4

     

    This patient with lower extremity necrotizing fasciitis underwent a radical debridement of fascia and surrounding tissues. The most common culture isolate from necrotizing fasciitis is polymicrobial, which should be the target of initial antibiotic treatment. Of the above options, intravenous Linezolid and Meropenem would be an acceptable empiric therapy as Linezolid would cover MRSA and invasive group A Streptococcus while Meropenem would cover gram-positive/negative anaerobic coverage and enteric organisms.

     

    Necrotizing fasciitis is an aggressive infection with rapid spread along fascial planes. While the initial presentation may suggest cellulitis, rapid progression along fascial planes may result in skin necrosis and subsequent myofascial necrosis. The time from admission to debridement has been shown to be a significant predictor of outcomes. Immediate empiric antibiotics covering

    polymicrobial species including aerobic, anaerobic, gram-positive and gram-negative bacteria are of equal importance.

     

    Stoneback et al. reviewed the diagnosis and management of necrotizing fasciitis. They reported that while most necrotizing fasciitis infections are polymicrobial, group A Streptococcus remains one of the more prevalent organisms in isolation. They recommended surgical debridement as the utmost importance in necrotizing fasciitis management and emphasized appropriate antibiotic selection in treating sepsis and halting the further bacterial spread.

     

    Wong et al. developed the Laboratory Risk Indicator for Necrotizing Fasciitis (LRINEC) score, a novel diagnostic scoring system for distinguishing necrotizing fasciitis from other soft tissue infections based on laboratory tests routinely performed for the evaluation of severe soft tissue infections. This score allocated 1 point each to the following labs: total white cell count, hemoglobin, sodium, glucose, serum creatinine, and C-reactive protein. They reported that a cutoff value of 6 points had a positive predictive value of 92.0% and a negative predictive value of 96.0%, and concluded that the LRINEC score is a powerful score that can detect even clinically early cases of necrotizing fasciitis.

     

    Tsai et al. investigated whether clinical indicators predict laboratory findings during the initial stages of necrotizing fasciitis and the relationship between the LRINEC score and the diagnosis of vibrio infection. They reported that a systolic blood pressure of 90mmHg or less at the time of admission predicted mortality and that patients with a LRINEC score of >6 had an 11% chance of surviving vibro-specific necrotizing fasciitis.

     

    Figure A is a clinical image depicting necrotizing fasciitis of the lower extremity, with characteristic ischemic patches, cutaneous gangrene, dermal induration, bullae formation. Illustration A is a table from Wong's study that demonstrates the components, clinical values, and scoring system for the LRINEC score.

    Scores of 6 or greater are highly concerning. They concluded that severe hypoalbuminemia, severe thrombocytopenia, and increased leukocytosis were reliable markers.

     

    Incorrect Answers:

    Answer 1: While intravenous Vancomycin would cover MRSA and aggressive streptococcus species, it would not cover gram-negative anaerobic organisms. Answer 2: This patient has chronic kidney failure, which would be worsened by 2 antibiotic agents that are known to be nephrotoxic.

    Answer 3: Intravenous Mentamicin alone would not cover the gram-positive organisms.

    Answer 5: Intravenous Micafungin would cover fungal organisms, but would fail to cover bacteria.

     

     

     

     

     

     

    OrthoCash 2020

     

  10. A 27-year-old male is involved in a motor vehicle collision and presents to the ER with the right lower extremity injury shown in Figures A and B. He undergoes immediate closed reduction and the post-reduction CT is shown in Figures C and D. The patient undergoes percutaneous surgical screw fixation of the injury. At 2 years followup, he presents with a supination deformity with decreased eversion of the foot at rest. Radiographs reveal no evidence of talus subchondral sclerosis or collapse. Which of the following is the most likely cause of the finding in this patient?

     

     

     

     

     

     

     

     

     

     

     

    1. Avascular necrosis

    2. Subtalar arthritis

    3. Tibiotalar arthritis

    4. Varus malunion

    5. Planovalgus

     

    Corrent answer: 4

     

    The patient has sustained a displaced talar neck fracture with medial comminution which requires open reduction and internal fixation to restore anatomic alignment. The patient has likely sustained varus malunion which is a common deformity noted with non-anatomical reduction leading to hindfoot supination and decreased eversion.

     

    Talar neck fractures, while uncommon, can lead to significant morbidity in the ambulatory patient. Treatment of these injuries requires prompt reduction due to the tenuous blood supply to the talus. The most common complications noted after a talar neck fracture are avascular necrosis, subtalar arthritis, tibiotalar arthritis, and varus malunion. A talus varus malunion generally occurs due to medial talar neck comminution and/or non-anatomic reduction of the injury. This deformity can lead to decreased eversion and a supination deformity causing patients to ambulate on the lateral aspect of the foot. These malunions are generally treated with a medial opening wedge osteotomy.

     

    Fortin and Balazsy review the pathology and treatment of traumatic talus fractures. They cite that injuries to the head, neck, or body of the talus can result in permanent pain, loss of motion, and deformity. They note that failure to recognize fracture displacement (even when minimal) can lead to undertreatment and poor outcomes. They conclude that unrecognized medial talar neck comminution can lead to varus malunion and a supination deformity with a decreased range of motion of the subtalar joint.

     

    Lindvall et al. performed a retrospective review to evaluate the long-term results (average of 74-month followup) of surgical treatment of isolated, displaced talar neck and/or body fractures with stable internal fixation in 25 patients. They noted an overall union rate of 88% regardless of surgical timing. The authors noted posttraumatic subtalar arthritis in all patients (100%) and talus osteonecrosis in 13/26 (50%) of patients. The authors concluded that a delay in surgical fixation does not appear to affect the outcome, union, or prevalence of osteonecrosis and that posttraumatic arthritis is a more common complication than osteonecrosis following operative treatment.

     

    Figures A and B are the Lateral and AP radiographs of a displaced talar neck fracture with tibiotalar dislocation. Figures C and D are the post-reduction sagittal foot CT cuts that reveal a displaced talar neck fracture with medial

    comminution.

     

    Incorrect Answers:

    Answer 1: The patient does not have any radiographic signs of avascular necrosis according to the vignette.

    Answer 2: Subtalar arthritis is the most common complication following talar neck fractures but the clinical appearance of the patient's foot is due to varus malunion.

    Answer 3: Tibiotalar arthritis is a common complication following talar neck fractures but the clinical appearance of the patient's foot is due to varus malunion.

    Answer 5: A planovalgus foot is unlikely to occur after a talus fracture, furthermore a planovalgus foot would lead to a pronation deformity.

     

     

     

    OrthoCash 2020

     

  11. Figures A-C are a 32-year-old female who presents to the trauma bay after falling off a roof. Paramedics state that she was initially complaining of back pain, but she now appears irritable and lethargic. She has received 2 liters of crystalloid since arriving in the trauma bay. Her blood pressure is now 76/42. A Foley catheter is placed, and her urine output is 12 ml/hour. What additional finding would suggest hemorrhagic shock over neurogenic shock?

     

     

     

     

     

     

     

     

    1. Tachycardia

    2. Bradycardia

    3. Decreased cardiac output

    4. Vasodilation

    5. Warm dry skin Corrent answer: 1

    This patient sustained a multi-traumatic injury and is suffering from hemorrhagic shock. The presence of tachycardia would favor the diagnosis of hemorrhagic shock over neurogenic shock.

     

    In initial hypovolemic shock (stage I, <15% blood loss), heart rate, blood pressure, urine output, pH, and mental status are largely unaffected, as the body can typically compensate for this amount of volume loss. With increasing blood loss, tachycardia, hypotension, low urine output, decreased pH, hemoconcentration, cool clammy skin, and altered mental status can be observed. This patient sustained a splenic injury and T12 and L3 vertebral injuries. Both hemorrhagic and neurogenic shock may result in a decrease in cardiac output and hypotension; however, neurogenic shock may also present with warm dry skin, vasodilation, loss of sympathetic tone, and paradoxical bradycardia.

     

    Schouten et al. review the initial management of the spinal column and spinal cord injury (SCI) in the emergency room. The authors discuss that acutely, deep tendon reflexes are absent and paralysis is flaccid, but that this transitions to hyper-reflexia and spasticity with time. They conclude that spinal shock is a temporary physiologic state of the acutely traumatized spinal cord, evident by the transient absence of reflexive function caudal to the spinal cord injury followed by gradual return of reflex activity. Additionally, the diagnosis of a "complete" spinal cord injury cannot be made until the neural "shock" resolves.

     

    Spector et al. review cauda equina syndrome. They report on the characteristic findings such as varying patterns of low back pain, sciatica, lower extremity sensorimotor loss, and bowel and bladder dysfunction. Classical findings include urinary retention, saddle anesthesia of the perineum, bilateral lower extremity pain, numbness, and weakness. The authors conclude that there should be a high index of suspicion in the postoperative spine patient with back and/or leg pain refractory to analgesia, especially in the setting of urinary retention.

     

    Hadley et al. review the presentation of acute SCI. The authors report that hypotension has been associated with an increased risk of mortality following

    spinal cord injuries. They conclude that hypotension (systolic blood pressure

    <90 mm Hg) should be avoided after acute SCI, with the maintenance of mean arterial blood pressure at 85 to 90 mm Hg for the first 7 days after acute spinal cord injury in order to improve spinal cord perfusion.

     

    Sekhon et al. review the epidemiology, demographics, and pathophysiology of SCI. They discuss primary and secondary injury, with the mediators of secondary injury including vascular mechanisms, excitatory amino acids, calcium, sodium, free radicals, inflammation, and apoptosis.

     

    Figures A is a sagittal CT image demonstrating T12 and L3 vertebral fractures. Figure B is an axial CT image demonstrating a splenic injury with hematoma. Figure C is a clinical photograph demonstrating left abdominal flank injury.

    Illustration A shows the classification of hemorrhagic shock.

     

    Incorrect Answers:

    Answer 2: Bradycardia is a paradoxical response that occurs with spinal cord injury. In the setting of hypovolemia, there should be a responsive tachycardia, as seen with hemorrhagic shock

    Answer 3: Decreased cardiac output occurs in both hemorrhagic and neurogenic shock.

    Answer 4: Vasodilation is a response seen following the loss of sympathetic tone with spinal cord injuries. This finding would favor neurogenic shock over hemorrhagic shock.

    Answer 5: Warm dry skin is also a response that occurs with the loss of sympathetic tone seen with spinal cord injury. With a hemorrhagic shock, the skin is usually cool, clammy and pale.

     

     

     

     

     

     

    OrthoCash 2020

     

  12. A 32-year-old inebriated male falls from a mechanical bull at a bar and sustains a closed displaced intra-articular distal radius fracture. He presents to your clinic and given his age and the fracture characteristics, he is taken for open reduction with volar locking plate

    fixation. After completing instrumentation, radiocarpal screw penetration is best assessed on which fluoroscopic view?

    1. Extended wrist tangential view

    2. Flexed wrist tangential view

    3. PA anatomic tilt view

    4. Radial inclination view

    5. Standard lateral view Corrent answer: 4

    While no single radiographic view can completely rule out intra-articular screw penetration, radial inclination views (most commonly a 22-degree anatomic tilt lateral view) best demonstrates screw penetration of the radiocarpal joint.

     

    Intra-articular screw penetration is a potential complication of distal radius fracture fixation. Due to the complex native geometry, standard PA and lateral fluoroscopic views are not effective in evaluation of the joint reduction, screw length or screw position. For this reason, various supplemental views have been described to facilitate identification of articular incongruity as well as dorsal or intra-articular penetration of screws, either into the radiocarpal or distal radioulnar joint.

     

    Tweet et al. reviewed the most commonly used imaging techniques and views for the evaluation of intra-articular screw penetration during volar plate fixation of distal radius fractures. Of nearly 700 ASSH members surveyed, 87% relied exclusively on fluoroscopy, three-quarters (74%) of which implemented tilt views or rotational fluoroscopy. Cadaveric analysis supported use of a 22-degree lateral projection and rotational fluoroscopy as adjuncts for the detection of intra-articular screw penetration. They concluded that no combination of imaging effectively allowed the detection of all intra-articular screws.

     

    Patel et al. evaluated the utility of supplemental radiography in assessing screw placement in distal radius fracture fixation using volar locking plates. The authors found that supplementation of the standard AP and lateral views with lateral tilt views significantly increased accuracy and confidence.

    Specifically, more acute angles (15 and 23-degree views) provided better visualization of ulnar screws while larger angles (30 degrees) allowed assessment of more radial-sided distal plate screws.

     

    Illustration A is an example of an extended wrist tangential view showing clear visualization of the sigmoid notch and distal radioulnar joint (DRUJ), with intraarticular penetration of the DRUJ shown on the left and extra-articular screws

    free of the DRUJ.

    Illustration B is an example of a flexed wrist tangential view, or “skyline” view, demonstrating no dorsal penetration of the screws, which can be obscured on a true lateral due to Lister’s tubercle.

    Illustration C shows an example of a standard PA view (left) and 11-degree anatomic tilt PA view, allowing better visualization of the articular surface. Illustration D shows a standard true lateral view (left) and 22-degree radial inclination lateral view (right), allowing better visualization of screw relationship to the radiocarpal joint.

     

    Incorrect Answers:

    Answer 1: The extended wrist tangential view, or “sigmoid notch” view, has been described as a technique by which intra-articular penetration of the sigmoid notch can be evaluated.

    Answer 2: The flexed wrist tangential view, or “skyline” view, best demonstrates dorsal screw penetration.

    Answer 3: The PA anatomic tilt view is an 11-degree anatomic tilt view that best demonstrates the degree of articular congruity of the joint surface.

    Answer 5: The standard lateral view can demonstrate the position of a volar plate with respect to the watershed line but has limited utility in evaluating radiocarpal screw penetration due to radial inclination.

     

     

     

     

     

     

     

     

     

     

     

     

     

    OrthoCash 2020

     

  13. A 40-year-old slips on the ice on a wintery Michigan day and sustains a comminuted intra-articular distal radius fracture. Which of the regions on the patient's injury AP radiograph in Figure A, if not addressed properly during surgery, represents a risk for radiocarpal instability?

     

     

     

     

    1. A

    2. B

    3. C

    4. D

    5. E

     

    Corrent answer: 5

     

    It is critical to reduce, buttress, and secure the volar lunate facet (E) to stabilize the radiocarpal joint.

     

    The volar lunate facet, the intermediate column of the distal radius, is the attachment site for the radiolunate ligaments. Because the lunate is the critical structure for force transmission between the radius and the carpus, palmar instability of a displaced volar rim fracture can lead to volar carpal subluxation after ORIF of a distal radius fracture. These typically can be properly buttressed with a standard volar locking plate. However, very distal fractures

    may require fragment specific fixation.

     

    Harness et al. reviewed 7 patients with lunate facet shearing distal radius fractures who lost fixation and subsequently developed volar carpal displacement. Four underwent repeat ORIF, one underwent a wrist fusion, and two declined treatment. The two who declined further treatment had persistent radiocarpal dislocation. After revision fixation, the average wrist extension/flexion was 75%/64% of the contralateral side. The authors underscore the importance of the reduction and securing the volar lunate facet.

     

    Dr. Medoff reviewed the essential radiographic features of distal radius fractures. Distal radius fractures can often be incompletely understood on conventional radiographs. Ulnar variance can be assessed on a PA view and will change based on pronation/supination of the wrist (illustration A). The lateral view is critical to the relationship between the capitolunate articulation and the shaft of the radius, which should form a straight line. In addition, on the lateral, the teardrop angle should be evaluated (Illustration B). An abnormal teardrop angle indicates incongruity at the volar lunate facet.

     

    Figure A demonstrates a comminuted intra-articular distal radius fracture with a displaced lunate facet. In addition, the patient has an ulnar styloid fracture. Illustration A demonstrates the change in position of the ulnar styloid on a PA wrist radiograph. Note the change from neutral (A), supination (B), and pronation (C). Illustration B demonstrates the teardrop angle between the radius-capitolunate articulation and a line through the volar lunate facet. A normal angel is 70 degrees. An abnormal value indicates a displaced volar lunate facet.

     

    Incorrect Answers:

    Answer A: Provided that the DRUJ is stable after distal radius fracture fixation, the ulnar styloid does not typically need to be fixed and does not play a role in radiocarpal stability.

    Answer B: It is important to assess the DRUJ following distal radius fracture fixation, but it is not the primary factor in radiocarpal stability.

    Answer C: Restoration of radial height is important in fixation of distal radius fractures – but is not as critical as the lunate facet in radiocarpal stability following fixation.

    Answer D: The scapholunate ligament is commonly injured during distal radius fractures but does not have a role in radiocarpal stability following ORIF.

     

     

     

     

     

     

     

     

    OrthoCash 2020

     

  14. When considering operative intervention for the injury shown in Figures A and B, which of the following factors has the LEAST correlation with complication rate?

     

     

     

     

     

    1. Skin integrity

    2. Age

    3. Smoking status

    4. Peripheral neuropathy

    5. Diabetes

     

    Corrent answer: 2

     

    In patients with an unstable ankle fracture, age has not shown to have an effect on complication rate.

     

    An increasing number of elderly patients retain an active lifestyle with high functional demands. Cast immobilization for highly unstable ankle fractures (especially inherently unstable pronation-abduction fractures that are frequently seen in the elderly) leads to malunion or nonunion rates between 48% and 73%. Therefore, in the absence of severe comorbidities, the principal indications for surgery should not differ from those in younger patients.

     

    Herscovici et al. evaluated 243 elderly patients with high-energy foot and ankle injuries, 73 of which had ankle fractures, who underwent surgical fixation.

    They report on their preferred approach to treatment of these patients. Their article does not report on any outcome variables but states that, in their expert opinion, elderly patients should be considered for operative intervention as a young patient would.

     

    Pagliaro et al. analyzed outcomes after surgical treatment of unstable ankle fractures in 23 patients who were at least 65 years old. They found that open reduction and internal fixation of unstable ankle fractures in geriatric patients is an effective treatment that leads to a complication rate comparable to the general population.

     

    Figure A and B demonstrate AP and lateral radiographs of a trimalleolar ankle fracture.

     

    Incorrect Answers:

    Answer 1, 3-5: All of these factors lead to an increased risk of complications for unstable ankle fractures.

     

     

     

    OrthoCash 2020

     

  15. Which of the following statements is true regarding the outcomes when using a high-pressure pulse lavage irrigation system versus

    simple low-pressure bulb irrigation for the treatment of the open fracture shown in Figure A?

     

     

     

     

    1. It is more cost effective when including the cost of reoperation

    2. There is an increased rate of associated nerve injury

    3. There is an increased rate of primary wound healing problems

    4. There is increased rate of infection

    5. There is no difference in union rates Corrent answer: 5

    There is no difference in union rates when either high-pressure (pulse lavage) or low-pressure (bulb) irrigation with saline is used for open tibia fractures.

     

    The mainstay of early treatment of open fractures includes debridement and irrigation along with IV antibiotics. Multiple trials have looked at the most efficacious and cost-effective irrigation method both in terms of the type of solution and irrigation method. With regard to the irrigation method, recent studies have shown simple bulb irrigation has the same union rate as more expensive high-pressure pulse lavage systems. With regard to the type of solution, recent studies have shown castile soap actually leads to a decreased rate of primary wound healing problems as compared using an irrigation solution with antibiotics.

     

    Bhandari et al. in the FLOW study (Fluid Lavage of Open Wounds), a multicenter randomized trial demonstrated that the rates of reoperation were similar regardless of irrigation pressure, a finding that indicates that very low pressure is an acceptable low-cost alternative for the irrigation of open fractures.

    Anglen et al. demonstrated that the use of a nonsterile liquid soap additive (castile soap) was at least as effective as the use of bacitracin with regards to the rate of postoperative infection and fracture healing, but showed a significant decrease in problems with soft-tissue healing.

     

    Bhandari et al. as a follow up by the FLOW group in the study above analyzed 2551 patients to look for risk factors for reoperation. The rates of reoperation were similar regardless of irrigation pressure, a finding that indicates that very low pressure is an acceptable low-cost alternative for the irrigation of open fractures. The reoperation rate was higher in the castile soap group than in the saline group.

    Figure A is a clinical photograph demonstrating a Type IIIB open tibia fracture. Incorrect Answers:

    Answers 1: The high-pressure irrigation system is more expensive and has not

    been shown to decrease reoperation rates.

    Answer 2: There is not a higher risk of nerve injury with high-pressure irrigation.

    Answer 3: There has been no difference in regards to wound healing problems when comparing high and low-pressure irrigation.

    Answer 4: No difference in postoperative infection has been found, regardless of high or low-pressure irrigation.

     

     

     

    OrthoCash 2020

     

  16. A 71-year-old man presents after 9 months of functional bracing for a closed right humeral shaft fracture. Current radiographs are shown in Figure A. Infectious workup was negative. Appropriate next steps in evaluation will most likely reveal which of the following?

     

     

     

    1. Vitamin D deficiency

    2. Low testosterone

    3. Iron deficiency

    4. Thyroid deficiency

    5. Magnesium deficiency

     

    Corrent answer: 1

     

    Metabolic/endocrinologic workup for an atrophic nonunion following appropriate treatment for a humeral shaft fracture is likely to reveal a vitamin D deficiency.

     

    Nonunion of the humeral shaft occurs approximately 10% of the time following open or closed treatment. In the case of nonunion, there is likely a lack of mechanical and biologic factors. Biologic workup for nonunion should include ruling out infection followed by an evaluation for metabolic and endocrine abnormalities.

     

    Brinker et al. reviewed 37 low-energy fractures that went on to nonunion. These patients were evaluated by clinical endocrinologists for evaluation of metabolic abnormalities. Thirty-one of the 37 patients (84%) had a metabolic issue, with 68% (25 of 37 patients) having a Vitamin D deficiency. They conclude any patient with a nonunion should be referred to an endocrinologist for metabolic screening and treatment.

     

    Kuo et al. looked at the compliance of 155 patients treated for osteoporosis

    shortly after sustaining a fragility fracture. They found 47% had a prior fracture with only 26% having a prior BMD and 15% on calcium and vitamin D. They concluded that osteoporotic treatment provided at the time of fracture leads to long-term compliance in 90% and 73% of osteoporotic and osteopenic patients, respectively.

     

    Figure A shows a radiograph of the right humerus demonstrating a nonunion of a humeral shaft fracture. Illustration A shows postoperative radiograph after open reduction internal fixation with compression plating.

     

    Incorrect Answers:

    Answer 2: Low testosterone would be possible given the patient demographics but would not be included in the appropriate next steps in the evaluation of this nonunion.

    Answer 3: Iron deficiency may be present but would not be the next best laboratory study in evaluating a nonunion.

    Answer 4 & 5: Metabolic and endocrinologic workup is the next appropriate step; however, thyroid and magnesium deficiencies are less common than a vitamin D deficiency.

     

     

     

     

     

    OrthoCash 2020

     

  17. A 62-year-old woman falls off a bike and sustains the injury shown in Figure A. She presents to clinic for her 6-month follow-up appointment and reports persistent pain. Current imaging is shown in Figure B. Which of the following factors is not a risk factor to the development of this patient’s diagnosis?

     

     

     

     

     

     

     

    1. Shortening of 3cm

    2. Comminution

    3. Sling immobilization as opposed to figure-of-eight brace

    4. Female gender

    5. Advanced age

     

    Corrent answer: 3

     

    This patient presents with a symptomatic non-union of a midshaft clavicle fracture (classic "z-type" fracture for it's shape), which is not associated with type of immobilization (sling vs. figure-of-eight brace).

     

    Clavicle fractures treated non-operatively are at risk for complications such as non-union, poorer cosmesis, and decreased shoulder strength and endurance. The risk of non-union, in particular, ranges from 1-5% and has been shown to increase with shortening > 2cm, comminution, female gender, and advanced age. Open reduction internal fixation of clavicle fractures has also shown to have increased union rates compared to those treated non-operatively but

    presents its own risks such as hardware complications and infections.

     

    Robinson et al. performed a prospective observational cohort study to evaluate the prevalence of and risk factors for non-union of clavicle fractures treated non-operatively. They found that the risk of non-union was significantly increased by advancing age, female gender, displacement of the fracture, and the presence of comminution.

     

    Lazarides et al. performed a retrospective study to determine the relevance of shortening and clinical outcomes for midshaft clavicle fractures treated nonoperatively. They found that final clavicular shortening of > 18mm in male patients and of > 14mm in female patients was significantly associated with an unsatisfactory result.

     

    The Canadian Orthopaedic Trauma Society performed a multicenter prospective randomized clinical trial to compare outcomes of clavicle fractures treated non-operatively versus operatively with plate fixation. They found that operative fixation of a displaced clavicular shaft fracture results in improved functional outcome and a lower rate of malunion and nonunion compared with nonoperative treatment at one year of follow-up.

     

    Figure A is an AP radiograph demonstrating a displaced midshaft clavicle fracture. Figure B is an AP radiograph demonstrating non-union of a midshaft clavicle fracture.

     

    Incorrect Answers:

    Answers 1, 2, 4, 5: Each of these factors is associated with an increased risk of non-union.

     

     

     

    OrthoCash 2020

     

  18. Which of the following statements about polytrauma and organ failure is most correct when comparing children to adults?

    1. Adults have a more robust initial inflammatory response, affecting the pulmonary system first

    2. Adults have a dampened immune response, affecting all organ systems simultaneously

    3. Adults have a more robust initial inflammatory response, affecting all organ systems simultaneously

    4. Children have a more robust initial inflammatory response, affecting all organ systems simultaneously

    5. Children have a dampened immune response, affecting the pulmonary system first

    Corrent answer: 1

     

    Adults have been shown to have a robust initial inflammatory response that affects the pulmonary system first.

     

    There is a difference in the systemic response to trauma between children and adults. Children initially have a dampened systemic immune response. They do, however, have a robust local inflammatory response that causes organ failure and affects all organ systems simultaneously. Adults have a robust systemic response with organ failure occurring 2-3 days after injury. The pulmonary system is usually the first organ system affected in adults.

     

    Wood et al. performed a review of the inflammatory response to injury in children. They found that the inflammatory response to injury in children is functionally and mechanistically unique. The incidence of multi-organ failure in children is rare, but affects all organ systems simultaneously when it does occur.

     

    Pandya et al. performed a review of the pediatric polytrauma patient. They report that the orthopaedic surgeon plays an important role in managing hemodynamic instability, vascular insult, and neurologic damage in the child with multiple injuries. They recommend that, if surgical fixation is deemed urgent in the pediatric patient, it can be performed in the early period of multi-system insult.

     

    Incorrect Answers:

    Answers 2 & 3: Adults initially have a robust, systemic inflammatory response. When organ failure does occur, the pulmonary system is usually the first affected.

    Answers 4 & 5: Children initially have a dampened systemic inflammatory response. When organ failure occurs, it affects all organ systems simultaneously.

     

     

     

    OrthoCash 2020

     

  19. A 63-year-old patient presents with right ankle pain after a fall down four stairs. Figures A and B are the radiographs of the injury. The patient reports a history of diabetes mellitus type 2 and peripheral neuropathy with a most recent hemoglobin A1c of 9.8. The injury is

    reduced and placed in a well-padded bivalved cast. The patient is then discharged with outpatient follow-up without DVT prophylaxis. Twelve weeks later, the patient presents to the clinic for the first time in the same bivalved cast. The has remained non-weight bearing and the bottom of the cast confirms this. What is the expected outcome at this point?

     

     

     

     

    1. Charcot arthropathy

    2. Diabetic foot ulcer

    3. Deep vein thrombosis

    4. Elevation of A1c

    5. Fracture displacement

     

    Corrent answer: 5

     

    The patient is a diabetic with poorly controlled blood sugar and peripheral neuropathy presenting with a bimalleolar ankle fracture with a loss to follow-up for 3 months. All of the answer choices are potential complications with nonoperative treatment, but fracture displacement is the most likely to occur.

     

    Ankle fractures in diabetic patients are associated with increased complications compared to nondiabetic counterparts. The glycation damage to cellular

    components as well as the decreased blood flow with microangiopathic changes results in substantially decreased bone healing. Patients diabetic comorbidities, such as retinopathy, nephropathy, vasculopathy, and neuropathy have a higher rate of wound complications, deep infections, malunion, nonunion, and Charcot arthropathy. Due to the delayed healing, surgical treatment with supplemental fixation is recommended.

     

    Lovy et al. performed a retrospective review of 28 diabetic ankle fractures with 20 patients being treated nonoperatively and 8 patients treated operatively initially. Nonoperative treatment was associated with a 21-fold increase in complication risk. Delayed surgery (>3 weeks from injury) for nonunion or malunion was associated with greater complications than immediate ORIF. The authors concluded that nonoperative treatment for diabetic ankle fractures was associated with unacceptably high consequences.

     

    Wukich et al. reviewed the treatment of ankle fractures in diabetic patients. Nonoperative treatment was recommended as a viable option for nondisplaced isolated unimalleolar and stable ankle fractures. Additionally, they proposed diabetic patients without diabetic comorbidities can be surgically treated with standard internal fixation constructs as they have similar outcomes. Finally, despite surgical treatment having a higher rate of complications than nondiabetic patients, operative management is more likely to result in a functional lower extremity in displaced and unstable ankle fractures.

     

    Chaudhary et al. performed a review of ankle fractures in diabetic patients and concluded that diabetic ankle fractures are at high risk for fracture displacement and delayed union, even with surgical treatment. The authors recommend supplemental fixation with external fixation or multiple syndesmotic screws. Postoperatively patients should undergo prolonged immobilization, 2-3 times that of nondiabetics.

     

    Figure A demonstrates an AP radiograph of a bimalleolar ankle fracture. Figure B demonstrates a lateral radiograph of a bimalleolar ankle fracture.

     

    Incorrect Answers:

    Answer 1: Charcot arthropathy occurs in diabetic ankle fractures, especially in neuropathic patients treated nonoperatively and with delayed treatment.

    However, fracture displacement is more likely to occur.

    Answer 2: Diabetic foot ulcers are a potential complication, especially in neuropathic patients. However, with a well-padded cast and the patient adhering to non-weight bearing recommendations, this is not as likely as fracture displacement.

    Answer 3: Deep vein thrombosis is a potential complication of fractures in the

    lower extremity, but the risk is typically higher in surgically managed patients. Answer 4: Non-operative treatment of ankle fractures in diabetic patients has not been associated with increases in A1c. However, with non-weight bearing activities the patient is likely to have poorer control of blood glucose, but the incidence of this has not been characterized.

     

     

     

    OrthoCash 2020

     

  20. Regarding Syme amputations, which of the following is true?

     

    1. It does not require a patent tibialis posterior artery

    2. It is dependent on a stable heel pad

    3. It is less energy efficient than a midfoot amputation

    4. The primary complication is an equinus deformity

    5. It is also known as a hindfoot amputation Corrent answer: 2

    Syme amputations rely on a stable heel pad.

     

    Syme amputations, or ankle disarticulations, are the most proximal type of ankle/foot amputations but are more energy efficient than midfoot amputations. A successful Syme amputation requires a patent tibialis posterior artery and a stable heel pad. The two most common problems are skin sloughing from a compromised vascular supply and migration of the heel pad due to instability. A hypermobile heel pad can cause difficulty with prosthesis wear and damage to the soft tissues, eventually leading to failure.

     

    Finkler et al. performed a retrospective study on the long-term outcomes of Syme amputations. They found that patients who underwent Syme's ankle disarticulation amputation appeared to fare better than similar patients with transtibial amputations. These patients required less rehabilitation and achieved improved levels of functional independence as demonstrated by patient-reported outcome measures.

     

    Pinzur et al. performed a retrospective study on Syme ankle disarticulation in patients with diabetes. They found that patients managed with a Syme ankle disarticulation had better ambulatory function and survival compared to transtibial amputations. They concluded that the results of this retrospective review support the value of Syme ankle disarticulation in diabetic patients with infection or gangrene.

     

    Illustration A is an example of a Syme amputation.

    Incorrect Answers:

    Answer 1: Syme amputations require a patent tibialis posterior artery. Answer 3: Syme amputations are more energy efficient than midfoot amputations.

    Answer 4: Equinus deformites are not a complication of Syme amputations. Answer 5: Syme amputations are also known as ankle disarticulations, not hindfoot amputations.

     

     

     

     

     

     

    OrthoCash 2020

     

  21. A 65-year-old patient, who is a current smoker, presents with left hip pain after a fall from standing height and is unable to bear weight. The patient's radiograph is shown in figure A. The use of a sliding hip screw construct has been shown to have what benefit compared to multiple cannulated screws?

     

     

     

    1. Decreased fracture shortening

    2. Decreased non-union rate

    3. Decreased implant failure

    4. Lower reoperation rates

    5. Lower rate of avascular necrosis Corrent answer: 4

    The patient is a current smoker presenting with a left displaced basicervical femoral neck fracture. The use of a sliding hip screw (SHS) in this fracture pattern in patients that are current smokers has been associated with decreased reoperation rates compared to multiple cannulated screws (MCS).

     

    Femoral neck fractures are highly morbid injuries and are the most expensive fractures to treat. Treatment options include MCS fixation, SHS, hemiarthroplasty, and total hip arthroplasty. Hemiarthroplasty is preferred in the low demand and elderly population. In younger patients with higher activity levels, reduction and internal fixation with MCS or SHS provide bone-preserving treatment options. Total hip arthroplasty is preferred in patients

    >65 years of age with higher activity levels.

     

    The Fixation using Alternative Implants for the Treatment of Hip fractures

    study group (FAITH) performed a multicenter randomized control trial that included 1108 patients treated with either dynamic SHS or MCS. Fixation with SHS was associated with increased rates of avascular necrosis compared to MCS, 9% vs 5%, but was not significant. Overall, there were no differences in reoperation rates or union rates between the two groups. However, subgroup analysis revealed lower reoperation rates with SHS in displaced fractures, basicervical fractures, and in current smokers.

     

    Zielinski et al. 2013 performed a subanalysis of 76 patients from the FAITH study examining femoral neck shortening, WOMAC scores, gait analysis, and hip muscle strength in patients treated for femoral neck fractures. Increased fracture shortening was observed with increased patient weight, age, and higher Pauwel's classifications. There was no comparison between implant type and gait outcomes.

     

    Lee et al. 2008 performed a retrospective review of femoral neck fractures in 90 patients undergoing either minimally invasive SHS, MCS, or conventional SHS. The authors found that conventional DHS was associated with significantly larger incisions, greater blood transfusion requirements, and longer hospital stays compared to minimally invasive SHS and MCS. The authors recommended minimally invasive SHS due to lower soft tissue complications.

     

    Figure A demonstrates an AP hip radiograph with a right femoral neck fracture. Illustration A depicts an SHS implant providing internal fixation for a femoral neck fracture. Illustration B depicts MCS fixation of a femoral neck fracture.

     

    Incorrect Answers:

    Answer 1: Fracture shortening has not been found to occur at lower rates with SHS or MCS.

    Answer 2: Non-union rates are not significantly different between the two fixation methods.

    Answer 3: Implant failure has not occurred differently between SHS and MCS. Answer 5: Avascular necrosis in the FAITH was found to occur at a higher rate in the SHS group, but this finding was not significant.

     

     

     

     

     

     

     

     

    OrthoCash 2020

     

  22. Which of the following will most likely result with the use of a fracture table when treating the injury shown in Figures A and B?

     

     

     

    1. Internal malrotation deformity

    2. External malrotation deformity

    3. Recurvatum deformity

    4. Varus deformity

    5. Valgus deformity

     

    Corrent answer: 1

     

    The fracture table has been shown to induce an internal malrotation deformity when it is used for treating femoral shaft fractures.

     

    Intramedullary nailing with reaming of the bone and static locking of the nail is the treatment of choice for femoral shaft fractures in adults. The fracture table has been used with excellent results. Compared to manual traction, the fracture table provides an advantage as only one assistant is necessary.

    However, it may limit access to the patient if other body areas are injured. Additionally, the fracture table has been shown to induce an internal malrotation deformity, which results from internally rotating the limb to facilitate access to the starting point and to improve visualization of the hip in the lateral plane. Rotation may also be difficult to assess clinically once the limb is positioned and draped.

     

    Kelly et al. performed a retrospective study to determine complications associated with the use of the fracture table in pediatric femoral fractures.

    They found 8 palsies related to positioning. They also found that patients who developed nerve palsies were significantly heavier. They ultimately concluded that the fracture table is safe for fixation but every effort should be made to minimize time in traction to avoid iatrogenic nerve palsy, particularly in heavier children (> 80 kg).

     

    Stephen et al. performed a randomized study comparing the fracture table and manual traction when nailing femoral shaft fractures. They found that internal malrotation was significantly more common when the fracture table was used. Operative time was also greater for the fracture table group. There were no other differences in complications. They concluded that manual traction is an effective technique that decreases operative time and improves the quality of reduction.

    Figures A and B are AP radiographs demonstrating a femoral shaft fracture. Incorrect Answers:

    Answers 2, 3, 4, & 5: Internal malrotation has been shown to be a risk when

    using a fracture table to treat femoral shaft fractures by intramedullary nailing.

     

     

     

    OrthoCash 2020

     

  23. A 35-year-old patient is involved in a motor vehicle collision and sustains multiple injuries, including an APC pelvis fracture. He is stabilized following placement of a pelvic binder and receiving blood products as part of a massive transfusion protocol. He is subsequently taken to radiology for radiographs and a chest, abdomen, and pelvis CT with 4mm cuts. What effect will these modalities have on the radiographic appearance of his pelvis fracture and what further intervention should be performed?

    1. Make the injury look more severe; obtained radiographs with the binder removed

    2. Make the injury look less severe; perform fluoroscopic examination under anesthesia

    3. Accurate representation of injury severity; no further studies needed

    4. Make the injury look less severe; repeat CT with 0.625 mm cuts

    5. Make the injury look more severe; obtain Judet views of the pelvis Corrent answer: 2

    Application of a pelvic binder and CT scans prior to standard AP pelvis radiographs results in an underestimation of pelvic ring injury severity. Since this patient was hemodynamically unstable and required a massive transfusion,

    removal of the pelvic binder is not recommended and further injury stability should be evaluated by fluoroscopic examination under anesthesia.

     

    Pelvic ring fractures are high energy injuries that usually follow motor vehicle collisions. There is a high rate of hemodynamic instability with these injuries due to the increase in pelvic volume and abundant venous bleeding. Proper identification of injury severity in the initial evaluation is crucial to ensure appropriate definitive treatment. Ideally, if the patient is hemodynamically stable, AP pelvis radiographs without a pelvic binder have been found to be effective at identifying unstable pelvic injuries.

     

    Fagg et al. performed a retrospective study of 97 patients presenting with pelvic ring fractures requiring operative stabilization. The authors found that 7% of patients that initially had binder-on imaging were found to have unstable fractures that were not identified on the original images. The authors suggested that pelvic CT scans with the binder on are insufficient in determining the severity of pelvic fractures.

     

    Swartz et al. performed a retrospective study of 43 patients presenting with pelvic ring fractures that received AP pelvic radiographs, pelvic binder, pelvic CT, and fluoroscopic stress examination in sequential order. They found that pre-binder radiographs had a greater sensitivity at detecting unstable fractures than pelvic CT with a binder. The authors concluded that placement of a binder prior to imaging can underestimate the severity of the pelvic injury and in cases where a binder is placed prior to imaging a fluoroscopic examination under anesthesia can be useful in detection.

     

    Gibson et al. performed a retrospective review of 72 patients that presented with pubic diastasis requiring internal fixation. The authors found that pelvic CT scans without a pelvic binder underestimated the diastasis by 6.6mm compared to AP pelvic radiographs. The authors recommend obtaining radiographs prior to performing a CT, as underestimation of pubic diastasis can alter definitive management decisions.

     

    Incorrect Answers:

    Answer 1: Placement of a pelvic binder prior to radiographs and early CT scans underestimate the severity of pelvic ring injuries. In this case, the patient is hemodynamically unstable and removing the binder would likely result in increased pelvic bleeding by destabilizing clots.

    Answer 3: The CT scan with the binder in place will underestimate the injury severity in this patient. In order to provide proper definitive care, the severity of the injury must be determined.

    Answer 4: The current study has underestimated the degree of pelvic

    instability. However, decreasing the cut thickness on CT will be unlikely to better determine the degree of pelvic ring instability.

    Answer 5: The current injuries are underestimated by the current image study regimen with binder placement. Judet views, especially with the binder in place, will not enhance the detectability of pelvic ring instability.

     

     

     

    OrthoCash 2020

     

  24. A 26-year-old male with a BMI of 37 is involved in a motor vehicle collision and requires extrication. During evaluation in the trauma bay, he becomes hemodynamically unstable and is found to have the injury shown in Figure A, as well as an associated bladder injury. Which of the following statements regarding the patient's injury is true?

     

     

     

     

    1. His male gender places him at a lower risk for post-operative infection

    2. His BMI places him at a higher risk for post-operative infection

    3. The mechanism of his injury was likely a lateral compression force

    4. The internal pudendal artery is the most likely source of arterial hemorrhage

    5. Pelvic binder placement is contraindicated in this patient given his bladder injury

    Corrent answer: 2

     

    The patient has an APC pelvis injury. Increased BMI has been associated with a higher risk of infection after surgical intervention of pelvic and acetabular fractures.

     

    Pelvic fractures result from high-energy mechanisms and frequently require surgical intervention. When performing open procedures in the pelvis, the risk of infection is of concern. Elevated BMI has been shown to be the most

    important risk factor for infection. Other risk factors include prolonged operative time, prolonged ICU stay, increased red blood cell transfusions, leukocytosis, Morel-Lavallée lesion, and associated genitourinary and abdominal trauma. Preoperative angioembolization as a risk factor for SSI remains controversial but is thought to be associated with embolization and not with angiography itself.

     

    Iqbal et al. performed a retrospective study on surgical site infection following fixation of acetabular fractures. They found that prolonged operation time, increased BMI, prolonged ICU stay, larger amount of packed red blood cell transfusions, and associated genitourinary and abdominal trauma were associated with surgical site infection following acetabular fracture fixation.

    They concluded that measures should be undertaken to attenuate the chances of surgical site infection in this major surgery by considering the risk factors significantly associated with it.

     

    Sagi et al. performed a retrospective study on the risk for deep postoperative wound infection after pelvic and acetabular surgery. They found that obesity, obesity plus leukocytosis, and preoperative angioembolization were strong predictors of postoperative infection.

     

    Firoozabadi et al. performed a retrospective study to evaluate the impact of pelvic embolization on postoperative infection rate after acetabular fracture fixation. They found that, despite previous reports of high infection rates after pelvic embolization, the deep infection rate was only 4% after embolization in their cohort. They therefore state that concerns for higher rates of infection are not substantiated, and pelvic embolization should be performed when indicated.

     

    Figure A is an AP radiograph of the pelvis demonstrating symphyseal widening and left SI joint widening.

     

    Incorrect Answers:

    Answer 1: Gender has not been found to affect the risk of post-operative infection.

    Answer 3: The patient's mechanism of injury was likely an anterior-posterior compression force, not a lateral compression force.

    Answer 4: The superior gluteal artery is the most likely source of arterial hemorrhage in patients with APC pelvic fractures, while the internal pudendal artery or obturator artery is the most likely source of hemorrhage in patients with LC pelvic fractures.

    Answer 5: Urogenital injury is not a contraindication for pelvic binder placement.

     

     

    OrthoCash 2020

     

  25. A 70-year-old female with a history of osteoporosis complains of vague left hip pain. She does not complain of right hip pain. Her imaging is shown in Figures A & B. What is the next best step in treatment?

     

     

     

     

    1. Observation and pain control

    2. Bone scan

    3. Intramedullary nailing of the left femur

    4. Intramedullary nailing of bilateral femurs

    5. Referral to an orthopaedic oncologist Corrent answer: 3

    This patient has bisphosphonate-related changes of her femurs. Given that the left side is the only symptomatic side, it should be prophylactically nailed.

     

    Bisphosphonates have been implicated in causing atypical low-energy femur fractures. Radiographic clues for this impending injury include lateral cortical thickening (beaking), varus remodeling of the femur, and increased diaphyseal cortical thickness. When a fracture occurs, it usually occurs in the subtrochanteric region. The fracture patterns are usually transverse or short-oblique in orientation. There is also a lack of comminution. The ideal treatment for these injuries is an intramedullary nail, ensuring to restore native alignment and avoiding varus.

     

    Vasikaran performed a review of low-energy femoral fractures with prolonged bisphosphonate use. He reported that long-term bisphosphonate therapy is thought to lead to profound suppression of bone turnover, increased

    microdamage accumulation, and alteration of bone tissue mineral properties as well as possibly bone tissue composition by altering collagen cross-linking, all of which can in turn cause brittleness of bone and propensity to fracture with minimal trauma. He concludes that, if a patient becomes symptomatic or sustains a fracture, the cessation of bisphosphonate therapy should be considered. If a fracture has not occurred and the patient is having prodromal symptoms, prophylactic intramedullary nailing should be performed.

     

    Balach et al. performed a review on atypical femur fractures associated with disphosphonate use. They report that following the diagnosis of a complete or impending atypical femur fracture, discontinuation of diphosphonates is recommended to help promote a more favorable fracture healing environment. They conclude that surgical treatment should allow endochondral fracture healing such as an intramedullary nail.

     

    Figures A and B are AP radiographs of the left and right proximal femurs demonstrating lateral cortical beaking, cortical thickening, and mild varus remodeling.

     

    Incorrect Answers:

    Answer 1: Observation and pain control is not appropriate in this case as the patient has an impending fracture.

    Answer 2: A bone scan is not necessary in this case as the diagnosis can be made from radiographs.

    Answer 4: Intramedullary nailing of both sides is not necessary at this time as the right side is asymptomatic.

    Answer 5: Referral to an orthopaedic oncologist is not necessary for treating these injuries.

     

     

     

    OrthoCash 2020

     

  26. A 26-year-old male recreational basketball player sustained an ankle injury 6 months prior. He continues to complain of ankle pain and instability. Current imaging is shown in Figures A & B. Imaging of the proximal fibula is unremarkable and there is little concern for syndesmotic injury. What is the next best step in treatment?

     

     

     

     

     

    1. Open reduction and internal fixation (ORIF) with autograft

    2. Fragment excision

    3. Obtain stress radiographs

    4. Physical therapy and management of symptoms

    5. Percutaneous skeletal fixation Corrent answer: 1

    This patient sustained a medial malleolar fracture that went on to nonunion. The next best step in treatment would be ORIF with autograft.

     

    Historically, nonunion rates for malleolar fractures are somewhere between 5 and 30%. Nonunion following rotationally unstable ankle fractures can have detrimental effects on patient outcomes, leading to continued debilitating pain, instability, and late posttraumatic osteoarthritis. Patients with symptomatic nonunion should have surgical treatment. Fragment excision may be possible in patients with small distal fractures. However, if the fragment is large and well-preserved, ORIF with bone graft augmentation is the treatment of choice.

     

    Khurana et al. performed a study to evaluate the outcomes following operative treatment of nonunited rotational distal fibula and medial malleolar fractures. They identified 17 patients. All patients were treated with open reduction with plate and screw fixation or screw fixation alone. Adjunctive grafting was performed in all but 2 cases. They found that after a mean of 33.9 months all nonunions had resolved. They conclude that patients who have surgery for nonunion will have outcomes similar to patients who undergo surgery for acute ankle fractures.

     

    Capogna and Egol wrote a guide regarding the treatment of nonunions after malleolar fractures. They report that most medial malleolar nonunions are addressed by fixation with partially threaded cancellous screws. They suggest that osteopromoting materials or bone graft should be added before reduction. They conclude that fixation with two partially-threaded screws is sufficient for reliable fracture healing.

     

    Figure A is an AP radiograph demonstrating a medial malleolar nonunion. Figure B is a coronal CT slice demonstrating a medial malleolar nonunion. Illustrations A and B are AP and lateral radiographs, respectively, demonstrating a medial malleolar nonunion that has been stabilized with 2 cannulated screws. Bone graft augmentation was also performed. Illustrations C and D are AP and lateral radiographs, respectively, demonstrating a healed medial malleolar nonunion.

     

    Incorrect Answers:

    Answer 2: Fragment excision is not appropriate in this case as the fragment is large and well-preserved.

    Answer 3: Stress radiographs are unnecessary in this case as the patient is symptomatic and the diagnosis is made with the available imaging.

    Answer 4: Non-operative treatment of symptomatic malleolar nonunions is not recommended.

    Answer 5: Percutaneous skeletal fixation in the presence of a nonunion is not recommended as the fracture site cannot be debrided and it will not be possible to augment with bone graft.

     

     

     

     

     

     

     

     

    OrthoCash 2020

     

  27. A 76-year-old left-handed female presents to your office after a fall with the injury seen in Figure A. When treating this fracture with hemiarthroplasty, which of the following is the most important for a successful outcome?

     

     

     

    1. Anatomic healing of the tuberosities

    2. Stem retroversion of 25 degrees

    3. Surgical fixation within 24 hours

    4. Anterior superior labral repair

    5. Age over 75

     

    Corrent answer: 1

     

    When performing a hemiarthroplasty for a displaced four-part proximal humerus fracture, the best predictor of a successful outcome is anatomic healing of the tuberosities.

     

    Although most proximal humerus fracture can be treated nonoperatively, especially in elderly low-demand patients, some debate remains regarding the use of a hemiarthroplasty vs. reverse total shoulder arthroplasty. When performing a hemiarthroplasty for a displaced four-part proximal humerus fracture, it is of the utmost importance to achieve a stable and anatomic reduction of the tuberosities. Studies have shown malpositioned or migrated tuberosities leads to stiffness, weakness, and persistent pain. Restoration of humeral height and version is also important for successful outcomes of hemiarthroplasty.

     

    Rangan et al., in the PROFHER trial, studied the outcomes of patients with displaced proximal humerus fractures treated with surgical vs. nonsurgical intervention. Of note, in the surgical group, 83% of patients were treated with ORIF, 9% were treated with hemiarthroplasty, and the remainder were treated with some other fixation method. They found no difference at 2 years between the surgical and nonsurgical groups with regard to outcome scoring, complications from surgery or nonoperative care, secondary surgery, new therapy requirement, or mortality. They concluded the current increase in surgery in patients with displaced proximal humerus fractures is not supported by this data.

    Boileau et al. followed 66 patients with displaced proximal humerus fractures treated with hemiarthroplasty to evaluate for clinical and radiographic parameters that could lead to worse outcomes. They found that 50% of patients had malpositioning of the greater tuberosity, which correlated with unsatisfactory results, superior migration of the prosthesis, stiffness or weakness, and persistent pain. They concluded that the primary goal of arthroplasty for proximal humerus fractures should be obtaining anatomic tuberosity healing.

    Figure A shows a displaced four-part proximal humerus fracture. Incorrect Answers:

    Answer 2: Although appropriate retroversion is important for optimal function,

    anatomic healing of the tuberosities is felt to contribute more to successful outcomes.

    Answer 3: Surgical timing has not been reported to be associated with improved outcomes in this fracture pattern.

    Answer 4: Anterior superior labral repair would not be indicated and would be of little benefit in this setting.

    Answer 5: Age over 75 has been shown to predict worse tuberosity healing as subsequent worse outcomes.

     

     

     

    OrthoCash 2020

     

  28. A 25-year-old man sustains a left leg injury during a motorcycle accident. A radiograph is provided in Figure A. The fracture is treated in a minimally invasive manner with a lateral locking plate and percutaneous screw fixation. A post-operative radiograph is provided in Figure B. Which of the following complications has been associated with this fixation construct?

     

     

     

     

     

     

    1. Compartment syndrome

    2. Common peroneal nerve injury

    3. Superficial peroneal nerve injury

    4. Deep peroneal nerve injury

    5. Popliteal artery injury Corrent answer: 3

    Figures A and B demonstrate pre and post-operative radiographs of a comminuted proximal tibia fracture treated with a long locking plate. Compartment syndrome is a significant complication associated with this fracture. However, there is no greater risk of compartment syndrome with use of this specific construct plate. Long lateral locking plates have been shown to put the superficial peroneal nerve at risk with insertion of distal locking screws. Use of a larger incision and careful dissection down to the plate in this region may minimize the risk of damage to the nerve.

     

    Deangelis et al performed a cadaveric study evaluating 14 extremities and the relationship between the superficial peroneal nerve to the percutaneous screws of the 13-hole proximal tibia Less Invasive Stabilization System (LISS) plate.

    The average distance from the superficial peroneal nerve to the center of holes 11, 12, and 13 was 10.0 mm, 6.8 mm, and 2.7 mm, respectively. In 12 of 14 legs (86%), the superficial peroneal nerve was 5.0 mm or less from the center of hole 13.

     

    Cole et al reviewed his experience of 77 proximal tibia fractures treated with the LISS plate. He reported a high rate of union (97%) with only 2 infections, 2 nonunions, 1 nerve palsy, and 8 patients with an angular malunion approaching 10 degrees.

     

     

     

    OrthoCash 2020

     

  29. A 48-year-old male returns to your office 8 months after sustaining a proximal humerus fracture that was successfully treated nonoperatively. However he is still having persistent anterior shoulder/arm pain that worsens with most activities. He has not done any physical therapy nor received a corticosteroid injection. What nerve innervates the structure identified by the red arrow in Figure A, what is its primary function, and where does it originate from?

     

     

     

    1. Subscapular nerves, shoulder internal rotation, ventral scapula

    2. Musculocutaneous nerve, elbow pronation, coracoid

    3. Axillary nerve, shoulder external rotation, dorsal scapula

    4. Musculocutaneous nerve, elbow supination, supraglenoid tubercle

    5. Suprascapular nerve, shoulder abduction, dorsal scapula Corrent answer: 4

    The figure demonstrates a rupture of the long head of the biceps brachii (LHB). The biceps primarily supinates the elbow via innervation from the musculocutaneous nerve.

     

    The long head of the biceps originates in the shoulder at the supraglenoid tubercle, traverses through the bicipital groove, forms the muscle belly of the biceps brachii with the short head from the coracoid process. This then inserts distally onto the radial tuberosity in the proximal radius. This consistent anatomy allows it to be used as an anatomical reference during repair of comminuted proximal humerus fracture patterns.

     

    Tosounidis et al. compared the histological appearance of LHB tendons from individuals undergoing hemiarthroplasty for proximal humerus fractures to control cadaveric specimens. They noted a significant rate of degenerative lesions within the LHB in the fracture cohort. They use this as a basis for performing tenotomy/tenodesis of the LHB during fracture surgery.

    Soliman et al. performed a prospective randomized trial of individuals undergoing shoulder hemiarthroplasties who either received an LHB tenodesis or it was neglected. They reported improved Constant and PRO scores for those who received a tenodesis while there was no strength difference noted. Based on this they concluded biceps tenodesis during hemiarthroplasty is indicated.

     

    Nho et al. reviewed the pathology and management of biceps pathology. They note the LHB can be the primary isolated pathologic process but more often LHB lesions are in conjunction with other intra-articular pathology. Various methods exist do address biceps pathology, ranging from simple tenotomy to open or arthroscopic tenodesis.

     

    Figure A is a fluid-sensitive axial MR sequence with the arrowhead pointing to the LHB within the groove.

     

    Incorrect Answers:

    Answer 1: This describes the subscapularis muscle. Answer 2: The biceps is responsible for elbow supination. Answer 3: This describes the teres minor muscle.

    Answer 5: This describes the supraspinatus muscle.

     

     

     

    OrthoCash 2020

     

  30. A 23-year-old-male was involved in a motorcycle accident. Imaging is shown in Figure A. In order to prevent the most common deformity associated with intramedullary nailing of this injury, where should blocking screws be placed and what deformity are they trying to prevent?

     

     

     

    1. Lateral and posterior to the nail in the proximal segment; procurvatum and valgus

    2. Medial and posterior to the nail in the proximal segment; procurvatum and varus

    3. Lateral and posterior to the nail in the proximal segment; recurvatum and varus

    4. Medial and anterior to the nail in the proximal segment; recurvatum and valgus

    5. Anterior and posterior to the nail in the proximal segment; recurvatum Corrent answer: 1

    When using intramedullary nails (IMNs) for treatment of proximal tibia fractures, blocking screws can be used to prevent the typical deformity of valgus and apex anterior. Traditionally, these blocking screws are placed lateral and posterior to the nail in the proximal segment.

     

    Proximal tibia fractures have a tendency to displace into valgus and procurvatum due to deforming forces from the pes anserinus and anterior compartment (valgus) and patellar ligament (procurvatum). It is difficult to correct or prevent this mal-alignment with IMN insertion alone due to the capacious nature of the proximal tibial metaphysis compared to the canal fit of

    the IMN in the diaphyseal segment. Blocking screws placed lateral and posterior to the nail have been found to prevent this translation by effectively decreasing canal space for the intramedullary device. In the proximal tibia, a good "rule of thumb" is to place the blocking screws on the concavity of the deformity in the metaphyseal segment; however blocking screws are not only used this way.

     

    Krettek et al. performed a study to evaluate the mechanical effects of medial and lateral blocking screws in supplementing intramedullary nail fixation of high proximal and low distal tibia fractures treated with small-diameter intramedullary nails. They found that the addition of a medial blocking screw in the proximal tibia increased the deformation of the construct. They conclude that medial blocking screws can increase the primary stability of proximal metaphyseal fractures after nailing. This study exemplifies the use of blocking screws not only to help achieve alignment when treating a fracture but also maintain the alignment during fracture healing when the IMN is not canal-fitting around a fracture site.

     

    Another study by Krettek et al. evaluated the clinical use of blocking screws as a supplement to stability after fixation with statically locked intramedullary nails in metaphyseal fractures. They prospectively studied 21 tibial fractures, 10 in the proximal third and 11 in the distal third. They found that losses of reduction were minimal when using blocking screws. They also had no complications related to the use of blocking screws. They conclude that blocking screws are a useful adjunct when nailing metaphyseal fractures of the tibia.

     

    Figures A and B are AP and lateral radiographs of the tibia and fibula, respectively, demonstrating proximal tibia and fibula fractures. Illustrations A and B are AP and lateral images of the same injury, respectively, treated with an intramedullary nail. Note the use of blocking screws lateral and posterior to the nail.

     

    Incorrect Answers:

    Answers 2, 3, 4, & 5: Blocking screws are typically used to "block" the IMN from creating or fixing a mal-reduction. In this case, they should be placed lateral and posterior to the nail to prevent a valgus and apex anterior deformity.

     

     

     

     

     

    OrthoCash 2020

     

  31. Which of the following distal radius fractures is associated with volar translation of carpus relative to the radial articulation?

    1. Displaced impaction fracture of the lunate fossa

    2. Displaced intra-articular fracture with a fragment consisting of the volar-ulnar corner

    3. Displaced radial styloid fracture

    4. Displaced extra-articular fracture with apex volar

    5. Displaced extra-articular fracture with apex dorsal Corrent answer: 2

    A displaced intra-articular fracture with a fragment containing the critical volar-ulnar corner rim of the distal radius would result in volar translation of the carpus.

     

    Distal radius fractures are one of the most common orthopedic injuries and can result from low-energy trauma in older and osteoporotic patients or high-energy trauma in young patients. Intra-articular involvement poses treatment

    challenges for these fractures as the fragments are crucial to articular stability and are difficult to achieve fixation with traditional distal radius plates.

    Fractures with intra-articular comminution of the distal radius that involve either the dorsal or volar rim of the lunate fossa, which can destabilize the radiocarpal joint, can lead to volar or ulnar translation of the carpus.

     

    Orbay et al. performed a retrospective review of patients undergoing either hook plate fixation or volar opening wedge osteotomy for volar marginal fragment distal radius fractures. For the patients treated with hook extension plates, there was a 90% success rate in the prevention of volar subluxation of the carpus. For patients that required volar open wedge osteotomy to redistribute joint loading forces in those that developed avascular necrosis of the volar marginal fragments, all patients had improved pain, function, and radiographic concentric reduction of the radiocarpal joint. They concluded that hook plate fixation provides an effective means of fixation of a volar marginal fracture of the distal radius with volar opening wedge osteotomy as an effective salvage procedure.

     

    O'Shaughnessy et al. performed a retrospective study of hook plate fixation of distal radius fractures with volar marginal rim fragments. There were no patients in the study that lost fixation of the volar marginal fragments after treatment with hook plates with the most common complication being symptomatic hardware requiring removal of both the traditional plate and the hook plate. The authors concluded that hook plate fixation provides a highly effective means of distal radial fragment fixation that is not amenable to traditional plate fixation.

     

    Illustration A depicts a lateral radiograph of the right wrist with a fracture of the volar-ulnar corner resulting in volar translation of the carpus. Illustration B depicts a lateral radiograph of the wrist with a ulnar hook plate buttressing an articular surface rim fracture.

     

    Incorrect answers:

    Answer 1: Impaction fractures of the lunate fossa do not result in translational instability of the carpus.

    Answer 3: A radial styloid fracture can result in ulnar translation of the carpus if the fragment is large enough to involve the large radiocarpal stabilizing ligaments.

    Answer 4: An extra-articular fracture of the distal radius with apex volar angulation may have dorsal comminution, but there is no added risk of volar carpal translation.

    Answer 5: An extra-articular distal radius fracture with apex dorsal angulation

    leads to volar tilting of the articular surface, but there is no volar translation of the carpus.

     

     

     

     

     

     

     

     

     

    OrthoCash 2020

     

  32. Which of the following acetabular fractures is classified as an elementary fracture pattern that involves two columns?

     

     

     

     

     

     

     

     

     

     

     

     

     

     

    1. Figure A

    2. Figure B

    3. Figure C

    4. Figure D

    5. Figure E

     

    Corrent answer: 2

     

    Figure B is a radiograph of a transverse acetabular fracture, which is the only elementary fracture pattern that involves two columns.

     

    The acetabulum consists of two columns arranged in an inverted Y-shaped configuration. Fractures are divided into five elementary patterns and five associated patterns. Elementary fractures involve only one fracture plane or surface while associated fractures involve multiple fracture planes or surfaces. The transverse fracture is the only elementary pattern that involves two

    columns. Six fundamental landmarks of the acetabulum must be evaluated on each AP pelvic radiograph: the lateral border of the anterior wall; the lateral border of the posterior wall; the acetabular roof, representing the weightbearing dome; the teardrop; the ilioischial line, representing the posterior column; and the iliopectineal line, representing the anterior column or pelvic brim.

     

    Mauffrey et al. performed a review of the radiographic evaluation of acetabular fractures. They report that plain radiographs remain the initial step in the classification of acetabular fractures. The ability to accurately interpret the injury allows the surgeon to develop a thorough preoperative plan and to evaluate the quality of the reduction and fixation intraoperatively. Though CT and three-dimensional imaging is obtained in addition to radiographs, radiographs still have a vital role to play in the diagnosis and treatment of acetabular fractures.

     

    Durkee et al. performed a study to characterize and classify the radiographic and CT appearance of common acetabular fractures. They report that for a transverse fracture, there is no involvement of the obturator ring. It causes disruption of both the iliopectineal and ilioischial lines, however. On CT, this fracture line is sagittally oriented. It is transverse relative to the plane of the acetabulum.

     

    Figures A-E are AP radiographs of the pelvis demonstrating an associated both column (ABC) acetabular fracture, a transverse acetabular fracture, a posterior wall acetabular fracture, a T-type acetabular fracture, and a normal pelvis, respectively. Illustration A is an image demonstrating normal acetabular lines and morphology on the right and a transverse acetabular fracture on the left.

    Illustration B is an algorithm to determine acetabular fracture patterns (Mauffrey et al.).

     

    Incorrect Answers:

    Answers 1, 3, 4, & 5: The transverse acetabular fracture is the only elementary pattern to involve two columns.

     

     

     

     

     

     

     

     

    OrthoCash 2020

     

  33. A 90-year-old patient presents after a fall down a flight of stairs and sustains a hip fracture, several rib fractures, a right pneumothorax, a right grade 1 open humerus fracture, and a subdural hematoma. The patient requires intubation in the trauma bay. The trauma team assesses the Injury Severity Score to 36. Which factor is associated with the highest risk of mortality at 1-year?

    1. Presence of a head injury

    2. The need for mechanical ventilation

    3. Injury severity score greater than 25

    4. Presence of a hip fracture

    5. Presence of an open fracture Corrent answer: 2

    The patient is a nonagenarian presenting with multiple injuries after trauma. The need for mechanical ventilation is associated with the highest risk of mortality at 1-year with 78.7% risk.

     

    Geriatric patients pose different treatment challenges in the trauma setting compared to patients under the age of 65. The treatment course is complicated by many medical comorbidities, polypharmacy, lower physiologic reserve, and increased risk of medical complications. Several factors have been described as predictors of mortality in admission including the presence of a head injury, the need for mechanical ventilation, higher Injury Severity Score (ISS), and treatment at a facility other than a trauma center. Of these factors, the need for mechanical ventilation has the highest association with mortality, especially at 1-year.

     

    Hwabejire et al. performed a retrospective study of mortality predictors in trauma patients greater than 90 years of age. The in-hospital mortality was significantly associated with Injury Severity Score, cervical spine injury, and the need for mechanical ventilation with an overall in-hospital mortality rate being 9.5%. At 1-year, mortality rates were increased to 78.7% if the patient required mechanical ventilation at the time of admission. The authors concluded that despite the low in-hospital mortality these risk factors are strong predictors to 1-year from the injury.

     

    Mock et al. performed a retrospective cohort study investigating the predictors of mortality in geriatric trauma patients. The found through multivariate analysis that the need for mechanical ventilation, admission lactate, and the length of mechanical ventilation were significant predictors of mortality. They stipulated that the need for mechanical ventilation is a predictor of mortality, but this finding is confounded with the withdrawal of care.

     

    Meldon et al. performed a retrospective cohort study looking at outcomes and care settings of trauma patients over the age of 80. The authors reported a significantly higher mortality rate with treatment performed at acute care hospitals compared to level 1 and level 2 trauma centers. Additionally, mortality was associated with increased ISS and decreased Glasgow Coma Score. They concluded that treatment at a trauma verified center as a survival benefit for geriatric trauma patients.

     

    Incorrect answers:

    Answer 1: The presence of a head injury at the time of admission does increase the mortality risk at 1-year, with one study citing 51.1%.

    Answer 3: An injury severity score greater than 25 has been associated with increased mortality at 1-year, but several studies have linked the need for

    mechanical ventilation to greater mortality risk. One study reported 73.2% mortality at 1-year in patients with ISS greater than 25.

    Answer 4: The effect of a hip fracture in geriatric trauma patient mortality has not been characterized, but 1-year mortality risk is approximately 30%.

    Answer 5: The mortality risk of an open fracture in a geriatric trauma patient has been seldom studied, but some authors report risks similar to geriatric hip fractures.

     

     

     

    OrthoCash 2020

     

  34. What is the spur sign and on which view is it best seen?

     

    1. Intact ilium; obturator oblique

    2. Intact ilium; iliac oblique

    3. Posterior wall fragment; iliac oblique

    4. Posterior wall fragment; obturator oblique

    5. Posterior wall fragment; oburator inlet Corrent answer: 1

    The spur sign represents the intact posteroinferior aspect of the ilium that is seen in an associated both column (ABC) acetabular fracture. It is best seen on the obturator oblique view.

     

    The spur sign is the portion of nonarticular ilium above the acetabulum that becomes visible on the obturator oblique view with medial displacement of the acetabulum. Nondisplaced associated both-column fractures will not have a spur sign on the obturator oblique view. Computed tomography should be used to verify the continuity between the axial skeleton and the acetabular articular surface.

     

    Mauffrey et al. performed a radiographic evaluation of acetabular fractures. They report that, when evaluating acetabular fractures, CT scans are useful because they allow the clinician to trace an intact strut of bone from the sacroiliac joint to the weight-bearing dome of the acetabulum. If a piece of bone is not present in continuity between the sacroiliac joint and the acetabulum, the fracture must be an associated both-column fracture. If a spur sign is not present and a continuous bony bridge between the sacroiliac joint and the acetabulum is present, the fracture is classified as an anterior column posterior hemitransverse fracture.

     

    Kellam and Messer performed a study to assess acetabular joint congruity and postoperative reconstructions. They evaluated the use of coronal and sagittal

    reconstruction of transaxial CT scans. They found in all cases of marginal impaction coronal and sagittal CT reconstruction showed that the fragments were larger than they appeared to be on the transaxial CT scan. They conclude that coronal and sagittal CT reconstruction provided more detailed evaluations of complex fractures than other studies.

     

    Illustration A is an electronically simulated image demonstrating an associated both column acetabular fracture on the left side. Illustration B is an obturator oblique of the same image with the spur sign indicated by the arrow.

     

    Incorrect Answers:

    Answers 2, 3, 4, & 5: The spur sign is a portion of the intact ilium that is best seen on the obturator oblique view. It is indicative of an associated both column fracture.

     

     

     

     

     

     

     

     

    OrthoCash 2020

     

  35. An errant screw is placed during the procedure shown in Figure

    A. What is the most likely physical exam manifestation?

     

     

     

    1. Weakness in knee extension

    2. Weakness in ankle plantarflexion

    3. Weakness in great toe extension

    4. Numbness over the medial aspect of the leg

    5. Numbness over the posterior leg Corrent answer: 3

      The most likely location for an errantly placed screw is anterior. This would lead to injury in the L5 nerve root, which would manifest with weakness in great toe extension.

       

      Indications for iliosacral screw fixation include complete sacral fractures (i.e. fractures that extend through the sacrum in its entirety, anterior to posterior), sacroiliac joint disruptions, and combinations of these posterior pelvic injuries following reduction. The safe zone for screws is the area within the sacrum where the screws remain within the bone. The L5 and S1 nerve roots course from the spinal canal in an anterior-lateral-caudal and oblique direction. The L5 nerve root is located on the anterior-cranial sacral alar surface and may be injured by an errant screw that is placed too anteriorly. The S1 nerve root is located within the oblique neural tunnel, beginning at the spinal canal and exiting through the anterior sacral foramina between the upper and second sacral segments.

       

      Kaiser et al. performed a study to evaluate anatomic determinants of sacral dysmorphism and implications for safe screw placement. They found the prevalence of upper sacral segment dysplasia at 41%. The major determinants of sacral dysmorphism are upper sacral segment coronal and axial angulations. They developed a sacral dysmorphism score, derived with the equation: (first sacral coronal angle) + 2(first sacral axial angle). They conclude that the sacral dysmorphism score quantifies dysmorphism and can be used in preoperative planning of IS screw placement.

       

      Miller and Routt performed a review of variations in sacral morphology and

      implications for iliosacral screw fixation. They report that knowledge of the posterior pelvic anatomy, its variations, and related imaging is critical to performing reproducibly safe surgery. They report that the dysmorphic pelvis has several key characteristics. The upper portion is colinear with the iliac crests on the outlet view. Mamillary bodies (underdeveloped transverse processes) may be present as well. A residual upper sacral disk may be present along with an acute alar slope. They conclude that the surgeon must be knowledgeable about individual patient anatomy to ensure safe iliosacral screw placement.

       

      Figure A is an illustration of a sacroiliac-style iliosacral screw. Illustration A is an inlet fluoroscopic image demonstrating the drill within the sacrum with an arrow indicating the location of the L5 nerve root. Illustration B is an axial CT of a patient with normal sacral anatomy through the first sacral and second sacral segments, respectively. Illustration C is an axial CT of a patient with dysmorphic sacral anatomy through the first sacral and second sacral segments, respectively. The L5 nerve roots are indicated by the white circles.

       

      Incorrect Answers:

      Answer 1: Weakness in knee extension would indicate injury to the L4 nerve root.

      Answer 2: Weakness in ankle plantarflexion would indicate injury to the S1 nerve root.

      Answer 4: Numbness to the medial leg would indicate injury to the L4 nerve root.

      Answer 5: Numbness over the posterior leg would indicate injury to the S1 nerve root.

       

       

       

       

       

       

       

       

       

       

       

      OrthoCash 2020

       

  36. During which surgical approach to the acetabulum is the sciatic nerve placed at risk, and how should the patient be positioned to minimize the risk of injury?

    1. Kocher-Langenbeck; hip extended and knee extended

    2. Kocher-Langenbeck; hip extended and knee flexed

    3. Kocher-Langenbeck; hip flexed and knee extended

    4. Ilioinguinal; hip extended and knee extended

    5. Ilioinguinal; hip extended and knee flexed Corrent answer: 2

    The sciatic nerve is placed at risk when performing the Kocher-Langenbeck approach to the acetabulum. Positioning the patient with the hip extended and the knee flexed minimizes the risk of injury to the sciatic nerve.

     

    During the surgical fixation of acetabular fractures through a Kocher-Langenbeck approach, the sciatic nerve is placed at risk. It can be injured during the internal fixation of acetabular fractures, as well as during plate fixation and retractor placement. The sciatic nerve is maintained in the least

    tension by keeping the hip extended and the knee flexed.

     

    Isaak et al. published a review on sciatic nerve injuries associated with acetabular fractures. They reported that sciatic nerve injuries may occur as a result of the initial acetabular trauma (posttraumatic), as a result of iatrogenic trauma during surgery (perioperative), or as a later complication of surgery (postoperative). Iatrogenic injuries, in particular, can result from: (1) a stretch injury due to excessive retraction of the posterior gluteal muscle mass with the hip in flexion or continuous extension of the ipsilateral knee, (2) placement of retractors or reduction clamps in the lesser or greater sciatic notch, or (3) drilling or placing screws into the greater sciatic notch.

     

    Fassler et al. performed a study on functional outcomes of sciatic nerve injuries after a displaced acetabular fracture. They found that all but one patient had a satisfactory (fair or better) functional outcome, but 11 patients had residual neurological sequelae that ranged from minor paresthesia to footdrop. They also found that the patients who had isolated mild involvement of the peroneal nerve had a favorable prognosis, but those who had a severe injury of the peroneal component, whether it was isolated or associated with an injury of the tibial component, did not recover good function.

     

    Incorrect Answers:

    Answers 1 and 3: The sciatic nerve is maintained in the least tension when the hip is extended and the knee is flexed.

    Answers 4 and 5: The femoral and lateral cutaneous femoral nerves, not the sciatic nerve, are at risk of injury when performing the ilioinguinal approach to the acetabulum.

     

     

     

    OrthoCash 2020

     

  37. A 59-year-old patient presents with right elbow pain after a fall off a ladder. There is swelling of the elbow with a visible deformity. The patient is neurovascularly intact with decreased sensation in the ring and small fingers. The patient states that this has been present for several years. Figures A and B are radiographs of the right elbow. What is the most appropriate treatment that provides the greatest construct rigidity and appropriately addresses the patient's ulnar nerve symptoms?

     

     

     

     

     

    1. Bicolumnar orthogonal plating with submuscular transposition

    2. Bicolumnar parallel plating with all distal screws passing through a plate and in situ cubital tunnel release

    3. Bicolumnar parallel plating with all distal screws passing through a plate and observation of ulnar neuropathy

    4. Bicolumnar orthogonal plating with in situ cubital tunnel release

    5. Interfragmentary screw fixation of the articular surface with neutralization plating and subcutaneous transposition of the ulnar nerve

    Corrent answer: 2

     

    The patient is presenting with a H-type distal humerus fracture with previous cubital tunnel symptoms. The biomechanically strongest treatment option would include bicolumnar parallel plating following the fixation objectives of O'Driscoll and a cubital tunnel release to address the ulnar nerve symptoms.

     

    Distal humerus fractures are potentially complex fractures that require special fixation considerations when performing open reduction and internal fixation. The fixation objectives of O'Driscoll state that every screw should pass through a plate, each screw should be as long as possible, engaging as many articular fragments as possible, screws from each plate should interdigitate, and that compression is achieved in both columns through the supracondylar region and shaft. Studies have demonstrated that parallel plating provides the greatest construct rigidity than orthogonal plating. Bicolumnar parallel plating has been found to have significant greater biomechanical strength and rigidity compared to orthogonal plating, but clinical differences between the two plate types are minimal to absent. In the setting of chronic cubital tunnel syndrome, an in situ ulnar nerve decompression should be performed at the time of open reduction and internal fixation.

     

    Cobb et al. performed a retrospective study of elderly patients, with a mean age of 72 years, undergoing a primary total elbow arthroplasty for distal humerus fractures. The authors reported there to be excellent outcomes in 75% of patients at 3.3 years according to the Mayo elbow performance score with the mean range of motion being 25-130 degrees and only one implant requiring revision. The authors concluded that total elbow replacement is an effective option for severely comminuted distal humerus fractures in elderly patients, but is not recommended in younger patients.

     

    Taylor et al. performed a biomechanical cadaveric study testing for construct rigidity and strength between parallel and orthogonal distal humeral plating. They found the parallel plating had greater external and internal torsional rigidity, flexion and extension bending rigidity, and bend to failure strength. They concluded that parallel plating was a significantly stronger construct that

    may be desirable in the treatment of comminuted distal humerus fractures.

     

    Shearin et al. performed a meta-analysis comparing ulnar nerve decompression to ulnar nerve transposition during surgical treatment of distal humerus fractures. The pooled data in their analysis revealed an overall 19.3% incidence of ulnar nerve neuropathy with a 15.3% incidence with an in situ release and 23.5% incidence with ulnar nerve transposition. They concluded that transposition of the ulnar nerve did not have a protective effect against post-operative neuropathy, but there could have been a confounding effect of increased fracture severity observed in this group.

     

    In a current concepts review in the Journal of Hand Surgery, Fajolu et al. propose that a transposition should only be performed at the conclusion of the case if the surgeon feels that the previously released nerve is in contact with the hardware or if the surgeon feels that there was excessive traction on the nerve during the case.

     

    Figures A and B depict an AP and lateral radiograph of the elbow with a severely comminuted bicolumnar distal humerus fracture. Illustrations A and B depict the postoperative radiographs with bicolumnar plating following the fixation objectives of O'Driscoll.

     

    Incorrect answers:

    Answer 1: Parallel plating provides the greater biomechanical construct rigidity than orthogonal plating. Submuscular transposition is indicated if there is subluxation of the ulnar nerve or recurrence of cubital tunnel release.

    Answer 3: Observation of chronic ulnar neuropathy is not recommended since these symptoms are likely to worsen after open reduction and internal fixation if not addressed.

    Answer 4: Parallel plating provides the greater biomechanical construct rigidity than orthogonal plating.

    Answer 5: Use of interfragmentary lag screws that are not passing through a plate is not part of the fixation objectives of O'Driscoll.

     

     

     

     

     

    OrthoCash 2020

     

  38. A 35-year-old female fell while riding a motorcycle and sustained the left elbow injury shown in Figures A and B. Figures C and D are the CT scan and 3D reconstruction of the injury. After closed reduction, the elbow is unstable with valgus stress at 40 degrees of flexion. What is the most appropriate definitive treatment?

     

     

     

     

     

     

     

     

     

     

     

    1. Radial head ORIF and MCL repair

    2. Radial head replacement and LUCL repair

    3. Radial head excision, coronoid ORIF, and LUCL repair

    4. Radial head replacement, coronoid ORIF, and LUCL repair

    5. Radial head excision and LUCL repair Corrent answer: 4

    The patient has a terrible triad injury of the elbow, which will require a radial head replacement, coronoid ORIF, and lateral ulnar collateral ligament (LUCL) repair.

     

    Terrible triad injuries are the result of a radial head fracture, a coronoid tip fracture, and a posterior elbow dislocation. The structures damaged with this injury pattern begins with the disruption of the LUCL, then the anterior capsule, and possibly the medial collateral ligament. Treatment involves ORIF of the radial head if <40% involvement of the articular surface and without comminution. Radial head arthroplasty should be performed for highly comminuted fractures or with >40% articular involvement. Repair of the LUCL to the humeral insertion is required in most cases. Coronoid fractures result from avulsion of the anterior capsule and usually involve small bone fragments.

     

    Tejwani et al. reviewed the management of fractures of the radial head and neck. The authors recommended performing an open reduction and internal fixation when fractures of the radial head involve three or fewer pieces. Radial head excision is not recommended when there is associated elbow dislocation or ligamentous instability.

     

    Acevedo et al. reviewed the current literature on implant designs for radial head arthroplasty (RHA). Loose-fitting prostheses settle to a level of anatomic stability over time and have low complication rates. Press-fit stems obtain intramedullary fixation through hoop stresses. However, implants with plasma-sprayed titanium stem have been associated with increased revision rates because of subsequent loosening. Bipolar prostheses consist of a press-fit stem that articulates with a polyethylene head component at the head and neck junction and is associated with increased dislocation rates, especially in the setting of a terrible triad injury. Understanding the prosthesis design of radial head implants is necessary to ensure good outcomes following terrible triad injuries.

     

    Wyrick et al. reviewed the management of complex elbow dislocations. The literature suggests terrible triad injuries should be urgently closed reduced to avoid potential neurovascular injuries and splinted in 90 degrees flexion to confer a stable reduction until operative management. The recommended sequence of repair begins with lasso fixation of the coronoid, followed by fixation or replacement of the radial head, and then repair of the LUCL.

     

    Figures A and B are AP and lateral radiographs of the left elbow with a terrible triad injury and comminution of the radial head. Figure C is the sagittal CT scan of the ulnohumeral joint demonstrating a coronoid fracture. Figure D is

    the 3D CT reconstruction of the elbow with comminution of the radial head and a type 2 fracture of the coronoid.

     

    Incorrect answers:

    Answer 1: The patient has a comminuted fracture of the radial head that would require an RHA rather than an ORIF. MCL is less likely than a LUCL injury.

    Answer 2: The patient sustained a coronoid fracture with greater than 10% involvement, which should be addressed with ORIF.

    Answers 3 and 5: Radial head excision should never be performed in the setting of elbow instability.

     

     

     

    OrthoCash 2020

     

  39. A 35-year-old male sustains the injury shown in Figure A. He is currently not cleared for surgery due to a severe head injury. At what time point after the injury is there an increased risk of a poor outcome?

     

     

     

     

    1. 48 hours

    2. 5 days

    3. 1 week

    4. 2 weeks

    5. 3 weeks

     

    Corrent answer: 5

     

    This patient has sustained an anterior column acetabular fracture. Delay of fixation for more than 3 weeks is associated with difficulty in achieving adequate reduction and poor outcomes.

    Historically earlier treatment of acetabular fractures has been shown to improve clinical and functional outcomes. Delays of more than 3 weeks have been associated with difficulty obtaining an anatomic reduction and decreased overall patient outcomes. An increased delay has not been shown to increase the chances of neurological injury, infection, or heterotopic ossification.

     

    Plaisier et al. performed a study to evaluate the effect of the timing of fixation for acetabular and pelvic ring fractures on patient outcome. They found that early fixation of acetabular fractures was associated with a decreased total length of stay. Functional outcome was improved in early fixation of acetabular fractures with a greater proportion of patients being discharged home rather than to rehabilitation or skilled care. They conclude that patients with early repair of acetabular fractures had significantly less organ dysfunction and exhibited improved functional outcome.

     

    Madhu et al. performed a retrospective case review to determine if the time to surgery was predictive of radiological and functional outcomes of acetabular fractures. For elementary fractures, an increase in the time to surgery of one day reduced the odds of an excellent/good functional result by 15% and of an anatomical reduction by 18%. For associated fractures, the odds of obtaining an excellent/good result were reduced by 19% and an anatomical reduction by 18% per day. They conclude that the time to surgery is a significant predictor of radiological and functional outcome for both elementary and associated displaced fractures of the acetabulum.

    Figure A is an AP pelvis demonstrating a displaced fracture of anterior column. Incorrect Answers:

    Answers 1, 2, 3, & 4: Delay of fixation for more than 3 weeks is associated

    with difficulty in achieving adequate reduction and poor outcomes.

     

     

     

     

    OrthoCash 2020

     

  40. A 43-year-old male presents to the trauma bay with the injury sustained in Figure A after a motor vehicle collision. After appropriate resuscitation, you have decided to proceed with open reduction and internal fixation via a Kocher approach and percutaneous fixation of the anterior column. The inlet view can be used to help visualize which of the following during guidewire placement?

     

     

     

    1. Anterior-posterior position of the wire within the ramus

    2. Ensure appropriate reduction of the fracture

    3. Cranial-caudal position of the wire

    4. Rule out joint penetration of the wire

    5. Ensure the wire is within the tables of the ilium Corrent answer: 1

    An inlet view of the anterior pelvis can be used to help visualize the anterior-posterior position of the guide-wire in reference to the ramus during the placement of an anterior column screw.

     

    Anterior column fixation can be performed percutaneously with a medullary screw as a less-invasive alternative to ORIF of the anterior column. Medullary implants can be inserted safely through the anterior column osseous fixation pathway from anterograde or retrograde start points (Illustrations A and B). When placed anterograde, the start point is on the gluteal pilar and the implant is then inserted and directed towards the pubic tubercle. The combined obturator oblique outlet (COOO) view helps ensure the screw does not penetrate the hip joint nor the vascular lacuna. An inlet view demonstrates the anterior-posterior limits of the osseous fixation pathway in the ramus portion of the corridor. Some authors have described a modified inlet with iliac-oblique rollback to it, however the tangential lines of the ramus are usually best seen on a pure inlet view.

    Starr et al. wrote a technique on percutaneous fixation of the columns of the acetabulum. They report to safely place an anterior column screw, the C-arm should be oriented perpendicular to the superior pubic ramus. The C-arm is rotated during screw placement between an iliac inlet view and an obturator outlet view. The iliac inlet view is used to ensure that the guidewire does not penetrate the inner cortex of the superior ramus. The obturator outlet view is used to make sure the wire does not penetrate the hip joint.

     

    Mauffrey et al. performed a review of the radiographic evaluation of acetabular fractures. They report that the acetabulum consists of two columns of bone arranged in an inverted Y-shaped configuration and attached to the ilium. They report that radiographic evaluation and classification of acetabular fractures is necessary to guide treatment decisions. They conclude that CT scans provide invaluable information in classifying these injuries.

     

    Figure A is the AP pelvis radiograph revealing a right transverse acetabulum fracture. Illustration A demonstrates the path for retrograde placement of an anterior column screw. Illustration B demonstrates the path for anterograde placement of an anterior column screw. Illustration C and D are intra-operative fluoroscopy views revealing an inlet view and obturator outlet views, respectively.

     

    Incorrect Answers:

    Answer 2: The best view ensuring appropriate reduction is direct visualization of the fracture (open reduction).

    Answer 3: The obturator outlet view is used to determine the cranial-caudal position of the wire during the placement of an anterior column screw.

    Answer 4: The obturator outlet view is used to help rule out joint penetration during the placement of an anterior column screw.

    Answer 5: The obturator inlet view is used to help determine if a supra-acetabular wire is within the tables of the ilium.

     

     

     

     

     

     

     

     

     

     

     

     

     

    OrthoCash 2020

  41. An 87-year-old female sustained the injury shown in Figure A. Which of the following is/are factor(s) that increase the risk of perforation of the anterior cortex during surgical treatment with a long cephalomedullary nail?

     

     

     

     

    1. Radius of curvature mismatch between the bone and the implant

    2. Anterior starting point on the greater trochanter

    3. Posterior starting point on the greater trochanter

    4. Answers 1 and 2

    5. Answers 1 and 3

     

    Corrent answer: 5

     

    This patient sustained an intertrochanteric femur fracture. A mismatch of curvature between the nail and a posterior start point are both risk factors for anterior cortex perforation during surgical treatment with a long cephalomedullary nail.

     

    When treating elderly patients with long intramedullary nails, there are certain factors that need to be accounted for. There is a mismatch between the radius of the femoral nails currently available and the femoral canal, ie, nails with a larger radius of curvature (straighter) are being inserted in a femur with a

    smaller radius of curvature (less straight than the nail). As a result, the distal tip of the nail is forced into a nonanatomic anterior position during final seating of the nail distally. To avoid this complication, a lateral view of the distal femur while inserting the nail is recommended. Additionally, a posterior starting point on the greater trochanter is also a risk factor for anterior perforation of the nail.

     

    Ostrum et al. performed a case report of 3 cases with penetration of the distal femoral anterior cortex during intramedullary nailing for subtrochanteric femur fractures. They report that the difference in the femoral anteroposterior bow between the bone and the implant is a contributing factor to distal femoral anterior cortex penetration. The radius of curvature of the femur is less than the radius of curvature of the nails.

     

    Roberts et al. performed a retrospective review to determine risk factors for impingement and penetration of the anterior cortex of the distal femur during intramedullary nailing of proximal femur fractures. The found that shorter patients and patients with an increased femoral bow were likely to have an anterior nail tip position or cortical impingement. They conclude that anterior starting points should be used to avoid this complication.

     

    Figure A is an AP radiograph demonstrating a left intertrochanteric femur fracture.

     

    Incorrect Answers:

    Answers 1, 2, 3, 4: Risk factors for perforation of the anterior cortex while placing a long nail in an elderly patient are a mismatch in the radius of curvature and a posterior start point.

     

     

     

    OrthoCash 2020

     

  42. A 65-year-old man sustained the closed injury seen in Figures A and B and is being treated nonoperatively in a functional brace. At what time point after the injury does the lack of callus formation and motion at the fracture site first become concerning for nonunion?

     

     

     

     

     

    1. 2 weeks

    2. 4 weeks

    3. 6 weeks

    4. 12 weeks

    5. 6 months

     

    Corrent answer: 3

     

    In a closed humeral shaft fracture, gross motion at the fracture site and no callus formation on radiographs at 6 weeks has a specificity of 100% and positive predictive value of 100% of going on to nonunion.

     

    Closed midshaft humerus fractures treated with nonoperative measures such as functional bracing have been shown to go on to union in most patients.

    Treatment with functional bracing relies on hydrostatic compressive forces from the surrounding soft tissues to provide adequate stability for healing. However, approximately 10% of patients go on to nonunion and require subsequent surgical stabilization. Predictors of nonunion include no callous at the fracture site and gross motion at the fracture site at 6 weeks from injury.

     

    Papasoulis et al. reviewed the clinical studies of humeral shaft fractures treated with functional bracing. They found that this set of studies showed healing at

    10.7 weeks from injury at a union rate of 94.5%. They concluded that functional bracing has many known benefits and remains a reliable treatment however certain parameters such as functional outcome, residual deformity, and loss of joint motion remain unclear and require further research.

     

    Driesman et al. followed 84 consecutive patients with diaphyseal humeral shaft fractures treated nonoperatively. They found that mobile fracture sites at 6 weeks from injury predicted nonunion with 82% sensitivity and 99% specificity. They concluded that knowledge of fracture motion can help in determining the appropriate management in decision making in nonoperatively treated humeral shaft fractures.

     

    Figures A and B show AP and transthoracic lateral radiographs of a mid diaphyseal spiral humeral shaft fracture, respectively.

     

    Incorrect Answers:

    Answers 1 & 2: 2 weeks and 4 weeks would be too early to reliably determine the risk of nonunion.

    Answers 4 & 5: 12 weeks and 6 months have not been described as a timeframe for prediction of nonunion of humeral shaft fractures treated nonoperatively.

     

     

    OrthoCash 2020

     

  43. A patient presents with the injury shown in figures A and B. What has been associated with the technique depicted in figures C and D?

     

     

     

     

     

     

     

     

     

     

     

    1. Longer operative times

    2. Increased deep surgical infection rates

    3. Unacceptably high malunion/nonunion rates

    4. Slower early return to function

    5. Longer hospital stays Corrent answer: 3

    Treatment of Shatzker V and VI tibial plateau fractures with hybrid external fixation is associated with increased malunion and nonunion rates.

     

    Hybrid external fixation for treating tibial plateau fractures involves the use of an external fixator to achieve reduction through ligamentotaxis. Additional fracture reduction is achieved through limited open incisions with fixation augmented through percutaneous cannulated screws. Definitive treatment with this technique avoids soft tissue complications that have been associated with traditional open reduction and internal fixation with bicondylar plating.

    However, studies have reported high malunion and nonunion rates due to a lack of rigid fixation.

     

    Bertrand et al. performed a prospective cohort study of patients undergoing either open reduction and internal fixation versus hybrid external fixation for Schatzker V and VI tibial plateau fractures. Hybrid external fixation was associated with significantly shorter operative times but insignificantly increased complication rates. They concluded that there were limited statistically differences between these techniques, but further studies are required before advising hybrid external fixation for higher Schatzker tibial plateau fractures.

     

    Gross et al. performed a retrospective study of patients treated with hybrid external fixation for Shatzker V and VI tibial plateau fractures. The authors found there was an 80% union rate, a 70% satisfactory reduction rate, and a 52% rate of malunion. The development of osteoarthritis was associated with plateau widening, articular comminution, articular step-off, and incorrect mechanical alignment. The authors concluded that hybrid external fixation is an effective means for the treatment of tibial plateau fractures that minimizes tissue dissection, with decreased blood loss, and shorter operative times, but associated with a very high malunion rate.

     

    Hall et al. performed a multicenter randomized controlled trial comparing the treatment of Schatzker V and VI fractures with open reduction and internal fixation with hybrid external fixation. Patients with hybrid external fixation had less intraoperative blood loss, fewer unanticipated secondary procedures, slightly faster return to pre-injury activity at 6 months and 1 year, and shorter

    hospital stay. They concluded that both hybrid external fixation and open reduction and internal fixation provide effective means for fracture treatment, but hybrid external fixation avoids soft tissue complications with deleterious consequences.

     

    Figure A is an AP radiograph of the right knee with a Schatzker VI tibial plateau fracture. Figure B is an axial CT slice of the articular surface of the tibia with extensive comminution. Figures C and D are the AP and lateral radiographs of the knee with a hybrid external fixation construct for a tibial plateau fracture

     

    Incorrect answers:

    Answers 1, 2, 4, and 5: Hybrid external fixation for the treatment of bicondylar tibial plateau fractures is associated with higher malunion and nonunion rates, shorter operative times, increased superficial infection rates, faster earlier return to pre-injury activities, and shorter hospital stay.

     

     

     

    OrthoCash 2020

     

  44. An 89-year-old female sustained the injury shown in Figure A and underwent a hemiarthroplasty. Which of the following has been associated with increased rates of post-operative dislocation?

     

     

     

     

    1. Posterior approach

    2. Anterior approach

    3. Anterolateral approach

    4. Use of a bipolar implant

    5. Use of a monopolar implant

    Corrent answer: 1

     

    The incidence of dislocation after hemiarthroplasty is highest when using a posterior approach.

     

    Elderly femoral neck fractures are one of the most common fractures encountered by orthopaedists and will only become more common as the population continues to age. The displacement of the femoral head is associated with delayed union or nonunion, an increased risk of femoral head necrosis due to disrupted blood flow at the femoral neck, and failure of internal fixation devices. For this reason, displaced femoral neck fractures in older patients are often treated with hemiarthroplasty. Three approaches to hemiarthroplasty have been described: a lateral approach, a posterior approach, and an anterior approach. The posterior approach has been used more historically; however, its use has been called into question as it has been associated with increased dislocation rates.

     

    Parker performed a trial on all patients with intracapsular femoral neck fractures being treated with hemiarthroplasty. Patients were randomized to surgery using either a lateral or posterior approach. They found that there were no statistically significant differences observed for any of the outcome measures including mortality, degree of residual pain and regain of walking ability. They concluded that both surgical approaches appear to produce comparable functional outcomes.

     

    van der Sijp et al. performed a meta-analysis to compare the outcomes based on approaches for hemiarthroplasty in the treatment of proximal femur fractures. They found 21 studies and found that the posterior approach poses an increased risk of dislocation and reoperation compared to the lateral approach and anterior approaches. They conclude that there are no evident advantages of the posterior approach and its routine use for fracture-related hemiarthroplasty should be questioned.

     

    Figure A is an AP pelvis radiograph demonstrating a displaced right femoral neck fracture.

     

    Incorrect Answers:

    Answers 2, 3, 4, & 5: None of these factors have been associated with an increased risk of dislocation after hemiarthroplasty for a femoral neck fracture.

     

     

     

    OrthoCash 2020

  45. A 50-year-old male sustained a humeral shaft fracture treated operatively 6 months ago. He denies medical problems but smokes 10 cigarettes per day. His current radiograph is shown in Figure A. He continues to have pain in his arm that is affecting his quality of life. On physical examination, there is motion at the fracture site. C-reactive protein and erythrocyte sedimentation rate are within normal limits. Which is the most appropriate definitive treatment for this fracture?

     

     

     

     

    1. Exchange humeral nailing

    2. Augmentative plating

    3. Nail removal with open reduction compression plating

    4. Smoking cessation and medical optimization

    5. Nail removal with open reduction and compression plating with bone grafting

    Corrent answer: 5

     

    This patient has sustained an atrophic nonunion of a humeral shaft fracture treated with an intramedullary nail. The most appropriate definitive treatment is nail removal with open reduction and compression plating with bone grafting.

    Most diaphyseal humeral fractures can be managed non-operatively with functional bracing. Operative treatment is indicated under a number of circumstances including open fractures, associated neurovascular injury, proximal and distal articular extension of the fracture, and in patients with other multiple injuries. Surgical stabilization can be accomplished with different implants and techniques. The two most common are plate and screw fixation and intramedullary nailing. Plate fixation has the advantages of potential absolute stability and sparing the rotator cuff from an incision. Intramedullary nailing has to be inserted proximally with potential damage to the rotator cuff. It, however, can be inserted with small incisions. If a nonunion develops after intramedullary nailing, nail removal and compression plating is the preferred treatment choice.

     

    Heineman et al. performed a metanalysis on plate fixation or intramedullary nailing of humeral shaft fractures. They performed a literature search from 1967-2007 comparing nails and plates in patients with humeral shaft fractures that reported complications due to surgery. They found that the risk of a complication is lower when plating a fracture of the humeral shaft than when using an intramedullary nail.

     

    Gerwin et al. performed an anatomical study to define the course of the radial nerve in the posterior aspect of the arm, with particular reference to its relationship to operative exposures of the posterior aspect of the humeral diaphysis. They found that the radial nerve crosses the posterior aspect of the humerus from an average of 20.7 +/- 1.2 centimeters proximal to the medial epicondyle to 14.2 +/- 0.6 centimeters proximal to the lateral epicondyle. They found the approach to permit the most visualization was the triceps reflecting approach.

     

    Figure A is a lateral radiograph of an atrophic nonunion of a humeral shaft being stabilized with an intramedullary nail.

     

    Incorrect Answers:

    Answer 1: Exchange nailing is not a preferred treatment for humeral nonunions.

    Answer 2: Augmentative plating may be used for hypertrophic nonunions that require additional stability, but not for an atrophic nonunion.

    Answer 3: Bone grafting should be performed in this case to improve the biologic environment to induce fracture healing.

    Answer 4: Smoking cessation and medical optimization should be performed however they are not the preferred definitive treatment.

     

     

    OrthoCash 2020

     

  46. A 45-year-old man is struck while crossing a major highway and sustains the injury depicted in Figure A. Which of the following statements comparing the techniques in Figure B and C is most accurate?

     

     

     

     

     

     

     

     

     

    1. Technique depicted in Figure B is associated with an increased risk of septic arthritis

    2. Technique depicted in Figure B is associated with increased rate of anterior knee pain

    3. Technique depicted in Figure B is associated with improved postoperative fracture alignment

    4. Technique depicted in Figure C is associated with an increased risk of septic arthritis

    5. Technique depicted in Figure C is associated with improved postoperative fracture alignment

    Corrent answer: 5

     

    Compared to infrapatellar tibial nailing, suprapatellar tibial nailing is associated with improved postoperative fracture alignment.

     

    While antegrade tibial nailing results in postoperative anterior knee pain in approximately 20% of patients, there is no significant difference in the incidence of anterior knee pain when the conventional infrapatellar approach is compared to suprapatellar approaches. In open tibial shaft fractures, no difference has been observed in the incidence of knee sepsis with either approach. However, several studies have demonstrated that intramedullary nail fixation through a suprapatellar approach is associated with a more accurate entry position and a more accurate fracture reduction when compared with an infrapatellar technique, particularly in more proximal and distal shaft

    fractures, without evidence of a functional impact on the patellofemoral joint. Lastly, intraoperative radiography is generally less cumbersome with suprapatellar nailing.

     

    Marecek et al. performed a multicenter comparison study of suprapatellar and infrapatellar approaches and the risk of knee sepsis after treatment of open tibia fractures. They reported no differences in the rates of infection, deep infection, or reoperation between suprapatellar and infrapatellar nailing groups. They concluded that the risk of knee sepsis after suprapatellar nailing of open fractures is low.

     

    Avilucea et al. performed a retrospective cohort study comparing postoperative alignment after suprapatellar versus infrapatellar nailing for distal tibial shaft fractures. They reported a significantly increased rate of primary angular malalignment of greater than 5 degrees in the infrapatellar compared to the suprapatellar nailing cohort. They concluded that in the treatment of distal tibial fractures, suprapatellar nailing results in a significantly lower rate of malalignment compared with the infrapatellar nailing.

     

    Jones et al. performed a study comparing the radiologic outcome and patient-reported function after suprapatellar and infrapatellar intramedullary nailing. They reported no difference in anterior knee pain, however, found a more accurate fracture reduction, both in terms of angulation and translation in the coronal plane, with the use of the suprapatellar technique. They concluded that when compared with infrapatellar nailing, the suprapatellar technique was not associated with more anterior knee pain, yet more accurate nail insertion and fracture reduction.

     

    Figure A depicts a displaced distal third tibial shaft fracture. Figure B depicts the infrapatellar tibial nailing technique. Figure C depicts the suprapatellar tibial nailing technique.

     

    Incorrect Answers:

    Answer 1 & 4: No significant difference has been reported in the incidence of knee sepsis with either technique.

    Answer 2: No significant difference has been reported in the incidence of anterior knee pain with either technique.

    Answer 3: The suprapatellar technique, not the infrapatellar, is associated with more accurate alignment.

     

     

     

    OrthoCash 2020

  47. A 56-year-old woman sustains the closed injury depicted in Figures A-B. On examination, her wrist is mildly swollen and she is unable to actively oppose her thumb. She also complains of some paresthesias in her thumb and index finger. The patient undergoes closed reduction and splinting; however, her paresthesias worsen significantly in the next 12 hours. What is the likely mechanism of her paresthesias and what is the most appropriate treatment?

     

     

     

     

    1. Nerve compression; open reduction internal fixation with open carpal tunnel release

    2. Nerve laceration; open reduction internal fixation with primary nerve repair or grafting

    3. Decreased arterial inflow; fasciotomy with open reduction internal fixation

    4. Reflex sympathetic dystrophy; vitamin C

    5. Nerve compression; repeat closed reduction Corrent answer: 1

    This patient is presenting with signs of acute carpal tunnel syndrome (CTS) in the setting of a displaced distal radial fracture. The pathogenesis of acute CTS is nerve compression, requiring urgent open carpal release with open reduction internal fixation (ORIF).

    Acute CTS is a well-recognized phenomenon after distal radial fractures. Risk factors include ipsilateral upper extremity fractures, translation of the fracture fragments, and articular distal radius fractures (DRFs). Acute CTS can manifest with paresthesias in the median nerve distribution and opponens pollicis weakness. Acute CTS is an indication for urgent surgical decompression of the median nerve.

     

    Odumala et al. performed a study to evaluate the role of carpal tunnel decompression in the prevention of median nerve dysfunction after buttress plating of DRFs. They reported that prophylactic decompression of the carpal tunnel results in twice the relative odds of developing median nerve dysfunction, which routinely self-resolved. They concluded that prophylactic median nerve decompression does not alter the course of median nerve dysfunction and may actually increase postoperative morbidity.

     

    Medici et al. performed a case-control study to investigate whether carpal tunnel release (CTR) during fixation DRFs improves outcomes. They reported no statistically significant difference between the groups in VAS and Mayo Wrist Scores, however, an increased risk of subsequent CTR in the group who underwent ORIF with no CTR at the index procedure. They concluded that the release of the transverse carpal ligament during ORIF may reduce the incidence of postoperative median nerve dysfunction.

     

    Niver et al. reviewed CTS after DRFs. They reported that acute CTS noted at the time of DRF warrants urgent surgical release of the carpal tunnel and fracture fixation, and that delayed CTS presenting after a distal radius fracture has healed may be managed in the standard fashion for CTR. They concluded that there is no role for prophylactic CTR at the time of distal radius fixation in a patient who is asymptomatic.

     

    Figures A and B depict a displaced apex volar DRF and a mildly displaced ulnar styloid fracture.

     

    Incorrect Answers:

    Answer 2: Acute CTS after DRF most often commonly occurs after median nerve compression and contusion, not laceration.

    Answer 3: This describes compartment syndrome, which is less likely than acute CTS given the clinical exam described.

    Answer 4: Reflex sympathetic dystrophy usually occurs after the acute phase of the DRF.

    Answer 5: Acute CTS after DRF requires urgent open carpal tunnel release.

     

     

     

    OrthoCash 2020

     

  48. Figures A and B depict the closed injury radiograph of a 79-year-old right-hand-dominant woman who fell on her left wrist. According to meta-analysis and systematic reviews, which of the following statements is most accurate regarding her injury?

     

     

     

     

    1. Improved functional outcomes with open reduction internal fixation (ORIF) through FCR approach vs. closed treatment

    2. No difference in radiographic outcomes after ORIF vs. closed treatment

    3. No difference in functional outcomes after ORIF vs. closed treatment

    4. Improved functional outcomes with closed treatment vs. ORIF

    5. Improved functional outcomes with external fixation and K wire fixation vs. ORIF

    Corrent answer: 3

     

    This elderly patient has sustained a closed intra-articular and shortened distal radial fracture (DRF). Many studies have reported no difference in functional outcomes when patients aged 60 and over are treated in a closed manner versus operatively for unstable fractures.

     

    The treatment of DRFs in the elderly population is controversial. A variety of nonoperative and operative treatments are available, including closed reduction and splinting/casting, K wire stabilization, external fixation, and ORIF. While conservative management of DRFs in the elderly is common,

    recent systematic reviews and meta-analyses have demonstrated that despite worse radiographic outcomes after closed treatment of unstable fractures, functional outcomes were no different between patients treated closed versus surgically in patients over the age of 60 years.

     

    Ju et al. published a systematic review and meta-analysis comparing treatment outcomes between nonsurgical and surgical treatment of unstable DRFs in the elderly. They reported no significant differences in DASH score, VAS pain score, grip strength, wrist extension, pronation, supination, and ulnar deviation between the groups. They concluded that operative and nonoperative treatments result in similar outcomes in the treatment of unstable DRFs in the elderly, with no impact on subjective function outcome and quality of life with closed treatment.

     

    Diaz-Garcia et al. published a systematic review of the outcomes and complications after treating unstable DRFs in the elderly, comparing various treatment techniques. They reported significant differences in wrist motion, grip strength, DASH score, although these findings may not be clinically meaningful. They concluded that although the operatively treated group had improved radiographic outcomes, functional outcomes were no different when compared to the group treated in a closed manner.

    Figure A depicts an unstable intra-articular and shortened DRF. Incorrect Answers:

    Answers 1, 4 & 5: Systematic reviews and meta-analyses have demonstrated

    no difference in functional outcomes between operative and closed treatment modalities for DRF.

    Answer 2: Systematic reviews and meta-analyses have demonstrated superior radiographic outcomes after surgical treatment when compared to closed treatment.

     

     

     

    OrthoCash 2020

     

  49. An active 60-year-old woman falls from her attic and presents with the injury in Figure A. She undergoes successful closed reduction and sling immobilization. At follow up, she is unable to move her shoulder. New radiographs are depicted in Figures B and C. What is the next best step?

     

     

     

     

     

     

     

     

     

    1. Continued sling immobilization

    2. Closed reduction percutaneous pinning

    3. Open reduction internal fixation

    4. Hemiarthroplasty

    5. Reverse total shoulder arthroplasty Corrent answer: 3

    This active patient presents with a greater tuberosity fracture dislocation. Open reduction internal fixation (ORIF) is indicated, particularly when the greater tuberosity fragment is displaced greater than 5mm.

     

    Many proximal humerus fractures are minimally displaced and respond acceptably to nonoperative management. Isolated greater tuberosity fractures or rotator cuff injuries are associated with shoulder dislocations in the elderly population. The greater tuberosity fragment undergoes deforming forces by the supraspinatus and infraspinatus muscles. In active patients, it is well-accepted that greater tuberosity fracture displacement greater than 5mm is an indication for ORIF to restore their ability to perform overhead activities and prevent impingement.

     

    Schumaier et al. published a review article on the treatment of proximal humerus fractures in the elderly. They highlighted that while bone density was a predictor of reduction quality, social independence was a better predictor of outcome. They concluded that although the majority of minimally displaced fractures can be treated successfully with early physical therapy, treatment for displaced fractures should consider the patient's level of independence, bone quality, and surgical risk factors. They emphasized that there was no clear evidence-based treatment of choice, and the surgeon should consider their comfort level during their decision-making.

     

    George et al. published a review article on greater tuberosity humerus fractures. They reported that these fractures may occur in the setting of anterior shoulder dislocations or impaction injuries against the acromion or superior glenoid, with surgical fixation recommended for fractures with greater than 5 mm of displacement in the general population or greater than 3 mm of displacement in active patients involved in frequent overhead activity. They recommended close followup and supervised rehabilitation to increase successful outcomes.

     

    Figure A depicts a greater tuberosity fracture dislocation of the left shoulder. Figures B and C depict reduction of the glenohumeral joint with residual displacement of the greater tuberosity. Illustrations A and B depict radiographs after ORIF.

     

    Incorrect Answers:

    Answer 1: While most proximal humerus fractures can be treated nonoperatively, greater tuberosity fractures displaced 5 mm or greater should be treated with ORIF.

    Answer 2: ORIF allows for anatomical realignment, reducing the possibility of impingement and weakness from greater tuberosity malunion.

    Answer 4 & 5: Hemiarthroplasty and reverse total shoulder arthroplasty would be indicated in some 4-part proximal humerus fractures in active elderly patients.

     

     

     

     

     

     

     

     

     

    OrthoCash 2020

  50. A 21-year-old football player is tackled as he falls onto an outstretched arm. He sustains the injury shown in Figure A. He undergoes successful operative treatment of his injury. In which order did his injury occur?

     

     

     

     

    1. MCL > LCL > anterior capsule

    2. MCL > anterior capsule > LCL

    3. anterior capsule > MCL > LCL

    4. LCL > anterior capsule > MCL

    5. LCL > MCL > anterior capsule Corrent answer: 4

    The patient sustained a terrible triad injury of the elbow, which progresses from the LCL to the anterior capsule and then the MCL.

     

    Terrible triad injuries of the elbow are traumatic injuries that occur after a fall on an extended arm that results in a combination of valgus, axial, and posterolateral rotatory forces. The key features of a terrible triad injury include a radial head fracture, a coronoid fracture, and an elbow dislocation. Disruption of the structures in the elbow characteristically occurs from lateral to medial, affecting the LCL first, followed by the anterior capsule and MCL. Outcomes following terrible triad injuries have historically been poor; however, more recent literature has shown that good outcomes can be achieved with surgical stabilization of the elbow followed by an early rehabilitation protocol. Some authors use temporary immobilization, but range-of-motion exercises are typically initiated by 48 hours postoperatively. Active range of motion is particularly important, as it recruits muscles that act as dynamic stabilizers of the elbow. Depending on the injury, method of fixation, and stability that is achieved, the range of motion may be limited to 30° of extension during the early postoperative period but should allow full flexion.

    Giannicola et al. (2013) performed a study to determine the critical time period for recovery of functional range of motion after surgical treatment of complex elbow instability (CEI). They found that the first 6 months after surgery represent the critical rehabilitation period to obtain a functional elbow and that elbow flexion recovered at a rate slower than that of the other elbow movements. They recommend that, following CEI surgical treatment, a rehabilitation program should be started promptly and should be continued for at least 6 months because a significant improvement of ROM occurs in this period.

     

    Giannicola et al. (2015) performed a study analyzing the predictability of outcomes of terrible triad injuries (TTI) treated according to current diagnostic and surgical protocols. They found that the current diagnostic and therapeutic protocols allow for satisfactory clinical outcomes in a majority of cases but a high number of major and minor unpredictable complications still persist. Low compliance, obesity, and extensive soft elbow tissue damage caused by high-energy trauma represented negative prognostic factors unrelated to surgery.

     

    McKee et al. performed a review on their standard surgical protocol for the treatment of elbow dislocations with radial head and coronoid fractures. Their surgical protocol included fixation or replacement of the radial head; fixation of the coronoid fracture, if possible; repair of associated capsular and lateral ligamentous injuries; and, in selected cases, repair of the medial collateral ligament and/or adjuvant-hinged external fixation. They found that their surgical protocol restored sufficient elbow stability to allow early motion postoperatively, enhancing the functional outcome. They recommend early operative repair with a standard protocol for these injuries.

     

    Figure A is a lateral radiograph of the elbow demonstrating a terrible triad injury with a comminuted radial head/neck fracture, displaced coronoid fracture, and posterior elbow dislocation. Illustration A is a rendered image of the radiograph shown in Figure A with the components labeled.

     

    Incorrect Answers:

    Answers 1, 2, 3, and 5: Terrible triad injuries occur from the LCL to the anterior capsule and then the MCL.

     

     

     

     

     

     

    OrthoCash 2020

     

  51. An 82-year-old female sustains the fracture shown in Figure A as the result of a ground level fall. Which of the following has been shown to be a reliable predictor of postoperative lateral wall fracture for this injury after treatment with a sliding hip screw?

     

     

     

    1. Reverse obliquity fracture pattern

    2. Lateral wall thickness

    3. Previous contralateral hip fracture

    4. DEXA T-score <-2.0

    5. Calcar comminution

     

    Corrent answer: 2

     

    Lateral wall thickness has been shown to be a predictor of postoperative lateral wall fracture. As the lateral wall thickness decreases, there is an increased chance of fracture.

     

    Lateral wall fracture creates an unstable fracture pattern and increased screw sliding/collapse. This shortens the neck and abductors, leading to worse patient outcomes (radiographic and clinical). Recognition of a thin wall should lead toward the use of an intramedullary device or adjunct use of a trochanteric stabilizing plate with a sliding hip screw device.

     

    Baumgaertner et al. reported that the failure of peritrochanteric fractures that have been treated with a fixed-angle sliding hip-screw device is frequently related to the position of the lag screw in the femoral head. They established the tip-apex distance as the sum of the distance from the tip of the lag screw to the apex of the femoral head on an anteroposterior radiograph and this distance on a lateral radiograph, after controlling for magnification. Upon reviewing their series, none of the 120 screws with a tip-apex distance of twenty-five millimeters or less cut out, but there was a very strong statistical relationship between an increasing tip-apex distance and the rate of cutout, regardless of all other variables related to the fracture.

     

    Socci et al. performed a literature review of relevant papers and appropriate clinical databases and concluded that fixation of AO 31A1 fractures was best achieved with a sliding hip screw device and that all other types of intertrochanteric hip fractures be fixed with an intramedullary device.

     

    Utrilla et al. reported no difference in outcome in stable fractures, but better mobility at one year following intramedullary fixation of unstable fractures.

     

    Hsu et al. measured the thickness of the lateral wall of patients with AO/OTA 31-A1 and 31-A2 type intertrochanteric hip fractures. They found that the lateral wall thickness was a reliable predictor of postoperative lateral wall fracture for unstable AO Type A2 fractures and concluded that the lateral wall thickness threshold value for risk of developing a secondary lateral wall fracture was found to be 20.5 mm.

    Figure A shows a standard obliquity intertrochanteric hip fracture.

    Illustration A from the Hsu article demonstrates the measurement of the lateral wall thickness. The distance is measured along a 135-degree angle, between a point 3cm distal to the innominate tubercle of the greater trochanter and the fracture line (midway between the two cortical lines).

     

    Incorrect Answers:

    Answer 1: This pattern is unstable and requires treatment with an intramedullary nail as opposed to a sliding hip screw.

    Answer 3: There has been no proven association with this factor. Answer 4: There has been no proven association with this factor.

    Answer 5: Fracture comminution is not associated unless the lateral wall is already fractured or comminuted itself.

     

     

     

     

     

     

    OrthoCash 2020

     

  52. A 78-year-old patient presents with right hip pain and inability to bear weight after an unwitnessed fall at a nursing home. Figures A and B are the radiographs of the hip and pelvis. Which statement is true regarding the treatment of these injuries?

     

     

     

     

     

     

    1. Smaller lateral wall thickness favors sliding hip screw constructs

    2. Unstable fractures are best treated with sliding hip screw constructs

    3. Avoiding distal locking screws in intramedullary implants protects against refracture

    4. Stable fractures have no differences in outcomes between sliding hip screws and intramedullary implants

    5. Implant stability has a greater impact on outcomes rather than reduction quality

    Corrent answer: 4

     

    Studies have shown that in stable intertrochanteric femur fractures there are no differences in outcomes between sliding hip screws and intramedullary implants.

     

    Intertrochanteric femur fractures are one of the most common fractures in the geriatric population. Implant selection has been a great topic of research with most studies reporting minimal to no differences in outcomes between intramedullary and sliding hip screw constructs in stable fracture patterns.

    Unstable fractures, however, are reportedly better treated with a distally locked intramedullary implant. The quality of fracture reduction has a greater impact on the overall outcome than implant selection.

     

    Hsu et al. performed a retrospective study of risk factors for postoperative lateral wall fractures in patients treated with sliding hip screws for intertrochanteric femur fractures. They found that fracture classification and lateral wall thickness, which is measured from 3 cm distal from innominate tubercle and angled 135 degrees to the fracture line, were associated with postoperative lateral wall fracture. They recommended not treating intertrochanteric femur fractures with sliding hip screws if the lateral wall thickness is less than 20.5 mm.

     

    Socci et al. reviewed the literature regarding the treatment of intertrochanteric femur fractures. Based on the literature, they recommend treatment of AO/OTA type 31A1 fractures with sliding hip screws, type 31A2 fractures with short intramedullary implants, and 31A3 fractures with long intramedullary implants. Simple basicervical fractures of the femoral neck can be treated with sliding hip constructs whereas comminuted fractures treated with intramedullary devices due to the inherent instability of the pattern. The most import aspect in fracture healing is the quality of the reduction rather than the choice of implant.

     

    Lindvall et al. performed a retrospective study of refracture rates in patients treated with either long or short cephalomedullary nails. The authors found a 97% union rate with both implant types and refracture not associated with either long or short implants. Rather, refracture was associated with the lack of a distal locking screw. The authors recommended locking intramedullary implants to avoid refracture.

     

    Utrilla et al. performed a randomized control trial of elderly patients treated

    with compression hip screw or Trochanteric Gamma Nail for intertrochanteric femur fractures. They reported the only differences between the two implants were quicker operating time, less fluoroscopy use, and better walking with unstable fractures treated with intramedullary implants. The authors recommended either construct for stable fractures, but intramedullary implants for unstable fractures.

     

    Figures A and B are the AP and lateral radiographs of the right hip radiographs demonstrating a simple and minimally displaced intertrochanteric femur fracture, classified as an AO/OTA 31A1 fracture. Illustration A depicts the AO/OTA classification system for proximal femur fractures.

     

    Incorrect answers:

    Answer 1: Thinner lateral wall thickness in intertrochanteric femur fractures favors the use of an intramedullary implant due to the risk of lateral wall fracture.

    Answer 2: Unstable intertrochanteric femur fractures are best treated with intramedullary implants.

    Answer 3: The lack of distal locking screws has been shown to increase the risk of refracture.

    Answer 5: Reduction quality has been shown in several studies to have a greater impact on outcomes rather than implant selection.

     

     

     

     

     

    OrthoCash 2020

     

  53. A 28-year-old male that sustained a closed left femoral shaft fracture 12 months ago and underwent intramedullary nailing presents with persistent pain in the right thigh. The patient walks with an antalgic gait. He denies any fevers or chills. His surgical sites are well healed and there are no signs of drainage. Serum ESR and CRP are 12 mm/hr (reference <20 mm/hr) and 0.9 mg/L (reference <2.5 mg/L), respectively. Figures A and B are the AP and lateral radiographs of the left femur. Which treatment option offers the highest chance of union and enables immediate weight-bearing?

     

     

     

    1. Nail removal with compression plating and open bone grafting

    2. Closed reamed exchange nailing

    3. Nail dynamization

    4. Nail retention with plate augmentation and bone grafting

    5. Electrical bone stimulator Corrent answer: 4

    The patient is presenting with a hypertrophic nonunion of the femur below the isthmus, which studies have shown to have a higher union rate when treated with plate augmentation. Retention of the nail allows for full weight-bearing postop.

     

    Hypertrophic nonunion of the femur is the result of fracture site hypermobility with sufficient biology for healing. This is demonstrated with abundant callus formation without bridging trabeculae. Traditionally, this is treated with closed reamed exchange nailing which increased construct stiffness with a larger diameter nail, improved isthmic fit, and extrusion of reaming contents to the nonunion site. However, studies have demonstrated a higher union rate with open plate augmentation, bone grafting, and nail retention. This is due to the ability to correct nonunion site deformity, provide added compression at the nonunion site, and increase fracture site biology with bone graft.

    Lynch et al. reviewed the literature regarding the treatment options for femoral nonunions. The literature suggests high union rates when hypertrophic nonunions are treated with exchanged reamed nailing. However, the use of augmentative plate fixation allows for further deformity correction. The proposed mechanism by which exchange reamed nailing is increased construct stiffness with a large diameter nail, usually by 1-2 mm, increased isthmic fit, and autogenous bone graft extrusion into the nonunion site.

     

    Somford et al. performed a systematic review of the surgical treatment of femoral nonunions. Results demonstrate that exchange nailing provides a 73% union rate compared to plate augmentation of 96%. They speculated that there were increased indications for exchange nailing for oligotrophic nonunions in many of the included studies, which may have reduced the union rate. Further, plate augmentation does allow for deformity correction, which can further improve the union rate.

     

    Figures A and B are the AP and lateral radiographs of the femur with hypertrophic nonunion as suggested with the abundant callus formation and broken distal interlock screws. Illustration A and B are the AP and lateral radiographs of the distal femur subsequent plate augmentation and fracture healing.

     

    Incorrect Answers:

    Answer 1: Compression plating and bone grafting provide deformity correction across the nonunion site and added biology, but there is loss of weight-bearing benefits with nail removal.

    Answer 2: Closed reamed exchange nailing can provide healing of the nonunion by increasing the nail diameter and improving the isthmic fit, but studies have shown a lower union rate in subisthmic femoral nonunions compared to augmentation plating.

    Answer 3: Nail dynamization provides the benefit of weight-bearing forces across the nonunion site to enhance the differentiation of osteoblasts and promote fracture healing. In hypertrophic nonunions, fracture site stability is the cause of nonunion rather than the lack of fracture site biology.

    Answer 5: Electrical bone stimulators do not provide the added fracture stability that is required to address hypertrophic nonunion.

     

     

     

     

     

     

     

    OrthoCash 2020

     

  54. A 25-year-old male sustains the injury depicted in Figure A. He is splinted in the field, but on arrival to the emergency room, he complains of painful "tightness" around the leg and severe uncontrolled pain despite maximum dose narcotics. His pain is exacerbated when the toes and ankle are passively stretched in flexion and extension. What is the most appropriate next step in treatment?

     

     

     

     

    1. External fixation with serial doppler examinations

    2. Intramedullary nailing

    3. Open reduction internal fixation using plates and screws

    4. Immediate 2-compartment fasciotomies and external fixation

    5. Immediate 4-compartment fasciotomies and external fixation Corrent answer: 5

    This patient has clinical symptoms and signs of leg compartment syndrome and should undergo immediate fasciotomies of all 4 leg compartments, followed by external fixation for fracture stabilization.

     

    Tibial fractures are among the most common reasons for compartment syndromes of the leg. A clinical assessment is key in the diagnosis of acute compartment syndrome. If there is uncertainty, intracompartmental pressure measurement has been advocated to help confirm the diagnosis. An absolute

    compartment pressure >30 mm Hg or a difference in diastolic pressure and compartment pressure (delta p) <30 mmHg may help to confirm the necessity for fasciotomy.

     

    McQueen et al. published a report of 25 patients with tibial diaphyseal fractures which had been complicated by an acute compartment syndrome. They reported significant differences in any sequelae of acute compartment syndrome between patients who underwent compartment pressure monitoring and those who had not. They recommended that all patients with tibial fractures should have continuous compartment monitoring to minimize the incidence of acute compartment syndrome.

     

    Mawhinney et al. reported on three cases of tibial compartment syndrome after closed intramedullary nailing of the tibia. They reported that the only predisposing factors for the development of compartment syndrome were the surgery and the fracture itself. They concluded that tibial compartment syndrome is a relatively rare but significant complication of tibial nailing.

     

    Figure A is an AP and lateral radiograph of the leg with displaced, comminuted middle third tibia and fibula fractures.

     

    Incorrect Answers:

    Answers 1, 2, 3, & 5: This patient is presenting with signs of acute compartment syndrome, and should undergo immediate 4-compartment fasciotomies.

     

     

     

    OrthoCash 2020

     

  55. A 24-year-old male is brought to the ED after an MVC. He is found to have a closed comminuted segmental fibula fracture after a prolonged extraction from the vehicle. Several hours after arrival, the patient reports increasing pain and is noted to have an exacerbation of his pain with passive stretching of the ankle. He has a heart rate of 103 and a blood pressure of 141/87. Compartment pressures are obtained and are 27 mmHg in the anterior compartment, 47 mmHg in the lateral compartment, 28 mmHg in the superficial posterior compartment, and 27 mmHg in the deep posterior compartment. Which of the following correctly describes the initial pathophysiology of compartment syndrome and the neurologic deficit that would likely occur in this patient if left untreated?

    1. Decreased arterial inflow; decreased sensation on the dorsum of his foot involving the first webspace

    2. Decreased arterial inflow; decreased sensation on the dorsum of his foot involving the hallux, 3rd, and 4th toes

    3. Decreased arterial inflow; inability to dorsiflex his ankle

    4. Decreased venous outflow; decreased sensation on the dorsum of his foot involving the first webspace

    5. Decreased venous outflow; decreased sensation on the dorsum of his foot involving the hallux, 3rd, and 4th toes

    Corrent answer: 5

     

    Compartment syndrome initially results from a decrease in venous outflow relative to arterial inflow. This patient has elevated pressures in the lateral compartment of the leg, which is where the superficial peroneal nerve runs to supply sensation to the dorsum of the foot including the hallux and 3rd and 4th toes.

     

    Compartment syndrome results from compromised venous outflow from the leg relative to the arterial inflow. This venous congestion leads to elevated compartment pressures that ultimately lead to compromised arterial inflow without compartment release. There are 4 compartments in the leg: anterior, lateral, superficial posterior, and deep posterior. The anterior compartment contains the deep peroneal nerve, the lateral compartment of the leg contains the superficial peroneal nerve, and the deep posterior compartment contains the tibial nerve.

     

    McQueen et al. performed a study to determine risk factors for acute compartment syndrome. They found that young patients, especially men, were most at risk of acute compartment syndrome after injury. They recommend that, when treating such injured patients, the diagnosis should be made early, utilizing measurements of tissue pressure.

     

    Olson et al. published a review on acute compartment syndrome in lower extremity musculoskeletal trauma. They reported that acute compartment syndrome is a potentially devastating condition in which the pressure within an osseofascial compartment rises to a level that decreases the perfusion gradient across tissue capillary beds, leading to cellular anoxia, muscle ischemia, and death. They report that recognizing compartment syndromes requires having and maintaining a high index of suspicion, performing serial examinations in patients at risk, and carefully documenting changes over time.

     

    Illustration A is a diagram depicting the compartments of the leg and its contents.

    Incorrect Answers:

    Answers 1, 2, 3, and 4: Compartment syndrome initially results from a decrease in venous outflow relative to arterial inflow. The superficial peroneal nerve runs in the lateral compartment and provides sensation to the dorsum of the foot including the hallux and 3rd and 4th toes.

     

     

     

     

     

     

    OrthoCash 2020

     

  56. Which of the following amputations results in an approximate 40% increase in energy expenditure for ambulation?

    1. Syme

    2. Traumatic transtibial

    3. Vascular transtibial

    4. Traumatic transfemoral

    5. Vascular transfemoral

     

    Corrent answer: 3

     

    The energy expenditure of a vascular transtibial amputation is approximately 40% greater.

     

    The energy expenditure for ambulation increases with lower extremity amputation. Diabetics and vasculopathic patients who undergo amputation

    have significantly increased energy requirements compared with nondiabetic patients undergoing amputations for trauma. The metabolic cost for a vascular transtibial amputation is 40% compared to a 25% increase in normal patients who sustain a traumatic amputation.

     

    Huang et al. used a mobile instrument system to measure energy consumption by indirect calorimetry at rest and during ambulation in 25 unimpaired subjects, 6 unilateral below-knee (BK) amputee patients, 6 unilateral above-knee (AK) amputee patients and 4 bilateral AK amputee patients. They found that in comparison to unimpaired subjects, the mean oxygen consumption was 9% higher in unilateral BK amputee patients, 49% higher in unilateral AK amputee patients and 280% higher in bilateral AK amputee patients.

     

    Pinzur et al. performed a study to measure cardiac function and oxygen consumption in 25 patients who underwent amputation for peripheral vascular disease (PVD), and in five similarly aged control patients without PVD. They found Normal walking speed and cadence decreased and oxygen consumption per meter walked increased with more proximal amputation. They conclude that peripheral vascular insufficiency amputees function at a level approaching their maximum functional capacity and more proximal amputation levels, the capacity to walk short or long distances is greatly impaired.

     

    Incorrect Answers:

    Answer 1: A Syme amputation increases energy expenditure by 15%.

    Answer 2: A traumatic transtibial amputation increases energy expenditure by 25%.

    Answer 4: A traumatic transfemoral amputation increases energy expenditure by 68%.

    Answer 5: A vascular transfemoral amputation increases energy expenditure by 100%.

     

     

     

    OrthoCash 2020

     

  57. A 25-year-old man sustains the injury shown in Figures A-C. What is the primary advantage of using a trochanteric flip osteotomy (TFO) in treating this injury?

     

     

     

     

     

     

     

     

    1. It may be performed in a minimally invasive manner

    2. It involves minimal soft tissue stripping

    3. It leads to higher union rates

    4. It allows the surgeon to address all sites of injury through one approach

    5. Answers 1 and 2

     

    Corrent answer: 4

     

    This patient has sustained a right hip fracture-dislocation with fractures of the femoral head and posterior wall. The TFO allows the surgeon to address all sites of injury through a single approach.

     

    Femoral head fracture-dislocations are a result of high-energy trauma. Treatment ranges from closed reduction and conservative management to total hip arthroplasty. Intermediate options include open reduction and internal fixation or excision of fracture fragments. Complications of this injury include post-traumatic hip arthritis, avascular necrosis, and heterotopic ossification.

    The injury is further complicated when a fracture of the acetabulum is concomitantly present. There has been no consensus treatment on this injury constellation as it presents quite rarely. The TFO is one approach that allows the surgeon to treat and stabilize both injuries concurrently. It should be noted that a surgical hip dislocation is performed in conjunction with the TFO to allow access to the femoral head.

     

    Solberg et al. performed a retrospective study of patients sustaining Pipkin IV fracture/dislocations with a TFO. They had 12 patients over a 6 month period. They found that all patients healed radiologically and one patient developed osteonecrosis. 10 out of 12 patients had good to excellent outcomes. They concluded that using a surgical protocol with TFO rendered clinical results

    comparable to previously reported outcomes in a series of isolated femoral head fractures.

     

    Giannoudis et al. performed a systematic review to investigate data regarding femoral head fractures, particularly focusing on their management, complications and clinical results. They reported that fracture-dislocations were managed with emergent closed reduction, followed by definite treatment, aiming at an anatomic restoration of both fracture and joint incongruity. They concluded that neither the TFO nor an anterior approach seems to endanger femoral head blood supply compared to the posterior one, with the TFO possibly providing better long-term functional results and lower incidence of major complication rates.

     

    Henle et al. reported on the result of 12 patients of femoral head fractures with associated posterior wall fractures treated with a TFO. They found good to excellent results in 10 patients. The two patients with poor outcome developed avascular necrosis of the femoral head and underwent total hip arthroplasty.

    Heterotopic ossification was seen in five patients. They concluded that the TFO may lead to favorable outcomes in this injury constellation.

     

    Figure A is an AP radiograph of the right hip demonstrating a femoral head fracture-dislocation. Figure B is an axial CT image demonstrating a posterior wall fracture. Figure C is an axial CT image demonstrating a femoral head fragment within the acetabulum. Illustration A is the Pipkin classification of femoral head fractures: Type I is below the fovea, Type II is above the fovea, Type III is associated with a femoral neck fracture, and Type IV is associated with an acetabular fracture.

     

    Incorrect Answers:

    Answers 1, 2, & 5: The TFO is not a minimally invasive approach.

    Answer 3: There is no evidence that shows that this approach leads to higher union rates of this injury

     

     

     

     

     

    OrthoCash 2020

     

  58. A 30-year-old male is brought to your emergency department following a motor vehicle collision at high speed. He is intubated in the field for airway protection but is hemodynamically stable. Subsequent workup shows a displaced acetabular fracture, in addition to an intracranial bleed and liver laceration which do not require surgery. When placing an antegrade anterior column screw, what radiographic view should be used to avoid intra-pelvic screw penetration?

    1. Iliac oblique view with hip and knee flexed

    2. Iliac oblique inlet view

    3. Obturator oblique view with hip and knee flexed

    4. Obturator oblique outlet view

    5. Obturator oblique inlet view Corrent answer: 2

    The iliac oblique inlet view will best show the the anterior-posterior placement of an anterior column ramus screw.

    Percutaneous and limited-open acetabular fixation is becoming increasingly common as it avoids the morbidity of extensile pelvic dissection and allows early mobilization. However, it relies heavily on a mastery of radiographic landmarks and ability to interpret these images to reduce fracture fragments without direct visualization. Slight deviations of the fluoroscopy beam and/or fracture displacement will distort the radiographic image. Without a facile ability to interpret these and make appropriate adjustments, percutaneous fixation will be extremely onerous.

     

    Starr et al. described their early techniques for percutaneous and limited-open acetabular fixation. They first implemented this for minimally displaced fracture patterns but have expanded these to a wider range of pathology. They cite the benefit of earlier mobilization in the poly-traumatized patient as great use for this technique.

     

    Mauffrey et al. reviewed radiograph utilization during acetabular fracture care. Though CT has added tremendously to demonstrating subtleties of acetabular fractures, they state the use of AP and orthogonal iliac and obturator oblique Judet views cannot be overlooked. Interpreting these radiographs allows the surgeon to recreate 2-dimensional images into a 3-dimensional fracture pattern and better understand the character of the injury.

     

    Illustrations A and B demonstrate the iliac oblique inlet view and obturator oblique outlet views, respectively.

    Illustration C demonstrates the relationship of the critical structures at risk of injury during anterior column screw placement. Illustrations D and E show the starting point with screw trajectory, and position of the hip during posterior column screw.

     

    Incorrect Answers:

    Answers 1 and 3- These would be utilized when placing a posterior column screw.

    Answer 4- This view would show the supra-acetabular corridor with help with the starting point for external fixator placement.

    Answer 5- This shows the inner and outer tables of the iliac wing and confirms pins have not violated the cortex.

     

     

     

     

     

     

     

     

     

     

     

     

     

     

     

     

    OrthoCash 2020

     

  59. A 34-year-old male sustains the injury shown in Figures A and B. Which factor has been found to be elevated in the synovial fluid and contributes to post-traumatic arthritis?

     

     

     

     

     

     

    1. TGF-Beta

    2. RANKL

    3. IL-2

    4. IL-6

    5. cAMP

    Corrent answer: 4

     

    The patient has sustained a tibial plafond or pilon fracture as depicted in Figures A and B. IL-6 is one of many inflammatory molecules that has been found to be elevated in the synovial fluid following an intra-articular ankle fracture.

     

    Post-traumatic arthritis following intra-articular fractures is a known complication. It commonly appears 1-2 years following injury and is related to chondrocyte death at the margins. There has not been shown to be any association between prolonged non-weight bearing, poor patient compliance with weight-bearing restrictions, and hardware reactions with the development of post-traumatic arthritis. However, literature has shown that the inflammatory molecules present in the synovial fluid can have a significant effect on the development of posttraumatic arthritis. Important inflammatory factors that have been found to be elevated include IL-6, IL-8, MMP-1, MMP-2, MMP-3, MMP-9 and MMP-10.

     

    Adams et al. looked at the synovial fluid of 21 patients with an intra-articular ankle fracture and used the un-injured ankle as a control. They found the inflammatory molecules of GM-CSF, IL-10, IL-1 beta, IL-6, IL-8, IL-10, IL-12p70, TNF-alpha, MMP-1, MMP-2, MMP-3, MMP-9, MMP-10 were all elevated. They concluded that these inflammatory molecules may play a role in posttraumatic arthritis development.

     

    Adams et al. looked at the synovial fluid of 7 patients from his previous 21 patients that had intra-articular ankle fractures. They found that IL-6, IL-8, MMP-1, MMP-2, and MMP-3 were significantly elevated in comparison to the uninjured ankle. They concluded that the sustained elevated intra-articular inflammatory environment is a potential contributor to post-traumatic arthritis.

     

    Figures A and B are sagittal and axial CT slices, respectively, that depict a tibial plafond or pilon fracture.

     

    Incorrect Answers:

    Answer 1: TGF-Beta has not been shown to be elevated in the synovial fluid and contribute to post-traumatic arthritis. However, TGF-Beta has been shown to be a factor in the inflammatory cycle, immune system signaling and bone signaling (inhibits osteoclasts via osteoprotegrin).

    Answer 2: RANKL has not been shown to be elevated in the synovial fluid and contribute to post-traumatic arthritis. RANKL is an important part of bone signaling and metabolism.

    Answer 3: IL-2 has not been shown to be elevated in the synovial fluid and contribute to post-traumatic arthritis. IL-2 has important functions within the

    immune system.

    Answer 5: cAMP has not been shown to be elevated in the synovial fluid and contribute to post-traumatic arthritis. cAMP is an intra-cellular signaling molecule.

     

     

     

    OrthoCash 2020

     

  60. A 29-year-old female presents to the trauma bay from the scene of a high-speed motor vehicle accident. She is found to have a closed intraarticular distal radius fracture with a concomitant ulnar styloid base fracture. She subsequently undergoes ORIF of the distal radius fracture with a volar locking plate. The ulnar styloid fracture is not addressed. Which of the following, if present, is least likely to affect functional outcomes?

    1. Ulnar styloid nonunion

    2. DRUJ instability

    3. Articular step-off >3mm

    4. Radial shortening

    5. Workers compensation claim Corrent answer: 1

    Ulnar styloid non-unions do not affect the overall outcome of hand or wrist function following ORIF of distal radius fractures.

     

    Ulnar styloid base fractures can be associated with DRUJ disruption and TFCC rupture. The DRUJ should be independently evaluated following ORIF of the distal radius. Without instability, unlar styloid fractures do not need to be addressed. If instability exists, the DRUJ should be treated as a separate entity, typically cross-pinned using k-wires. The result of ulnar styloid nonunions are inconsequential to the overall outcome of patients undergoing distal radius ORIF.

     

    Daneshvar et al review the effects of ulnar styloid fractures on patients sustaining distal radius fractures. They report that patients with a concomitant ulnar styloid fracture had a slower recovery of wrist flexion and grip strength compared to those with an isolated distal radius fracture. They conclude, however, that even the presence of an ulnar styloid nonunion did not significantly affect outcomes.

     

    Buijze et al review the clinical impact of united versus non-united fractures of the proximal half of the ulnar styloid following volar plate fixation of the distal radius. They report no difference in motion, strength or outcome scores

    between the united and non-united groups at 6 months follow up. They conclude that nonunion of the ulnar styloid does not have an effect on the overall outcome of hand or wrist function.

     

    Incorrect Answers:

    Answer 2: Unrecognized DRUJ instability leads to decreased pronosupination and posttraumatic arthritis.

    Answer 3: With 1-2mm of articular step-off 90% of young adults will develop radioulnar arthrosis

    Answer 4: Radial shortening is one of the most important factors for predicting poor functional outcomes following distal radius ORIF.

    Answer 5: Workers compensation claims negative impact functional outcomes following fixation of distal radius fractures.

     

     

     

    OrthoCash 2020

     

  61. Which of the following proximal humerus fractures has the highest likelihood of developing humeral head ischemia?

     

     

     

     

     

     

     

     

     

     

     

    1. Figure A

    2. Figure B

    3. Figure C

    4. Figure D

    5. Figure E

    Corrent answer: 3

     

    Posteromedial calcar length of the humeral head less than 8 mm and a loss of medial hinge are among the most reliable predictors of ischemia in the surgical management of humeral head fractures.

     

    Proximal humerus fractures are classified based on the Neer classification, in which 4 parts are described: greater tuberosity, lesser tuberosity, articular surface, and the shaft. A fragment is considered a part if it is greater than 45 degrees angulated or displaced >1cm. The posterior humeral circumflex artery is the main blood supply to the humeral head. Following ORIF, humeral head ischemia may occur and is associated with the initial fracture pattern. Several factors including <8mm of calcar length attached to the articular segment, disruption of the medial hinge, displacement >10mm and angulation >45 degrees have been associated with a disruption of the vascular supply to the humeral head.

     

    Campochiaro et al review Hertel’s criteria of calcar length and medial hinge integrity and its reliability in predicting humeral head necrosis. They reported a 3.7% incidence of ischemia across all 267 fractures evaluated. In those patients that developed AVN, 30% had all of the predictors described by Hertel, however, in the non-AVN group, only 4.7% had these same findings.

    They concluded that while Hertel’s criteria are helpful, they may not be sufficient and the authors recommended 3-dimensional evaluation of any fracture involving the calcar.

     

    Xu et al reviewed avascular necrosis in patients with proximal humerus fractures who were treated surgically. They reported on 291 patients throughout 7 studies in which there was no difference in the incidence of AVN for those treated surgically or nonoperatively. However, they concluded through subgroup analysis looking at different fixation constructs that, plate fixation specifically was associated with a higher risk of AVN than conservative management of proximal humerus fractures.

     

    Figure A demonstrates a proximal humerus fracture with a medial calcar length of >8mm attached to the articular segment. Figure B is a proximal humerus fracture with a displaced greater tuberosity fragment. Figure C demonstrates a proximal humerus fracture with a medial calcar length of <8mm attached to the articular segment. Figure D is a displaced metadiaphyseal proximal humerus fracture in a skeletally immature patient. Figure E is a radiograph of a metadiaphyseal proximal humerus fracture in a skeletally mature patient with a medial calcar length >8mm.

     

    Incorrect Answers:

    Answer 1, 2, 5: These fracture patterns do not have the prognostic characteristics described by Hertel for humeral head ischemia: articular segment calcar length <8mm, displacement >10mm or angulation >45 degrees.

    Answer 4: Humeral head ischemia is very rare following pediatric proximal humerus fractures.

     

     

     

    OrthoCash 2020

     

  62. A 34-year-old man presents with the closed injury depicted in Figure A after a high energy twisting injury. Which of the other injuries below is most commonly associated with his known injury?

     

     

     

     

    1. Nondisplaced medial malleolus vertical shear fracture

    2. Nondisplaced Volkmann's fragment

    3. Nondisplaced Chaput's fragment

    4. Nondisplaced lateral wall talar fracture

    5. Posterior inferior tibiofibular ligament disruption Corrent answer: 2

    This patient has sustained a distal third tibial shaft spiral fracture, which is commonly associated with nondisplaced posterior tibial plafond fractures, with the classic Volkmann's fragment.

     

    Prior to operative management, distal third spiral tibial shaft fractures should always be evaluated for intra-articular extension. As this commonly associated injury can be missed on plain radiographs, an ankle CT is often recommended. This is especially important when intramedullary fixation is used for definitive management of the tibial shaft fracture, as nail insertion can displace a

    previously nondisplaced intraarticular fracture. Anterior to posterior lag screw fixation prior to nailing may be useful in these cases.

     

    Sobol et al. investigated the incidence of concomitant posterior malleolar fractures (PMFs) in operative distal third spiral tibial shaft fractures. They reported that spiral distal third tibial shaft fractures were identified with an ipsilateral posterior malleolus fracture in 92.3% of cases. They recommended a preoperative ankle CT in all cases with this specific fracture morphology to properly diagnose this commonly associated injury.

     

    Hou et al. investigated the posterior malleolar fracture association with spiral tibial shaft fractures. They reported that plain radiography (both preoperative and intraoperative) resulted in rare identification of these associated injuries, which resulted in missed injuries. They concluded that a CT or MRI ankle may be a higher yield method to detect these injuries.

     

    Figure A demonstrates a distal third spiral tibial shaft fracture. Illustration A is a schematic demonstrating the Volkmann, Chaput, and medial malleoli intraarticular fragments of the distal tibia.

     

    Incorrect Answers:

    Answer 1: Medial malleolar vertical shear fractures are not typically associated with distal third tibial shaft fractures.

    Answer 3: Nondisplaced anterior tibial plafond fractures (Chaput's fragment) are not as highly associated with spiral distal third tibial shaft fractures.

    Answer 4: Nondisplaced lateral wall talar fractures are not associated with spiral distal third tibial shaft fractures.

    Answer 5: The PITFL is typically intact with posterior malleoli fractures.

     

     

     

     

     

    OrthoCash 2020

     

  63. An 18-year-old male is admitted for a diaphyseal, open, tibial shaft fracture after falling off a motorcycle. He has a past medical history of nicotine dependence and obesity. He undergoes provisional splinting by the resident on call and is noted to be "neurovascularly intact" following splint placement. Throughout the evening, however, the patient has an increasing narcotic requirement and develops pain with passive stretch of his toes. What factor listed below is most associated with his progressive symptoms overnight?

    1. Age < 20

    2. Male gender

    3. Body mass index >/ 30 kg/m^2

    4. Open fracture

    5. Nicotine use

     

    Corrent answer: 1

     

    The highest prevalence of compartment syndrome is found in patients aged 12-19 years, followed by 20-29 years.

     

    One theory for the higher prevalence of compartment syndrome in younger patients is increased muscle mass in this cohort. If there is more muscle in a compartment, there is less room for swelling. On the flip side, elderly or deconditioned patients who have less muscle or fatty atrophy may be better able to accommodate muscle swelling. Additionally, a diaphyseal fracture location is associated with a higher risk of compartment syndrome. Again, this may be due to the fact that there is more muscle than tendon, and thus more swelling, in the proximal leg.

     

    Shadgan et al. retrospectively reviewed 1,125 patients with diaphyseal tibia fractures to look for risk factors associated with the development of compartment syndrome. Compartment syndrome occurred in approximately 8% of patients with this injury. They concluded that younger patients were at a higher risk of developing compartment syndrome and that male gender, open fracture, and intramedullary nailing were not risk factors.

     

    Beebe et al. set out to determine the correlation between the OTA/AO classification of tibia fractures and the development of compartment syndrome. they conducted a retrospective review of a prospectively collected database comprising 2,885 fractures. They concluded that age, sex, and the OTA/AO classification were highly predictive for the development of compartment syndrome in this cohort.

    McQueen et al. similarly looked at predictors of compartment syndrome after tibial fractures in a retrospective cohort study. There were 1,388 patients in their study with ages ranging from 12-98; identical to the Shadgan study, 69% of patients were male. They concluded the strongest risk factor was age, with the highest prevalence in 12 to 19-year-olds.

     

    Park et al. additionally analyzed 414 patients with tibia fractures in a retrospective cohort study. The main outcome measure of this study was the rate of clinically determined compartment syndromes requiring fasciotomy by anatomic region. The found that diaphyseal fractures were more frequently associated with the development of compartment syndrome than proximal (next most common site) and distal tibia fractures, specifically in younger patients.

     

    Incorrect Answers:

    Answers 2-5: Gender, body mass index, open fracture, and nicotine use are not associated with a higher risk for the development of a compartment syndrome.

     

     

     

    OrthoCash 2020

     

  64. Figure A is the radiograph of a 79-year-old female with elbow pain following a fall. Compared with a total elbow artrhoplasty, open reduction and internal fixation would most likely result in?

     

     

     

    1. Greater Mayo Elbow Performance Score

    2. Greater Disabilities of the Arm, Shoulder and Hand Score

    3. Increased flexion-extension arc

    4. Increased reoperation rate

    5. Decreased complication rate Corrent answer: 4

    This patient sustained a comminuted distal humerus fracture. Open reduction and internal fixation (ORIF) is found to have higher repoeration rates compared with total elbow arthroplasty (TEA) in the elderly: 27% versus 12%, respectively.

     

    Distal humerus fractures account for approximately 30% of elbow fractures. There is often a low energy mechanism of injury in the elderly patient. While ORIF and TEA may be utilized in bicolumnar distal humerus fractures in the elderly patient, recent literature has demonstrated favorable outcomes with TEA in this aged cohort. TEA is indicated in the low demand osteoporotic patients with bicolumnar distal humerus fractures that are not amendable to ORIF. Utilization of TEA has demonstrated greater functional outcome scores, greater motion, less complications, and a lower revision rate.

     

    Mckee et al. conducted a prospective, randomized, controlled trial comparing functional outcomes, complications, and reoperation rates in elderly patients with displaced intra-articular, distal humeral fractures treated with ORIF or primary semiconstrained TEA. They reports that patients who underwent TEA had significantly better motion, performance and outcome scores, lower reoperation rates compared with the ORIF group. They concluded that TEA for the treatment of comminuted intra-articular distal humeral fractures resulted in more predictable and improved 2-year functional outcomes compared with ORIF and that TEA is a preferred alternative for ORIF in elderly patients with complex distal humeral fractures that are not amenable to stable fixation.

     

    Githens et al. performed a systematic review and meta-analysis to analyze outcomes and complication rates in elderly patients with intra-articular distal humerus fractures being treated with either TEA or ORIF with locking plates. They report that TEA and ORIF for the treatment of geriatric distal humerus fractures produced similar functional outcome scores and range of motion.

    However, they found a non-statistical trend toward a higher rate of major complications and reoperation after ORIF. They conclude that the quality of study methodology was generally weak and ongoing research including prospective trials and cost analysis is indicated.

     

    Figure A is the AP radiograph of a comminuted bicolumnar distal humerus

    fracture. Illustration A are the radiographs of a comminuted distal humerus fracture in an elderly patient treated with a TEA. Illustration B is the postoperative radiographs of a comminuted distal humerus fracture treated with ORIF.

     

    Incorrect Answers:

    Answers 1-3,&5: TEA has demonstrated greater functional outcome scores, greater motion, less complications, and a lower revision rate compared with ORIF.

     

     

     

     

     

     

     

     

     

    OrthoCash 2020

     

  65. Figure A is the postoperative radiograph of an 82-year-old female who was treated with a cephalomedually nail for a left intertrochanteric hip fracture. Which of the following is the most common complication following use of this device?

     

     

     

    1. Anterior perforation of distal femur

    2. Breakage of the screw

    3. Implant cutout

    4. Malunion

    5. Nonunion

     

    Corrent answer: 3

     

    Intertrochanteric hip fractures are most commonly treated with a cephalomedullary nail. The most common complication following utilization of a cephalomedullary nail is implant failure and cutout.

     

    Intertrochanteric hip fractures are extra-capsular injuries that are common in the elderly osteoporotic patient. These injuries carrry a 20-30% mortality rate in the first year following fracture. Femoral cephalomedullary nails are often used to treat these injuries. Often a helical blade or screw may be used to provide fixation within the femoral neck. Overall, the most common complication following use of this device is implant failure and cutout, which occurs most commonly within 3 months following surgery. A known risk factor of this complication is an increased tip-apex distance, with a 60% failure rate reported with a distance exceeding 45mm.

     

    Gardner et al. reviewed the use of a helical blade device to stabilize

    intertrochanteric hip fractures. They reported a mean telescoping in unstable and stable fractures of 4.3 mm and 2.6 mm, respectively. They also found that blade migration within the femoral head averaged 2.2 mm overall, with no difference between stable and unstable fractures. They concluded that position changes occurred within the first 6 weeks postoperatively, with no subsequent detectable migration or telescoping.

     

    Haidukewych et al. reviewed patients with failed internal fixation of a hip fracture. They report that salvage options are dependent on the anatomic site of the nonunion, the quality of the remaining bone and articular surface, and patient factors such as age and activity level. They conclude that in younger patients with either a femoral neck or intertrochanteric fracture nonunion with a satisfactory hip joint, treatment typically involves revision internal fixation with or without osteotomy or bone grafting. Conversely, in older patients with poor remaining proximal bone stock or a badly damaged hip joint, conversion to hip arthroplasty is recommended.

     

    Figure A is the AP radiograph of the right hip treated with a cephalomedullary nail. Illustration A demonstrates screw cutout.

     

    Incorrect Answers:

    Answers 1,2,4,5: These are not the most common complications following use of a cephalomedullary nail

     

     

     

     

     

    OrthoCash 2020

     

  66. Figure A is the radiograph of a 42-year-old female who presents to the trauma bay following a motor vehicle collision. She subsequently undergoes ORIF through a posterior approach. Iatrogenic injury to which nerve in Figure B is most likely with this approach?

     

     

     

     

     

     

    1. A

    2. B

    3. C

    4. D

    5. E

    Corrent answer: 3

     

    Surgical approach, especially posterior and lateral approaches to the humerus, are associated with the highest rate of radial nerve palsy when treating humeral shaft fractures, 11% and 20%, respectively.

     

    Humeral shaft fractures are relatively common injuries with a bimodal distribution. Numerous treatment options exist including coaptation splinting, functional bracing, ORIF through various approaches (anterolateral, lateral, posterior), and the use of an intramedullary nails. Radial nerve palsy in the setting of humeral shaft fractures occur at an overall rate of about 7% and may occur at the time of injury or iatrogenically during surgery. Regarding iatrogenic injuries, only the approach used to fix the fracture has been associated with increased risk of radial nerve injury, with posterior and lateral approaches being most commonly indicted. The increased soft tissue dissection and variable anatomy of the radial nerve have been cited as reasons for this increased risk. The radial nerve is also at risk when placing a medial to lateral locking screw through an IMN, however, this risk is lower than that seen with ORIF via lateral or posterior approaches.

     

    Zhao et al review surgical interventions to treat humeral shaft fractures. They report that ORIF was associated with a higher incidence of radial nerve palsy when comparted to minimally invasive (MIPO) techniques or the use of intramedullary nails. They conclude that, when possible, MIPO techniques should be used given the lower rate of nerve injury.

     

    Classen et al review factors associated with radial nerve palsies following the treatment of diaphyseal humeral shaft fractures. They report an overall incidence of 7% with approach being the factor most closely related to nerve injury. They conclude that the highest risk was with the lateral approach (1 in 5), followed by the posterior approach (1 in 9). The anterolateral approach had the lowest rate of injury (1 in 23).

     

    Figure A is an AP radiograph demonstrating a diaphyseal humeral shaft fracture. Illustration A demonstrates the labeled brachial plexus

     

    Incorrect Answers:

    Answers 1, 2, 4, & 5: The most common iatrogenically injured nerve during the posterior approach to the humerus is the radial nerve.

     

     

     

     

     

    OrthoCash 2020

     

  67. Figures A is the CT angiogram of a 22-year-old male (Patient A) who presents to the trauma bay after sustaining a low-velocity gunshot injury to the femur. Compared to Figure B, which is a different patient (Patient B) who also sustained a low velocity gunshot wound, the associated injury seen in Patient A places him at increased risk for which of the following?

     

     

     

     

     

     

    1. Hip disarticulation

    2. Infection

    3. Malunion

    4. Hardware failure

    5. Neuropraxia

    Corrent answer: 2

     

    This patient sustained a low-velocity gunshot wound to the femur with an associated vascular injury. The presence of a vascular injury has been shown to increase the risk of infection.

     

    A low-velocity GSW is defined as a muzzle velocity <350 meters per second or

    <2,000 feet per second. Recent studies have demonstrated an increased risk of local infection when vascular injury and fracture occurred together.

    Therefore, these patients should receive antibiotics. Conversely, retained bullet fragments have not been shown to increase the risk of infection.

     

    Berg et al. examined the epidemiology, incidence, and distribution of firearm-related extremity trauma and the relationship between injury pattern and local or systemic complications. They report that an isolated fracture increased the risk of compartment syndrome, whereas vascular injury alone increased the risk of compartment syndrome and deep venous thrombosis. Fracture and vascular injury together also increased the risk of wound infection. They conclude that gunshot-related fracture increases the risk of vascular and nerve injury and that vascular injury, with or without fracture, is the biggest predictor of local complications.

     

    Nguyen et al. retrospectively reviewed the rates of infection in low-energy gunshot wounds (GSWs) to the extremity. They report that the overall infection rate was 15.7% (22 patients), and the deep infection rate was 3.6% (5 patients). They concluded that infections after low-energy extremity GSWs are infrequent, but soft tissue injuries without fracture treated with a single dose of intravenous antibiotics in the emergency department had a lower rate of infection.

     

    In a separate study, Nguyen et al. retrospectively reviewed infection rates and other complications after intra-articular (IA) gunshot wounds. They report that 3.6% of patients developed deep infections, all had associated vascular injuries. They concluded that the incidence of infection after IA gunshot injuries is low with the routine use of antibiotic prophylaxis and patients with vascular injury deserve special attention, as they are at higher risk of infection.

     

    Watters et al. retrospectively reviewed the use of surgical débridement in the management of low-velocity gunshot injuries (GSIs) resulting in pelvis fractures. They found a deep infection related to their pelvis GSIs occurred in 7% and 6% among patients who underwent surgical débridement and those who did not, respectively. They concluded that there was no increased incidence of infection in the absence of aggressive surgical débridement of pelvis GSI wounds. The study also demonstrated that retained bullets and

    bullet fragments did not increase the risk of infection, even after penetrating the GI tract organs.

     

    Figure A is a CT angiogram demonstrating a right femur fracture with a vascular injury. Figure B is a CT angiogram demonstrating a right femur fracture without a vascular injury.

     

    Incorrect Answers:

    Answer 1: Due to the location of the injury, Patient A may be at increased risk for amputation, but is not at increased risk for hip disarticulation

    Answer 3: The presence of a vascular injury has not been shown to increase the risk of malunion

    Answer 4: The presence of vascular injury has not been shown to increase the risk of hardware failure

    Answer 5: Nerve damage, not vascular injury, would increase the risk of neuropraxia

     

     

     

    OrthoCash 2020

     

  68. Figure A is the radiograph of a 24-year-old male who presents to the trauma bay following a motor vehicle accident. After initial evaluation the patient begins to become hemodynamically unstable and a pelvic binder is placed. The binder should be centered over what anatomic structure?

     

     

     

     

    1. Anterior superior iliac spine (ASIS)

    2. Lesser trochanter

    3. Greater trochanter

    4. Superior aspect of the iliac crest

    5. Umbilicis

     

    Corrent answer: 3

     

    Figure A demonstrates an APC 3 pelvic ring injury. In the setting of hemodynamic instability, they should be treated with a pelvic binder centered over the greater trochanters.

     

    APC pelvic ring injuries are highly associated with intrapelvic hemorrhage, most commonly from bridging veins. The pelvis can hold a significant amount of blood; thus, the purpose of the pelvic binder is to decrease the total intrapelvic volume. It is crucial that the binder be placed in the proper position at the level of the greater trochanter. Alternatively, a sheet can be applied to the pelvis if a binder is not available.

     

    Prasarn et al review the difference between external fixation and pelvic orthotic devices in a cadaveric study. They report that with logrolling the patient and with elevating the head of the bed the orthotic device placed at the level of the greater trochanters provided improved stability in all planes, however, this did not reach statistical significance. They conclude that there were no significant differences in the stability conferred by an external fixator and a binder for unstable pelvic ring injuries.

     

    Beltran et al review resuscitation and the treatment of shock. They report that placement of a pelvic binder over the greater trochanters serves as an immediate maneuver capable of reducing pelvic volume and stabilizing the pelvis, ultimately aiding in clot formation. They conclude that sheets may also be used and are widely available and can be cut for vascular access to allow for emergent laparotomy.

     

    Prasarn et al compare circumferential sheeting and the use of pelvic binders for unstable pelvic ring injuries. They report no difference in the motion of the hemipelvis during application, logroll, or elevating the head of the bed. They conclude that sheets are more readily available, cost less and are more versatile than pelvic binders.

     

    Incorrect Answers:

    Answers 1,2,4, & 5: For maximal volume reduction and stability, pelvic binders and circumferential sheets should be placed over the level of the greater trochanter.

     

     

    OrthoCash 2020

     

  69. A 54-year-old female sustains a closed comminuted, intraarticular distal radius fracture and a small ulnar styloid tip fracture. She works as a housekeeper and is otherwise healthy. You are considering external fixation (EF) with supplemental K-wire fixation of her articular surface versus volar locking plate (VLP) application. Which of the following considerations is true?

    1. Regardless of method of radius fixation, ulnar styloid tip fractures should be fixed to preserve DRUJ stability

    2. Placing a VLP proximal to the watershed line increases the likelihood for flexor tendon injury

    3. Wound complications are higher for the VLP

    4. Both methods of fixation will yield satisfactory results

    5. Pillar pain is likely in both groups Corrent answer: 4

    Both external fixation (EF) and volar locking plate (VLP) fixation produce successful results in the treatment of distal radius fractures.

     

    Indications for operative intervention on distal radius fractures generally include younger patients with dorsal angulation, displaced intra-articular fragments, radial shortening, intra-articular shear patterns, or die punch fractures of the lunate facet. EF can effectively reduce and stabilized distal radius fractures although the VLP has become more prevalent in recent times, as evident in the 13-fold increase in surgical fixation of distal radius fractures from 1998-2008. Despite this paradigm shift, external fixation remains an effective treatment for distal radius fractures.

     

    Williksen et al. randomized 111 unstable distal radius fractures to VLP or EF. At 5-year follow-up, the authors found no difference in DASH scores. However, for comminuted intra-articular fractures, VLP demonstrated mildly superior supination, flexion, grip strength, Mayo wrist score, and less ulnar shortening. The authors additionally noted that among the VLP cohort, 21% required HWR for surgical complications. They concluded that both methods of fixation achieved satisfactory outcomes at 5 years.

     

    Agee reviewed multiplanar ligamentotaxis in the reduction and stabilization of distal radius fractures. Ligamentotaxis refers to the tension applied across a fracture by the surrounding soft tissues. Dr. Agee contends that the use of an EF allows for adjustments in many planes which in turn helps restore anatomic alignment until the distal radius fracture heals.

    Egol et al. randomized 120 wrist fractures that received EF for fracture stabilization into three groups for pin site care: weekly dry dressings, daily pin site care with hydrogen peroxide, and chlorhexidine discs around the pins. The fixators remained in place for 5.9 weeks and 19% had a pin tract complication. There were no differences between the three groups in terms of the prevalence of pin-site complications. However, increasing the age of the patient was correlated to pin-track complications.

     

    Incorrect Answers:

    Answer 1: Ulnar styloid tip fixation is not necessary and an ulnar styloid tip non-union does not have a significant impact on clinical outcomes.

    Answer 2: Placement of the VLP DISTAL to the watershed line increases the risk for flexor tendon injury.

    Answer 3: Wound complications such as pin tract infection are significantly higher in the EF cohort.

    Answer 5: Pillar pain is frequent in carpal tunnel release, but is not commonly reported in distal radius fracture fixation.

     

     

     

    OrthoCash 2020

     

  70. An 8-year-old boy is involved in an ATV crash. Xrays were obtained in the trauma bay after being airlifted to your facility, shown in Figures A. On the day of presentation, you perform the treatment shown in the figure B. On postoperative day 1, the patient states his pain is controlled, however, you find that his calf is tense, his foot is cool and has diminished pulses compared to the contralateral extremity. What is the best next step of the options below?

     

     

     

     

     

    1. Remove your fixation and perform repeat reduction

    2. Measure leg compartment pressures

    3. Request vascular consultation

    4. Measure ankle-brachial index (ABI)

    5. Duplex ultrasound evaluation Corrent answer: 2

    This patient is presenting with symptoms concerning for compartment syndrome without significant pain. The next best step in evaluation would be to perform compartment pressure measurements.

     

    Compartment syndrome is the process where osseofascial compartment pressures rise to levels that decrease perfusion to the structures within. This is an orthopedic emergency, requiring opening the compartment via fasciotomy to relieve the pressure within. Delay in treatment can result in irreversible muscle and nerve damage. Compartment syndrome is a clinical diagnosis, and treatment should not be delayed if your index of suspicion is high. Diagnosis can be more difficult in children due to inability to verbalize symptoms.

    However, compartment pressures can be measured if the clinical picture is uncertain, such as when patients sedated or otherwise have altered mental status. Typically a difference of <30 mmHg between the compartment pressure and diastolic blood pressure (prior to induction of anesthesia) is used as an indication for fasciotomy.

     

    Ojike et al. systematic reviewed foot compartment syndrome. They had 39 cases of foot compartment syndrome that underwent fasciotomy. Crush injury was the most common mechanism. Neurologic deficit was present in 52% of cases, and 65% required split-thickness skin grafting for closure. 39% of patient had residual pain and stiffness, and only 10% could return to pre-injury activity state after fasciotomy.

     

    Badhe et al. present a case series of 4 patients who developed compartment syndrome without significant pain. One patient developed compartment syndrome in an uninjured leg, another after intramedullary nailing of a tibia fracture, a third after a tibial plateau fracture, and the last after a comminuted metaphyseal proximal tibia fracture. All four patients had good pain control but had calf swelling, paresthesias, or diminished pulses. The diagnosis was confirmed with compartment pressure measurements. They conclude the importance of pressure measurements when clinical suspicion is high despite appropriate pain levels.

     

    Flynn et al. retrospectively reviewed 43 pediatric patients with compartment syndrome of the leg. 83% had fractures of the tibia and fibula, and time to

    diagnosis of compartment syndrome was 18.2 hours after injury. Presenting symptoms included increasing pain (93%), paresthesias (26%), and absent or diminished pulses (14%). No infections occurred after fasciotomy, and 95% had no sequellae at the final follow up. The two patients with fair outcomes both had fasciotomies performed >48 hours after injury. They conclude by emphasizing the importance of extended vigilance in pediatric patients for the development of compartment syndrome.

     

    Figures A shows an AP radiograph of a tibial shaft fracture in a pediatric patient. Figure B shows a diagram of flexible nailing of a tibia.

     

    Incorrect answers:

    Answer 1: This would not aid in the diagnosis or treatment of compartment syndrome.

    Answer 3: While vascular compromise can be a late manifestation of the compartment syndrome, the treatment is fasciotomy to re-establish blood flow. Answer 4: An ABI would likely show an abnormality, however, this would not be the appropriate study.

    Answer 5: This is the appropriate workup if you were concerned about deep venous thrombosis.

     

     

     

    OrthoCash 2020

     

  71. A 72-year-old female presents to your office with a 12-month old painful nonunion of a 2-part (surgical neck) proximal humerus fracture which was managed non-operatively. Prior to her injury, she denied shoulder pain and had excellent range of motion. Since the injury, she has had persistent debilitating pain and dysfunction with motion above the shoulder level. X-Rays show good bone stock, no significant shoulder arthritis or avascular necrosis, and well-positioned tuberosities. Which of the following is the most optimal treatment for the patient?

    1. Closed reduction and percutaneous pinning

    2. Shoulder hemiarthroplasty

    3. Open bone grafting

    4. Open reduction and internal fixation (ORIF) with or without bone grafting

    5. Reverse total shoulder arthroplasty (rTSA) with lattisimus dorsi transfer to assist with internal rotation

    Corrent answer: 4

     

    A 2-part proximal humerus non-union in a patient with good bone stock without evidence of arthritis or osteonecrosis can be effectively managed with

    ORIF with locked plating with or without bone grafting.

     

    Treatment of a chronic nonunion of the proximal humerus in the elderly should be treated with fixation when possible. Critical attention should be paid to correct all deformities: tuberosity positioning, articular surface realignment, soft tissue balancing, rotator cuff repair (when needed), and treatment of soft tissue contractures. Attempts at arthroplasty are generally recommended only when there is significant osteopenia or avascular necrosis, if the tuberosities have resorbed, if the rotator cuff has a pre-existing tear, or if other findings are present that would limit the success rate of fixation.

     

    Quadlbauer et al. evaluated the results of ORIF with locked plating without bone grafting for 9 patients with proximal humerus non-unions with a mean 31-month follow-up. They noted that all 9 patients went on to achieve bony union with improvement in ROM in all planes except fo adduction. They concluded that ORIF with locked plating without bone grafting is a reasonable and safe option for treating proximal humerus nonunion with high union rates and minimal risk of complications

     

    Cadet et al. in a JAAOS review article discussed the various management options for proximal humerus non-unions. The authors noted that several recent series did demonstrate union rates >90% for patients treated with ORIF using locking plates and bone graft. They discuss that while reverse shoulder arthroplasty (rTSA) has also shown excellent clinical results, it should be reserved for patients with nonviable humeral heads, severe humeral osteoporosis or deficient rotator cuffs.

     

    Incorrect Answers:

    Answer 1: There is no role for closed reduction in the setting of a non-union Answer 2: While rTSA has shown great results with, similar results have not recently been shown for shoulder hemiarthroplasty

    Answer 3: Open bone grafting by itself would not be sufficient for the management of a proximal humerus non-union. The fracture will need stabilizing with locking plate fixation

    Answer 5: While rTSA is an effective treatment option for proximal humerus non-unions, the addition of a lattisimus dorsi transfer would assist with EXTERNAL rotation, not INTERNAL rotation.

     

     

     

    OrthoCash 2020

     

  72. A 68-year-old woman undergoes a hemiarthroplasty for a proximal humerus fracture through a deltopectoral approach. Care

    should be taken with which of the following activities in the immediate postoperative period due to concerns about lesser tuberosity fixation?

    1. Passive forward flexion of the shoulder in the scapular plane

    2. Passive internal rotation of the shoulder to the plane of the body

    3. Active abduction of the shoulder past 45 degrees

    4. Passive external rotation of the shoulder

    5. Active extension of the shoulder Corrent answer: 4

    Due to extensive pull on the lesser tuberosity via the subscapularis, care should be taken with increased levels of passive external rotation in the immediate post-operative period.

     

    Proximal humerus injuries resulting in head-splitting fractures, avascular necrosis, or nonunion can be treated reliably with arthroplasty. During reconstruction with arthroplasty, the tuberosities need to be anatomically fixed in order to allow for a proper range of motion post-operatively. The subscapularis tendon inserts on the lesser tuberosity and is the deforming force when placed under tension during external rotation and during active internal rotation. As such, post-operative passive external rotation and active internal rotation are both limited initially until the lesser tuberosity has healed in its anatomic location.

     

    Frankle et al. performed a biomechanical study to determine the importance of anatomic fixation of tuberosities after hemiarthroplasty for proximal humerus fractures. The authors noted that non-anatomic tuberosity reconstruction led to significant impairment in external rotation kinematics and an 8-fold increase in torque requirements whereas anatomic reconstruction produced indistinguishable results from normal shoulder controls. The authors concluded that the rotational alignment of tuberosities during reconstruction is critical in order to allow patients to perform an adequate range of motion postoperatively.

     

    Frankle and Mighell performed a review of the techniques of tuberosity fixation for shoulder hemiarthroplasty for the treatment of proximal humerus fractures. The authors describe that variable geometry and known anatomic relationships can facilitate tuberosity reconstruction. This requires a re-producible technique, an instrumentation system, and a prosthesis that can maximize tuberosity reconstruction.

     

    Incorrect Answer:

    Answers 1, 2, 3 and 5: Passive external rotation and active internal rotation

    both places stress on the lesser tuberosity due to the insertion of the subscapularis tendon and are limited in the immediate post-operative setting following hemiarthroplasty.

     

     

     

    OrthoCash 2020

     

  73. A 24-year-old man was involved in a MVC and sustained the injury shown in Figure A. A closed reduction is performed and a post-reduction CT scan is obtained. Which of the following correctly pairs the CT with the most appropriate management?

     

     

     

     

     

     

     

     

     

     

     

     

    1. Figure B; Protected weightbearing

    2. Figure B; Examination under anesthesia (EUA) to assess hip stability

    3. Figure C; Protected weightbearing

    4. Figure C; Examination under anesthesia (EUA) to assess hip stability

    5. Figure D; Protected weightbearing Corrent answer: 4

    This patient sustained a posterior hip dislocation. Of the answer choices listed, the fracture pattern seen in Figure C would be appropriately managed with EUA to assess hip stability.

    Posterior wall fractures are the most common type of acetabular fracture. Management of posterior wall fractures is dictated by the extent of posterior wall involvement, with those involving >40-50% best treated with ORIF. Management of fractures involving 20-40% of the posterior wall, on the other hand, should be based on hip stability. As suggested by Tornetta, assessment of hip instability with dynamic stress views is helpful to determine which posterior wall fractures are unstable and therefore require open reduction and internal fixation. Protected weightbearing may be employed if the hip is stable; however, hip stability must be determined first.

     

    Tornetta performed a study to assess the use of dynamic stress views in the non-operative management of acetabular fractures. He found that the use of dynamic stress views in determining the need for ORIF was effective, leading to good or excellent results in 91% of cases. He concluded that dynamic stress views can identify subtle instability in patients who would normally be considered for non-operative treatment.

     

    Moed et al. performed a retrospective study to evaluate the role of CT in predicting hip stability in posterior wall fractures of the acetabulum. They analyzed 3 methods of calculating posterior wall fracture fragment size on CT and compared it to findings on EUA. They found that the alternative method (which measures fragment size at the level of the largest posterior wall deficit) was more accurate than the Calkins method (which measures the smallest amount of intact acetabular arc) and the Keith method (which measures fragment size at the level of the fovea). However, given the low risk of EUA and the inherent problems making CT measurements, they concluded that dynamic fluoroscopic stress testing under general anesthesia should be the preferred method for the determination of hip stability status after posterior wall fractures of the acetabulum.

     

    Calkins et al. performed a study to assess the efficacy of CT in predicting hip stability based on measurements of the percentage of remaining posterior acetabulum. They found that all hips with <34% of the remaining posterior acetabulum were unstable, while those with >55% were stable. They concluded that a simple linear measurement of the remaining posterior acetabulum on CT can be done easily by a physician and can provide a means of predicting hip stability for individual patients.

     

    Figure A is an AP radiograph of the right hip demonstrating a posterior hip dislocation. Figure B is an axial CT of the right hip demonstrating a posterior wall fracture with intraarticular fragments. Figure C is an axial CT of the right hip demonstrating a posterior wall fracture involving > 20% of the posterior

    wall. Figure D is an axial CT of the right hip demonstrating a posterior wall fracture involving > 40% of the posterior wall.

     

    Incorrect Answers:

    Answer 1: Protected weightbearing would be inappropriate for a posterior wall fracture with intraarticular fragments.

    Answer 2: EUA would be inappropriate for a posterior wall fracture with intraarticular fragments.

    Answer 3: Posterior wall fractures involving 20-40% of the posterior wall may be treated with protected weightbearing; however, EUA to assess for instability is recommended prior to making the final decision to proceed with nonoperative treatment.

    Answer 5: Open reduction internal fixation is indicated in unstable fractures such as posterior wall fractures involving >40-50% of the posterior wall.

     

     

     

    OrthoCash 2020

     

  74. A 32-year-old female sustained a closed clavicle fracture after a fall as shown in Figures A and B. After a long discussion of the risks and benefits the patient elects to undergo nonoperative management. When discussing nonunion, which of the following is the best estimate for risk of nonunion with nonoperative treatment?

     

     

     

     

     

     

    1. 1%

    2. 5%

    3. 15%

    4. 30%

    5. 50%

     

    Corrent answer: 3

     

    The best estimate for nonunion in closed, displaced, middle-third clavicle fractures is 15%.

     

    Early evidence estimated nonunion rates of 1% in nonoperatively treated displaced middle-third clavicle fractures. More recent studies and randomized controlled trials have demonstrated a nonunion rate of roughly 15% in displaced middle third clavicle fractures. This along with improved functional outcomes and lower complications rates with newer surgical techniques have driven some surgeons and patients to favor operative treatment for closed displaced middle-third clavicle fractures.

     

    Kim et al. provide a thorough review of clavicle fracture management. They

    state modern studies of displaced middle-third clavicle fractures treated nonoperatively have an estimated malunion rate between 18-35%. They state other studies have associated symptomatic malunion with shortening and treatment often includes an osteotomy to reestablish the length of the clavicle.

     

    The Canadian Orthopaedic Trauma Society randomized 132 patients to operative or nonoperative treatment for displaced middle-third clavicle fractures and followed them for 1 year. They found improved Constant and Dash scores in the operative fixation group at all time points. In the nonoperative group, they found a nonunion rate of ~14% and symptomatic malunion rate of ~18%. They concluded operative fixation of displaced middle-third clavicle fractures leads to improved functional outcomes and a lower rate of nonunion and symptomatic malunion when compared to nonoperative treatment at 1 year follow up.

     

    McKee et al. looked at objective functional outcomes in 32 patients who underwent nonoperative treatment for a displaced middle-third clavicle fracture. They found substantial loss of strength of the affected shoulder when measured objectively as well as a mean Constant score of 71 points and mean DASH of 24.6 points. They concluded that surgeon based methods for measuring strength and endurance are not sensitive enough to capture the differences they found and that these differences likely are responsible for lower patient-based outcomes measures.

     

    Figures A and B show a displaced middle-third clavicle fracture with little to no comminution.

     

    Incorrect Answers:

    Answer 1: 1% was felt to be the rate of nonunion prior to more recent studies Answer 2: 5% is not the best estimate of nonunion for displaced middle-third clavicle fractures.

    Answer 4: 30% is not the best estimate of nonunion for displaced middle-third clavicle fractures.

    Answer 5: 50% is not the best estimate of nonunion for displaced middle-third clavicle fractures.

     

     

     

    OrthoCash 2020

     

  75. A 44-year-old male fell onto his arm and presents to the emergency department with elbow pain, swelling, and inability to use the right arm. The ED staff place the patient in a posterior mold splint and call after radiographs are obtained. The current images are shown in Figures A and B. What is the next best step in management?

     

     

     

     

     

    1. CT scan of the elbow

    2. Examination of the elbow under anesthesia

    3. Obtain traction views

    4. Continued immobilization and outpatient follow-up in three weeks

    5. Immediate ORIF of coronoid, radial head, and repair of LCL Corrent answer: 1

    The patient is presenting with an anteromedial facet fracture of the coronoid due to a posteromedial rotatory force. A CT of the elbow should be performed to asses the extent of articular involvement, degree of comminution, and the size of the coronoid fragment.

     

    The coronoid process is one of the main constraints that provide ulnohumeral joint stability. The coronoid process and the radial head provide a buttress against the posterior displacement of the elbow joint. The anteromedial facet of the coronoid process, also known as the sublime tubercle, is the insertion of the anteromedial bundle of the ulnar collateral ligament, which provides valgus stability of the elbow. Fracture of the coronoid process can be either an isolated finding following elbow dislocation or part of a more complex fracture-dislocation. This patient had a fracture to the of the anteromedial portion of the coronoid, therefore the medial side of the elbow is unsupported to varus stresses. As a result, the joint line will narrow from lateral to medial secondary to the medial collapse of the joint. On a lateral, the crescent sign, which represents the depressed anteromedial facet of the coronoid, may be seen.

     

    Steinmann reported that with an anteromedial coronoid fracture, the anteroposterior radiograph of the elbow will demonstrate progressive narrowing of the joint space from lateral to medial between the medial trochlea and the coronoid process. On a lateral radiograph, a coronoid fracture fragment may appear small and may be confused with a radial head fracture. A CT scan of the elbow is recommended in all cases of suspected coronoid fractures because such a fracture is readily seen with this imaging modality.

     

    Doornberg and Ring reported that coronoid fractures are associated with specific patterns of traumatic elbow instability. They concluded that large coronoid fractures were associated with anterior and posterior olecranon fracture-dislocations while small transverse fractures were associated with terrible triad injuries. Finally, anteromedial facet fractures were associated with varus posteromedial rotational instability.

     

    Ring and Doornberg wrote a surgical technique of repairing the anteromedial facet fracture. They report that fractures of the anteromedial facet of the coronoid may be treated with a plate, a screw, or sutures. They conclude that

    the secure fixation of this fragment usually restores good elbow function.

     

    Figures A and B are the AP and lateral radiographs of the right elbow with a fracture of the anteromedial facet of the coronoid.

     

    Incorrect Answers:

    Answer 2: Though an examination of the elbow would likely reveal instability, given the large fracture evident on radiographs, this would not provide further diagnostic information.

    Answer 3: Traction view radiographs are helpful in comminuted distal humerus fractures. In this patient, traction views would provide little diagnostic benefit. Answer 4: Prolonged immobilization of elbow injuries typically results in permanent elbow stiffness. Surgical fixation of the fracture would be preferable to permit early rehab after a CT assessment to determine the extent of the injury.

    Answer 5: This is the treatment for terrible triad injury of the elbow. The radial head does not appear to be affected in the current radiographs.

     

     

     

    OrthoCash 2020

     

  76. A 58-year-old male active smoker falls while working at his barn and sustains the closed injury depicted in Figures A and B. His neurovascular examination is unremarkable. The decision is made to perform open reduction internal fixation of his proximal humerus. Which of the following characteristics is MOST predictive of humeral head ischemia?

     

     

     

     

     

     

    1. Medial hinge disruption

    2. Calcar length of 4mm

    3. Patient age

    4. Lateral hinge disruption

    5. Number of fracture parts Corrent answer: 2

    This patient has sustained a comminuted 4-part proximal humerus fracture. Of the above fracture characteristics, the most predictive of humeral head

    ischemia is calcar length of 4mm.

     

    In complex intra-articular proximal humerus fractures, perfusion of the humeral head is an important consideration when deciding on treatment. Recent evidence has demonstrated that the posterior humeral circumflex artery is likely the main blood supply to the humeral head, and fracture displacement can lead to disruption of this blood supply along with the anterior humeral circumflex artery, causing humeral head ischemia. Several studies have demonstrated that residual perfusion of the humeral head relies predominantly on the medial and posteromedial metaphyseal extension (i.e. calcar length) that might remain attached to the head, with other protective factors including the integrity of the medial hinge and basic fracture patterns .

     

    Hertel et al. investigated predictors of fracture-induced humeral head ischemia at the time of surgery for articular proximal humerus fractures. They reported that the highest predictors of humeral head ischemia, from most accurate to least accurate, were calcar length <8mm, disruption of the medial hinge, fracture pattern, displacement of the humeral head >45 degrees, displacement of the tuberosities >10 mm, glenohumeral fracture-dislocation and head-split fractures. They concluded that the most relevant predictors of ischemia were the length of the posteromedial calcar, the integrity of the medial hinge, and the basic fracture type.

     

    Nho et al. reviewed innovations in the management of displaced proximal humerus fractures. They reported that treatment should be dictated by careful scrutiny of vascular status, bone quality, fracture pattern, degree of comminution, as well as patient factors, such as age and activity level. They concluded that Fractures with AO type C pattern, metaphyseal extension <8 mm, or medial hinge displacement >2 mm are associated with high probability of humeral head ischemia and likely are best treated with arthroplasty.

     

    Figure A demonstrates a plain radiograph of a 4-part proximal humerus fracture. Figure B is a CT 3D-reconstruction of the 4-part proximal humerus fracture. Illustration A highlights the vascular anatomy of the proximal humerus. Illustration B depicts the differences in posteromedial calcar lengths in proximal humerus fractures. Illustration C highlights the difference between an intact and disrupted medial hinge.

     

    Incorrect Answers:

    Answer 1 and 4: While medial (not lateral) hinge disruption may predict humeral head ischemia, a short calcar length (i.e. 4 mm) is more predictive. Answer 3: Patient age is not as strong of a factor in predicting humeral head ischemia like calcar length.

    Answer 5: The number of fracture fragment may predict humeral head ischemia, but it is not as strong and relevant of a predictor like calcar length.

     

     

     

     

     

     

     

     

     

     

     

     

    OrthoCash 2020

     

  77. You are called to evaluate a 32-year-old man with knee pain after being struck as a pedestrian. You recognize the fracture as a Schatzker IV tibial plateau fracture. Which of the following statements is most accurate?

    1. A fracture-dislocation must be suspected as the femur follows the displaced lateral tibial plateau

    2. This fracture may often be seen with medial meniscal and anterior cruciate

      ligament injuries

    3. Open reduction internal fixation with a lateral plating is the construct of choice

    4. Open reduction internal fixation with bicondylar plating is the construct of choice

    5. Vascular injuries are rare with this type of fracture Corrent answer: 2

    Of the above descriptions regarding the Schatzker IV tibial plateau fracture, the most accurate is that it may often be seen with medial meniscal and anterior cruciate ligament injuries.

     

    While the lateral plateau is smaller and convex, the medial plateau is larger and concave, resulting in an eccentric load distribution in which the medial plateau bears ~ 60% of the knee's load. The relative osseous strength of the medial plateau, the valgus anatomic axis of the lower extremity, and the susceptibility of the leg to a medially directed force all result in the increased prevalence of to lateral-sided injuries (Schatzker I and II) in low-energy fractures. High-energy plateau injuries result in increased less predictable fracture patterns that are often comminuted and involve the medial plateau (Schatzker IV through VI) and may be associated with medial meniscus, ACL, and vascular injuries.

     

    Bennett et al. reviewed tibial plateau fractures and associated soft tissue injuries. They reported a 56% rate of associated soft tissue injuries in their series, with the most common being medial, lateral collateral ligaments, and menisci. They observed that Schatzker IV and VI plateau fractures were associated with the highest rate of soft tissue injuries. They recommended pre-and post-fixation stress testing to diagnose collateral and cruciate ligament injuries

     

    Berkson et al. reviewed high energy tibial plateau fractures. They reported that high-energy injuries can lead to comminuted fractures with significant osseous, soft-tissue, and neurovascular injury, whereas lower energy injuries result in unilateral depression type fractures. They emphasized that treatment be directed at safeguarding tissue vascularity and restoring joint congruity and the mechanical axis of the limb.

     

    Illustrations A and B demonstrate a Schatzker IV tibial plateau fracture. Illustration C depicts the Schatzker classification of tibial plateau fractures. Schatzker I is a lateral split fracture, Schatzker II is a lateral split-depression fracture, Schatzker III is a lateral pure depression fracture, Schatzker IV is a medial plateau fracture, Schatzker V is a bicondylar fracture, and Schatzker VI

    is a metaphyseal-diaphyseal dissociation.

     

    Incorrect Answers:

    Answer 1: A fracture-dislocation must be suspected as the femur follows the displaced medial (not lateral) tibial plateau

    Answer 3 and 4: ORIF with a medial buttress plate is the definitive construct often used for Schatzker VI fractures.

    Answer 5: Vascular injuries are commonly associated with Schatzker IV plateau fractures.

     

     

     

     

     

     

     

     

    OrthoCash 2020

     

  78. A 36-year-old female sustains the closed injury shown in Figure A as the result of a high-speed motorcycle collision. She undergoes immediate closed reduction and placement of a knee-spanning external fixator with plans for definitive fixation in 2 weeks. Which of the following is an expected finding in this patient as a result of the placement of a temporizing external fixator?

     

     

     

     

    1. Increased rate of compartment syndrome during hospitalization

    2. Increased transient leg compartment pressures during external fixator placement

    3. Increased rate of ligamentous knee injury

    4. Increased risk of deep hardware infection after 2nd stage surgery

    5. Increased risk of malunion after 2nd stage surgery Corrent answer: 2

    Application of knee-spanning external fixation as a temporary measure for stabilization of high-energy proximal tibial fractures and dislocations may result in transient elevation of intracompartmental pressures of the leg.

     

    The use of a staged protocol for the treatment of high-energy tibial plateau

    fractures has been shown to decrease soft tissue complications. These high energy injuries are best treated with placement of a knee-spanning external fixator which allows ligamentotaxis to maintain overall length. The staged protocol generally consists of closed reduction and placement of a knee-spanning external fixator. Although some controversy exists, the pins are generally not placed within the site of injury or planned area of the plate, in order to minimize infectious risk. The limb is elevated, and ice can be used generously to allow for the resolution of edema. Once the swelling improves, and skin wrinkles are seen, the risk of wound healing complications and infection is decreased, and the surgeon can then proceed with surgery. During the placement of an external fixator, a transient elevation of compartment pressures is expected (DeltaP may fall below the threshold of 30 mm Hg), but this does not appear to lead to the development compartment syndrome.

     

    Egol et al. (2008) reviewed the effect of spanning external fixation in regards to compartment syndrome in 25 patients. They found that 9/22 (41%) had a transient DeltaP < 30 mm Hg at some point during surgery. No patient had a DeltaP < 30 mm Hg sustained through the conclusion of the procedure, and no compartments were released in any of these patients. None of the patients in the study developed compartment syndrome after surgery, and no sequelae of compartment syndrome were noted at minimum 6-month follow-up.

     

    Egol et al. (2005) reported on a multicenter study of 62 patients that underwent staged treatment of high-energy tibial plateau fractures. They found a significant decrease in wound complications and infection (5% overall) with this treatment method but noted a concern with temporary stiffness due to the spanning external fixator.

     

    Figure A shows a tibial plateau fracture involving both condyles with dissociation from the tibial shaft, indicating a severe, high-energy injury. This would be classified as a Schatzker VI tibial plateau fracture.

     

    Incorrect Answers:

    Answer 1: Application of a knee-spanning external fixator does not increase the rate of compartment syndrome above baseline levels for this injury.

    Answer 3: There are no reports of an increased rate of ligamentous knee injuries during the placement of a knee-spanning external fixator.

    Answer 4: There are no reports of increased infection rates as a result of temporizing external fixator placement. On the contrary, most studies confirm decrease rates of wound complications with a staged protocol.

    Answer 5: There are no reports of increased malunion rate as a result of staged fixation of tibial plateau fractures. However, DEFINITIVE fixation with

    an external fixator has shown very high malunion rates in tibial plateau fractures.

     

     

     

    OrthoCash 2020

     

  79. A 72-year-old female with a history of osteoporosis treated with bisphosphonates presents to the ED with bilateral thigh pain after falling backward at home. On examination, there is gross external rotation of bilateral lower extremities with shortening. There are no open skin lesions. Radiographs are shown in figures A and B. What is the total expected blood loss of these injuries?

     

     

     

     

     

    1. 500-1000 cc

    2. 1000-1500 cc

    3. 1500-2000 cc

    4. 2000-3000 cc

    5. 3000-4000 cc

     

    Corrent answer: 4

     

    Average blood loss in closed femur fracture averages between 1000 to 1500

    cc. With bilateral closed femoral shaft fractures, the expected blood loss would be between 2000 and 3000 cc.

     

    The estimated blood loss for closed fractures of the femur and tibia are 1000-1500 and 500-1000 cc, respectively. Blood loss can double in open fractures. Similarly, bilateral closed femoral shaft fractures can have double the blood loss of a single closed femoral shaft fracture. In the presence of exsanguinating external hemorrhage, control of hemorrhage should take precedence over the ABCDE primary survey.

     

    Lieurance et al. retrospectively reviewed 53 patients with isolated femur fractures. They found that 21 patients required transfusion of an average of

    2.5 units PRBCs. The estimated blood loss was 1276cc and not affected by the fracture pattern or high/low energy pattern. They concluded that preoperative hemorrhage contributed to transfusion needs more than intraoperative blood loss.

     

    Lee and Porter reviewed the prehospital management of lower limb fractures. They recommend a stepwise control of bleeding: direct pressure, elevation, wound packing, windlass technique (place a pen under a circumferential knot and rotate the pen until tight), indirect pressure (traction splint), tourniquet. In the presence of compressible/controllable bleeding, they recommend resuscitation to normal physiological parameters. In the presence of non-controllable bleeding, they recommend hypotensive resuscitation (SBP 80mmHg or to restore the radial pulse).

     

    Figures A and B are the AP radiographs of the left and right atypical femur fractures with characteristic cortical thickening, transverse fracture pattern with medial beaking, and lateral stress reaction at the site of the fracture.

     

    Incorrect Answers:

    Answers 1, 2, 3, 5: Average blood loss for a closed femoral shaft fracture is 1000-1500 cc, with bilateral fractures having double the blood loss. The other answer choices do not reflect this.

     

     

    OrthoCash 2020

     

  80. Lateral malleolus fractures can be treated with a variety of techniques, including posterior antiglide plating or lateral neutralization plating. What is an advantage of using lateral neutralization plating instead of posterior antiglide plating?

    1. Decreased joint penetration of distal screws

    2. Increased rigidity

    3. Decreased need for delayed hardware removal

    4. Decreased peroneal irritation

    5. Improved distal fixation Corrent answer: 4

    Posterior antiglide plating is a technique that involves placement of a plate on the posterior aspect of the distal fibula, using the plate as a reduction tool and direct buttress against distal fracture fragment displacement.

     

    Schaffer et al showed from a biomechanical standpoint that posterior antiglide plating was superior to lateral neutralization plating for distal fibula fracture fixation.

     

    Weber et al reported a (30/70) 43% rate of plate removal secondary to peroneal discomfort. In addition, peroneal tendon lesions were found in 9 of the 30 patients.

     

     

     

    OrthoCash 2020

     

  81. A 24-year-old female sustains a surgical neck proximal humerus fracture in a motor-vehicle collision. She undergoes open reduction and internal fixation but heals in 45 degrees of varus and has significant limitation of shoulder range of motion despite 9 months of conservative treatments. What is the most appropriate treatment at this time?

    1. Manipulation under anesthesia

    2. Humeral head resurfacing

    3. Shoulder hemiarthroplasty

    4. Revision open reduction internal fixation with osteotomy

    5. Reverse total shoulder arthroplasty Corrent answer: 4

    Malunions of the proximal humerus typically result in significant restrictions in range of motion. This young patient has sustained a proximal humeral malunion, and treatment should include a corrective osteotomy for improved outcomes, as she has failed conservative treatment.

     

    The cited reference by Williams et al as well as the referenced article by Siegel et al explain various techniques in management of proximal humerus malunions. They state that the two primary indications for surgical management of proximal humerus malunion include 1) pain and 2) diminished function resulting from limited range of motion. Because this patient is young, you would attempt revision ORIF/osteotomy as opposed to humeral head arthroplasty.

     

     

     

    OrthoCash 2020

     

  82. A 6-year-old boy with progressive bilateral genu varum undergoes the surgeries shown in Figure A. On postoperative rounds later that day, the patient appears sedated after several doses of pain medication. His toes are pink with brisk capillary refill however, passive motion of his toes causes pain. Among the answer choices listed, what is the best management strategy for this child?

     

     

     

     

    1. Elevate his legs and reevaluate on morning rounds

    2. Adjust his pain medication to accommodate for his increasing pain

    3. Administer a muscle relaxant for leg spasms

    4. Cast removal and measurement of compartment pressures with a standard device

    5. Examine the cast for areas of constriction and reevaluate in the morning Corrent answer: 4

    Intracompartmental pressure measurements should be performed when pain with passive motion of the toes is found in young patients with insufficient clinical data to establish a definitive diagnosis of compartment syndrome. The child in this clinical vignette has Blount’s disease which was treated with bilateral tibial osteotomies, a procedure commonly associated with compartment syndrome.

     

    Pain with passive stretch is the most sensitive clinical sign of elevated compartment pressures prior to the onset of ischemia in compartment syndrome. Pain is difficult to assess in children at baseline, therefore, a high level of suspicion should exist and compartment pressure monitoring should be performed in unreliable patients.

     

    Mubarak et al. reported on a series of 27 patients subjected to intracompartmental pressure monitoring for a clinical suspicion of acute compartment syndrome. The wick catheter technique was employed not only to aid in the diagnosis of compartment syndrome at an early stage but also to indicate the effectiveness of the decompressions when used intraoperatively during fasciotomies.

     

    Matsen et al. reported on 24 children with compartment syndrome following injuries and surgery. The most common etiologies identified were fractures, vascular injuries, and tibial osteotomies. Compartment pressure measurements were helpful in establishing the diagnosis of compartment syndrome in young patients and in those with neurologic or vascular injuries with ambiguous clinical findings.

     

    Figure A reveals an AP radiograph of bilateral knees status post valgus-producing tibial osteotomies and epiphysiolyses of the medial tibial physes in a 6-year-old male with Blount’s disease.

     

    Incorrect Answers:

    Answers 1 and 5 are wrong because immediate action should be taken to establish a definitive diagnosis and treat accordingly when a clinical suspicion of compartment syndrome arises.

    Answers 2 and 3 are incorrect because administering medications that can cause sedation will exacerbate the patient’s present state of drowsiness, confuse the clinical picture, and make the diagnosis of compartment syndrome even more difficult to establish.

     

     

     

    OrthoCash 2020

  83. A 16-year-old male was struck by an automobile while riding his bicycle. He sustained the injuries in Figure A. Which of the following orthopedic injuries is MOST associated with this injury?

     

     

     

     

    1. Brachial plexus injury

    2. Ipsilateral clavicle fracture

    3. Pelvic ring injury

    4. Rib fracture

    5. Spine fracture

     

    Corrent answer: 4

     

    Figure A demonstrates a scapula body fracture. Rib fractures are the most common orthopedic injury associated with these high-energy fractures, with a 52% incidence.

     

    Scapula fractures are associated with high-energy trauma and have a reported mortality rate of 2-5%. Approximately 50% of high-energy scapular fractures involve the body and spine. Most scapular fractures may be treated conservatively with sling immobilization followed by early motion with no expected functional deficits. Indications for operative management of scapular fractures include glenohumeral instability with >25% of glenoid involvement or

    >5mm of articular step-off, excessive medialization of the glenoid, displaced scapular neck or coracoid fractures, and open fractures.

     

    Baldwin et al. retrospectively reviewed 9,543 scapular fractures utilizing the US National Trauma Database. They reported that the most commonly associated fractures were rib fractures (52.9%), followed by fractures of the spine (29.2%), clavicle (25.2%), and pelvis (15.3%). They concluded that lung and head injuries occurred in 47.1% and 39.1% of the cases, respectively.

     

    Brown et al. retrospectively reviewed the association between scapular fractures (SF) and blunt thoracic aortic injury (BTAI). They found that in 35,541 blunt trauma admissions, SF and BTAI occurred in 1.1% and 0.6% of patients, respectively. They noted that most of the patients with SF had associated injuries (99%), but only four patients with SF had BTAI. The most common injuries associated with SF were rib (43%), lower extremity (36%), and upper extremity (33%) fractures. They concluded that SF is uncommon after blunt trauma, but patients with SF almost always have significant associated injuries and indicates a high amount of energy.

     

    Figure A demonstrates a high-energy scapular body fracture which may benefit from surgical intervention

     

    Incorrect Answers:

    Answers 1-3,5: Fractures of the spine (29.2%), clavicle (25.2%), and pelvis (15.3%) are all associated with high-energy scapula fractures, but occur less commonly than rib fractures (52%)

     

     

     

     

     

    OrthoCash 2020

     

  84. Which of the following amputations will lead to the greatest oxygen requirement per meter walked following prosthesis fitting?

    1. Above-knee-amputation (transfemoral)

    2. Below-knee-amputation (transtibial)

    3. Through Knee

    4. Syme

    5. Midfoot

     

    Corrent answer: 1

     

    The general trend is increasing energy requirement for more proximal amputations. Amputation should be performed at the lowest possible level in order to preserve the most function.

     

    Pinzur compared 5 patients with amputations at midfoot, Syme’s, BKA, through knee, and AKA with five controls. Walking speed and cadence decreased while oxygen consumption per meter walked increased with each more proximal amputation.

     

    The only exception is the Syme which was the most energy efficient even though it is more proximal to the midfoot amputation.

     

     

    OrthoCash 2020

     

  85. An otherwise healthy 30-year-old male sustains a left forearm injury as a result of a fall from a ladder. Initial examination in the emergency room reveals a clean 2 centimeter laceration over the volar forearm associated with the radiographs shown in Figures A and B. Treatment should consist of irrigation and debridement of the wound followed by which of the following?

     

     

     

     

    1. Closed reduction and casting of left radius and ulna

    2. Temporary external fixation of the left radius and ulna

    3. Definitive external fixation of the left radius and ulna

    4. Open reduction and internal fixation of the left radius and ulna with delayed skin closure

    5. Open reduction and internal fixation of the left radius and ulna with immediate skin closure

    Corrent answer: 5

     

    The clinical scenario is consistent with an open fractures of the distal radial and ulnar shafts. Literature shows that definitive plating of an open forearm fracture followed by primary closure of the wound is acceptable treatment at the time of injury.

    Chapman et al performed a retrospective review of 50 patients with immediate internal plate fixation of an open diaphyseal fracture of the forearm. The functional results were excellent or good in 85%.

     

    The review by Levin is a comprehensive review of the literature on early versus delayed closure of open fractures, and covers the change in thought from previous literature, including change in technology, surgical techniques, and a more critical review of previous literature.

     

     

     

    OrthoCash 2020

     

  86. During a Lisfranc (tarsometatarsal) amputation of the foot, which of the following is crucial to prevent the patient from having a supinated foot during gait.

    1. Releasing the posterior tibialis tendon

    2. Preserving the soft-tissue envelope (peroneus brevis, tertius and plantar fascia) around the fifth metatarsal base

    3. Myodesis of the anterior tibialis to the medial and middle cuneiforms

    4. Lengthening of the gastrocsoleus (achilles tendon)

    5. Osteotomy through 1st metatarsal Corrent answer: 2

    A Lisfranc amputation is through the tarsometatarsal joints, except the 2nd metatarsal, which is osteotomized to preserve the stability of the medial cuneiform. To prevent the patient from supinating the foot following this amputation, the evertors on the foot must be maintained. The principal evertors are the peroneus brevis and longus (Illustration A). Therefore, the function of the peroneus brevis must be preserved. Technically this is done preserving the soft-tissue envelope (peroneus brevis, tertius and plantar fascia) around the fifth metatarsal base.

    Illustration B depicts the level of a Lisfranc amputation of the foot. Incorrect Answers:

    1-The posterior tibialis is the primary supinator of the foot, and releasing it

    would lead to an eversion deformity. The tibialis posterior tendon attachment to the bases of the second and third metatarsals will actually be released with this amputation, but the main attachment to the navicular preserved.

    3-The anterior tibialis dorsiflexes and inverts the foot, but transferring it to the medial and middle cuneiforms would mimick its native function to dorsiflex and invert the foot.

    4-A lengthened Achilles would lead to increased dorsiflexion, not supination.

    5-Osteotomy of 2nd MT is crucial to preserve the medial cuneiform and midfoot stable.

     

     

     

     

     

     

     

     

     

    OrthoCash 2020

     

  87. A 39-year-old male is thrown from his motorcycle into a fast-food restaurant and sustains a closed pelvic ring injury. During placement of percutaneous iliosacral screws, the outlet radiograph in Figure A is obtained. What purpose does this view serve?

     

     

     

    1. Evaluation of possible injury to L5 nerve root

    2. Evaluation of anterior-posterior position of screw(s)

    3. Best visualization of sagittal curvature of sacral ala

    4. Best visualization of spinal canal

    5. Best visualization of sacral neural foramina Corrent answer: 5

    Figure A shows an intraoperative outlet view, which provides the best visualization of the neural foramina (and possible screw placement into these foramina). This view provides information regarding cephalad-caudad placement of the screw, whereas the inlet view provides information regarding the anterior-posterior position of the screw. The lateral sacral view provides information regarding the sagittal curvature of the sacral ala and gives information regarding possible iatrogenic L5 nerve injury as it goes over the sacral ala.

     

    The referenced article by Routt et al is a review article regarding the safety and techniques of percutaneous pelvic ring fixation.

     

     

     

    OrthoCash 2020

     

  88. A 35-year-old male sustains a closed Schatzker VI tibial plateau fracture. Two weeks following external fixation, examination reveals intact sensation, palpable pulses and no soft tissue compromise. An axial CT image is shown in Figure A. What is the optimal surgical plan?

     

     

     

    1. Medial and lateral plate fixation through two approaches

    2. Medial and lateral plate fixation through a single anterior approach

    3. Lateral locking plate fixation

    4. Continued external fixation until union

    5. Multiplanar transarticular external fixator Corrent answer: 1

    The Figure shows a bicondylar tibial plateau fracture. The goals that need to be met when treating tibial plateau fractures are the following: restoration of mechanical axis alignment, restoration of condylar width, articular reduction, and restoration of knee stability. Since the soft tissue envelope is favorable, open reduction internal fixation with dual incisions and dual plates will provide the best probablity of achieving those goals.

     

    Gosling et al did a biomechanical evaluation in cadavers comparing lateral locked plating with a combined medial and lateral plate and found no difference in resistance to vertical subsidence even with loads exceeding the average body weight. However, this was a cadaveric study with no mention and capability of analyzing articular reduction. Lateral locked plating only allows for indirect reduction of the medial plateau.

     

    Barei et al in a retrospective review found that comminuted bicondylar tibial plateau fractures can be successfully treated with open reduction and medial and lateral plate fixation using 2 incisions, and postulate that the use of 2 incisions may contribute to a lower wound complication rate. A two incision approach allows not necessarily for a stronger construct as some studies are controversial, but for a more accurate reduction and restoration of alignment.

     

     

    OrthoCash 2020

     

  89. Which of the following is true regarding the use of the saline injection load test to diagnose traumatic knee arthrotomies?

    1. Addition of methylene blue to the saline load test increases the sensitivity of the test

    2. Injection of 110ml of saline will diagnose 95% of knee arthrotomies

    3. Injection of 175ml of saline will diagnose 99% of knee arthrotomies

    4. A superomedial injection location requires significantly less fluid than a inferoeromedial injection location

    5. A history and physical exam by an orthopaedic surgeon has equivalent sensitivity to saline load test at detecting a traumatic arthrotomy

    Corrent answer: 3

     

    Injection of 175ml of saline will diagnose 99% of knee arthrotomies.

     

    Clinical evaluation alone to determine if a periarticular laceration has penetrated the joint can often be incorrect. A Saline Load Test (SLT) is an effective methods to detect intraarticular penetration. New studies have shown addition of methylene blue does not improve the diagnostic value of the saline load test. The use of a CT scan can be helpful, especially in the presence of intra-articular air.

     

    Voit et al. investigated the sensitivity of the clinical exam and a saline load test in 50 consecutive patients with periarticular lacerations suggestive of joint penetration. In 14 there was leakage of fluid on saline load test. In six of these patients, the clinician had judged there was no traumatic arthrotomy based on physical exam and clinical history. They therefore concluded performing a saline load test is important adjunct and the clinical exam alone can not be relied on to detect traumatic arthrotomies.

     

    Nord et al. found that the volumes of saline that were needed in order to effectively diagnose 75%, 90%, 95%, and 99% of the knee arthrotomies were

    110, 145, 155, and 175 mL, respectively. They also found that an inferomedial injection location required significantly less fluid than a superomedial injection location did.

     

    Metzger et al. studied 58 patients that underwent saline load test with about 100ml of saline injected (methylene blue 29, normal saline 29). They found that the false-negative rate was 67% (methylene blue 69%, normal saline 66%). They concluded the addition of methylene blue does not improve the diagnostic value of the saline load test.

    Konda et al. performed a study evaluationg the role of CT scan versus saline load test. They found that the sensitivity and specificity of the CT scan to detect traumatic arthrotomy was 100%. In a subgroup of 37 patients that received both a CT scan and the conventional saline load test, the sensitivity and specificity of the CT scan was 100% compared to 92% for the saline load test (p<0.001).

     

    Incorrect Answers:

    Answer 1: Addition of methylene blue to the saline load test does not increase the sensitivity of the test. (Metzger et al.)

    Answer 2: Injection of 110ml of saline will diagnose 75% of knee arthrotomies. (Voit et al.)

    Answer 4: A inferoeromedial injection location requires significantly less fluid than a superomedial injection location (Nord et al.)

    Answer 5: Physical exam performs worse than the conventional saline load test to detect traumatic knee arthrotomies

     

     

     

    OrthoCash 2020

     

  90. A 36-year-old rancher is involved in a tractor roll-over accident and sustains the injury shown in Figure A to his dominant right arm. After undergoing rigid anatomic fixation of the fracture, the distal radio-ulnar joint (DRUJ) remains incongruent. What is the next step in management?

     

     

     

     

    1. Revision plating of the fracture

    2. Revision reduction and intramedullary fixation

    3. Reduction of interposed extensor carpi ulnaris tendon

    4. Reduction of interposed pronator quadratus tendon

    5. Reduction of interposed flexor carpi ulnaris tendon Corrent answer: 3

    The most likely cause of persistent DRUJ incongruity after anatomic reduction and fixation of the radial shaft fracture of the answers above is interposition of the extensor carpi ulnaris (ECU) tendon. The tendon must be extricated from the joint to permit DRUJ reduction.

     

    Gaeleazzi fracture-dislocations, such as that seen in Figure A, are typically stable once the radial shaft fracture is anatomically reduced. After fixation, the DRUJ is translated in pronation, supination, and in a neutral position to test for stability. A “clunk” during passive motion of the DRUJ is further evidence of gross instability. Gross laxity can be treated by splinting in supination or by pinning the DRUJ. However, ECU tendon interposition has been reported as a possible cause of a persistently irreducible DRUJ. Radiographic findings typically demonstrate a dorsally displaced ulnar head and a widened DRUJ. The interposed tendon must be removed from the joint, often through a separate dorsal approach to permit DRUJ reduction.

     

    Bruckner et al. review the evaluation and management of complex dislocations of the DRUJ. The authors note that these injuries are associated with frequent irreducibility, recurrent subluxation, or soft reduction of the DRUJ secondary to interposed tissue. In their institutional series, four of the 11 cases of Galeazzi fractures were associated with complex DRUJ dislocations, most commonly due to displacement of the ECU tendon volar to the ulna, necessitating open reduction. They cautioned that unobtainable or unconvincing reductions should warrant surgical exploration.

     

    Paley et al. reported two cases of an irreducible DRUJ after radial shaft fracture fixation. The authors describe an empty ECU tendon sulcus on the dorsum of the wrist in both cases. One case was noted and addressed intraoperatively.

    However, the second case was not identified and this patient went on to endure persistent subluxation and diastasis of the DRUJ, ultimately experiencing a poor result. The authors advocate a separate dorsal exposure to reduce the ECU.

     

    Hanel and Scheid reported a case of entrapment of the ECU in the DRUJ in a skeletal immature 12 year old boy. They noted that intraoperative radiographic analysis was significant for a widened DRUJ and dorsally displaced ulnar head. These authors too advocated a separate dorsal exposure to approach and extricate the ECU tendon.

    Incorrect answers

    Answer 1: The fracture is anatomically reduced but the DRUJ is incongruent because of an interposed ECU tendon. Revision fixation will not address the DRUJ incongruity.

    Answer 2: The type of fixation is not the issue. Revising the fixation to another construct will not address the ECU interposition.

    Answer 4: Pronator quadratus interposition has not been described in the setting of irreducible DRUJ dislocations.

    Answer 5: Flexor carpi ulnaris tendon interposition has also not been described in the setting of irreducible DRUJ dislocations.

     

     

     

    OrthoCash 2020

     

  91. A 58-year-old right-hand-dominant computer programmer trips and falls onto his right arm. He reports right arm pain and that his elbow felt "sloppy". His initial lateral radiograph is shown in Figure A. The orthopedic junior resident counsels him that he will likely need a radial head arthroplasty, ligament repair, and possible fixation of the ulna. What factor would most significantly affect the decision to surgically address the ulna fracture?

     

     

     

     

    1. Degree of radial head comminution

    2. The deforming force acting on the avulsed fracture fragment

    3. Size of fragment and elbow stability after radial head replacement

    4. The degree of fracture displacement

    5. Patient age and bone quality Corrent answer: 3

    The size of the fragment and degree of elbow instability following radial head fixation or replacement most often determines the intraoperative decision on coronoid fragment fixation.

     

    The radial head is a secondary restraint to posterolateral rotatory instability (PLRI) of the elbow, while the coronoid provides an anterior and varus buttress to the ulnohumeral joint, resisting posterior dislocation. The medial ulnar collateral ligament attaches to the anteromedial facet of the coronoid, so large medial facet fractures may displace and cause varus posteromedial instability (PMRI). However, in terrible triad injuries (causing PLRI), small coronoid tip fractures are more common. These are typically left as they most often do not contribute to elbow instability. Therefore in most cases, radial head replacement and lateral ligamentous repair are sufficient to restore stability.

    However larger coronoid base fractures may require fixation in order to stabilize the elbow. In these cases, the coronoid fracture may be addressed through the lateral window after radial head resection and before the trial implant is assessed. The anterior capsule can be tied down to the ulna or if the fracture is large enough, it may sometimes tolerate internal fixation.

    Conversely, a buttress plate for the coronoid may be applied via a medial approach in the setting of a large coronoid base fracture as in PMRI.

     

    Ring et al. reviewed 11 terrible triad injuries including 7 of which had the radial head surgically addressed and 4 of which had undergone radial head excision. The authors found that all four patients who underwent radial head excision dislocated after surgery. Only four patients had a satisfactory result, and all of these had fixation of the radial head, two requiring concomitant repair of the lateral ulnar collateral ligament (LUCL). The authors concluded that terrible triad injuries were unstable, prone to redislocation and that radiocapitellar contact is critical for ulnohumeral stability.

     

    Pugh et al. reviewed 36 cases of terrible triad injuries in which all coronoid fractures were addressed, either with screw fixation or suture repair of the anterior capsule. The authors reported that the average arc of motion postoperatively was 112 degrees, Mayo score was 88, and concentric stability was restored in 34/36. There were 8 complications requiring re-operation. They concluded that coronoid fixation with radial head fixation or replacement yields a stable elbow suitable for early motion.

     

    Schneeberger et al. evaluated elbow instability after simulated terrible triad injuries. They found that radial head excision even in the setting of an intact LUCL lead to posterolateral laxity. Furthermore, if 30% of the coronoid tip was excised, the elbow dislocated at 60 degrees of flexion, but stability was restored with a radial head replacement. However, if 50% of the coronoid was

    excised, even with a radial head replacement dislocation occurred. They concluded that so long as the radial head is replaced, small coronoid fractures may not need to be repaired.

     

    Figure A demonstrates a comminuted radial head fracture, a small coronoid fracture, and subluxation of the ulnohumeral joint.

     

    Incorrect answers:

    Answer 1: The degree of radial head comminution impacts the decision to replace or fix the radial head. However, this does not affect the decision to fix the coronoid.

    Answer 2: Though the tip of the coronoid often remains attached to the anterior capsule which attaches slightly distal to the coronoid tip, there is no deforming force on this fracture which is instead a shear and not an avulsion injury.

    Answer 4: Elbow stability, not the degree of coronoid displacement, affects the decision to address the fracture.

    Answer 5: The patient's age and bone quality does not factor into elbow stability or decision making on coronoid fixation.

     

     

     

    OrthoCash 2020

     

  92. What is the most appropriate plating technique utilized for the medial malleolus fracture typically seen in a displaced supination-adduction ankle fracture?

    1. Tension band plating

    2. Antiglide plating

    3. Bridge plating

    4. Neutralization plating

    5. Submuscular plating

     

    Corrent answer: 2

     

    A supination-adduction ankle fracture leads to a vertical fracture of the medial malleolus. Traditional fixation of the medial malleolus with oblique screws from the tip of the malleolus directed proximally will ineffectively protect against shear forces at the fracture site; these also are directed quite obliquely to the vertical fracture line, and therefore have poor biomechanical resistance to failure. An antiglide plate is used medially to prevent displacement of the fracture segment due to shear forces.

     

    According to the referenced article by Toolan et al, placement of two horizontal (perpendicular to the fracture line) lag screws from medial to lateral are

    biomechanically the most important aspect of the construct whether a plate is used or not.

     

     

     

    OrthoCash 2020

     

  93. A 25-year-old Norwegian amateur curler slips on the ice, falling onto an outstretched right elbow. He is taken to the local teaching hospital and radiographs demonstrate a significantly comminuted radial head fracture and coronoid base fracture. His elbow is reduced and splinted. To restore stability and allow early range of motion, which of the following will most likely need to be performed in most cases?

    1. Radial head fixation or replacement

    2. Radial head fixation or replacement and coronoid fixation

    3. Radial head fixation or replacement, coronoid fixation, and lateral ulnar collateral ligament (LUCL) repair

    4. Radial head fixation or replacement, coronoid fixation, LUCL and medial ulnar collateral ligament (MUCL) repair

    5. Radial head fixation or replacement, coronoid fixation, LUCL and MUCL repair, and application of a hinged fixator

    Corrent answer: 3

     

    The patient has sustained a "terrible triad" injury, classically involving a radial head fracture, coronoid fracture, and elbow dislocation. These often involve LUCL injuries and a traumatic injury in the radiocapitellar joint. Stability is achieved with radial head replacement (or fixation), coronoid fixation (in cases with a large coronoid fracture), and lateral soft tissue repair.

     

    Posterolateral rotatory instability (PLRI) following a terrible triad injury is usually caused by a fall on an extended arm that produces a valgus, axial, and rotatory force. The mechanism of injury begins laterally and moves medially.

    Hence, the LUCL fails first, followed by the anterior capsule (or coronoid), and lastly the MUCL. Even following fixation, patients often lose some degree of their flexion-extension arc, may develop post-traumatic arthritis, or most commonly may have persistent instability. The radial head is a primary restraint to PLRI and must be either replaced with a prosthesis or fixed in the setting of a terrible triad injury. Replacement is typically chosen when the radial head is in more than 3 fragments. Coronoid fractures should be fixed when they involve >30-50% of the coronoid base. However, the best way to determine if coronoid fixation is necessary is with an intraoperative fluoroscopic examination.

    Forthman et al. reviewed outcomes following the management of 34 elbow fracture-dislocations. In all cases, the radial head was replaced or underwent fixation. The MUCL was not repaired in any case. The authors noted only 2 cases of post-operative instability, one terrible triad, and one combined capitellum and trochlea fracture. The authors noted that both cases were related to non-compliance. The remaining 32 averaged 120 degrees of flexion-extension and 74% had good-excellent outcomes. The authors concluded that MUCL repair is not required for elbow dislocations so long as the radial head, large coronoid fractures, and LUCL are addressed.

     

    Schneeberger et al. evaluated elbow instability after simulated terrible triad injuries. They found that radial head excision even in the setting of an intact LCL leads to posterolateral laxity. If 30% of the coronoid was excised, the elbow dislocated at 60 degrees of flexion, but stability was restored with a radial head replacement. However, if 50% of the coronoid was excised, even with a radial head replacement, dislocation occurred. This led the authors to conclude that long as the radial head was replaced, small coronoid fractures may not need to be repaired.

     

    Papatheodorou et al. reviewed 14 terrible triad injuries (all of which had Regan-Morrey type 1 or 2 coronoid fractures) that underwent surgical fixation or replacement of the radial head combined with LUCL repair. The authors noted that intraoperative stability was confirmed in all cases without coronoid fixation, MCL repair, or an external fixator. The authors concluded that coronoid fixation is not required in fractures up to 50% the height of the coronoid and that MCL repair is not necessary.

     

    Incorrect answers:

    Answer 1: Large coronoid base fractures need to be fixed in PLRI to confer elbow stability.

    Answer 2: The lateral collateral ligamentous complex should be repaired as this soft tissue sleeve is usually avulsed in elbow dislocations.

    Answer 4: In PLRI, ligamentous repair of the medial aspect of the elbow is often unnecessary.

    Answer 5: A hinged fixator (internal or external) is not typically required. This could be applied in a case where the elbow is persistently unstable at the conclusion of the case.

     

     

     

    OrthoCash 2020

     

  94. A 32-year-old female sustains the injury shown in Video A. The right-sided pelvic injury is best classified as which of the following?

    1. Lateral compression 1

    2. Lateral compression 2

    3. Vertical shear

    4. Anterior-posterior compression 2

    5. Anterior-posterior compression 3

     

    Corrent answer: 2

     

    The injury shown in Video V reveals a right sided posterior ilium fracture, which is known as a crescent fracture. The presence of a crescent fracture is consistent with a lateral compression type 2 injury; this differentiates this from a type I injury. The ipsilateral anterior sacrum has a small impaction injury anteriorly while the contralateral SI joint has a minor amount of anterior sacral impaction indicative of a lateral compression type I injury.

     

    The reference by Burgess et al is the primary source of the mechanism classification of pelvic ring injuries. Overall blood replacement averaged 5.9 units (lateral compression, 3.6 units; anteroposterior compression, 14.8 units; vertical shear, 9.2 units; combined mechanical, 8.5 units). Overall mortality was 8.6% (lateral compression, 7.0%; anteroposterior, 20.0%, vertical shear,

    0%; combined mechanical, 18.0%).

     

    Incorrect answers:

    1: The presence of a crescent fracture means this is at least a LC-2 injury. The left-sided fracture pattern is consistent with an LC-1 pattern.

    3: A vertical shear fracture pattern would exhibit some vertical displacement and does not typically exhibit the crescent fragment.

    4: The fracture pattern does not match an anterior-posterior compression pattern.

    5: The fracture pattern does not match an anterior-posterior compression pattern.

     

     

     

    OrthoCash 2020

     

  95. A 35-year-old zookeeper fell 10 feet while preparing an exhibit for a grand reopening, landing on his left arm. The patient is then evaluated by a keen orthopedic resident in the emergency room who describes the zookeeper's injuries to his chief. He describes a comminuted radial head fracture and posterolateral ulnohumeral dislocation. The chief resident orders a CT scan which demonstrates a coronoid fracture involving 50% the height with no involvement of the anteromedial facet. During surgery, the trauma surgeon replaces the radial head and repairs the lateral collateral ligament complex. The

    elbow is splinted in elbow flexion and pronation. The patient begins range of motion exercises with her occupational therapist 3 days after surgery, and her elbow dislocates. What is the most likely reason for her instability?

    1. Length of immobilization

    2. Position of immobilization

    3. Lack of coronoid fixation with medial buttress plate

    4. Lack of coronoid fixation from lateral approach

    5. Lack of medial collateral ligament repair Corrent answer: 4

    The zookeeper sustained a "terrible triad" injury with resulting posterolateral rotatory instability (PLRI). To prevent post-operative instability, large coronoid fractures should be fixed, and this would be performed through a lateral approach given that the radial head will be replaced.

     

    The coronoid serves as an anterior and varus buttress to the ulnohumeral joint, resisting posterior dislocation. Though the coronoid tip has no soft tissue attachments, the medial ulnar collateral ligament attaches to the anteromedial facet of the coronoid. A coronoid fracture through the anteromedial facet, such as in the setting of posteroMEDIAL rotatory instability, would require a medial buttress plate to restore varus stability. In a coronoid base fracture involving 50% of the coronoid, fixation is required even when a radial head arthroplasty is performed. This can be done with suture fixation via a bone tunnel through the ulna or with screws from dorsal to volar. Failure to fix a large coronoid base fracture would result in persistent instability, as with the patient in this vignette.

     

    Ring et al. reviewed 11 terrible triad injuries - including 7 of which had the radial head surgically addressed and 4 of which underwent radial head excision. They noted that all four patients who underwent radial head excision dislocated after surgery. Moreover, only four patients had a satisfactory result, and all of these had fixation of the radial head, with two requiring concomitant repair of the lateral ulnar collateral ligament (LUCL). The authors concluded that terrible triad injuries are unstable, prone to redislocation and that radiocapitellar contact is critical for ulnohumeral stability.

     

    Schneeberger et al. evaluated elbow instability after simulated terrible triad injuries. They showed that radial head excision even with an intact LUCL resulted in persistent posterolateral laxity. If 30% of the coronoid was excised, the elbow dislocated at 60 degrees of flexion, but stability was restored with a radial head replacement alone. If 50% of the coronoid was excised, even with

    a radial head replacement, dislocation occurred. The authors concluded that so long as the radial head was replaced, small coronoid fractures may not need to be repaired, but large fractures involving 50% or more would require fixation even if the radial head and LUCL were addressed.

     

    Moro et al. treated 25 unreconstructible radial head fractures with a metal radial head arthroplasty. The authors reported a DASH score of 17, PRWE of 17, and Mayo elbow score of 80. Poor outcomes were seen in those with psychiatric disorders or those involved in a worker's compensation claim. The authors reported that radial head arthroplasty resulted in stable elbows with mild-moderate physical impairment.

     

    Ring, Quintero, and Jupiter reviewed 56 patients with radial head fractures who underwent surgical fixation. Of the comminuted Mason type-3 fractures that underwent ORIF, they found that 13/14 had a poor result. The authors recommend that fractures with 3 or fewer fragments are amenable to fixation while those with 4 or more articular fragments are not.

     

    Incorrect answers:

    Answer 1: Early range of motion is appropriate after adequate fixation. Answer 2: Flexion and pronation is the position that stabilizes the lateral elbow. In the setting of PLRI, this is the correct position to splint the upper extremity in following surgery, therefore the position of immobilization is not the issue in this case.

    Answer 3: Large anteromedial facet fragments result in posteromedial rotatory instability. These do not typically occur in conjunction with a radial head fracture. These result in varus instability and need to be buttressed with a medial plate. In this case, a medial buttress plate is not needed.

    Answer 5: In PLRI, the medial side of the elbow is not typically addressed. The more obvious cause of instability is the lack of coronoid fixation.

     

     

     

    OrthoCash 2020

     

  96. Which of the following injuries would require plating of the radius along with closed reduction and evaluation of the distal radioulnar joint (DRUJ)?

    1. Nightstick fracture

    2. Galeazzi fracture

    3. Monteggia fracture

    4. Rolando fracture

    5. Smith fracture

    Corrent answer: 2

     

    A Galeazzi fracture is a fracture of the distal third of the radius with dislocation of the distal radioulnar joint. It commonly results from a fall onto an outstretched hand with the forearm in pronation.

     

    A Galeazzi fracture is an injury that requires surgical treatment in an adult. The algorithm includes anatomic reduction and fixation of the radial shaft, and closed reduction of the DRUJ with assessment of stability. If the DRUJ remains unstable, supination of the wrist may reduce the DRUJ. Otherwise, either open or closed reduction with pinning can be undertaken to stabilize the joint. The closer the radius fracture is to the DRUJ, the more likely it is to be unstable.

    Acute intervention results in improved outcomes as compared to delayed reconstruction.

     

    Rettig et al. retrospectively analyzed 40 patients with Galeazzi fracture-dislocations that were treated with open reduction and internal fixation of the radial shaft fracture. They noted that a radial shaft fracture located within

    7.5cm of the articular surface resulted in a 55% incidence of DRUJ instability, while 5.6% of those located further from the joint were unstable. They concluded that a high index of suspicion, early recognition, and acute treatment of DRUJ instability will avoid chronic problems in this complex injury.

    Illustration A is an AP and lateral radiograph demonstrating a Galeazzi fracture. Incorrect Answers:

    Answer 1: A nightstick fracture is an isolated ulnar shaft fracture.

    Answer 3: A Monteggia fracture is a proximal ulnar fracture associated with a radial head dislocation.

    Answer 4: A Rolando fracture is a comminuted intra-articular fracture through the base of the first metacarpal bone.

    Answer 5: Smith fractures are fractures of the distal radius with associated volar angulation of the distal fracture fragment(s).

     

     

     

     

     

    OrthoCash 2020

     

  97. In trauma patients with multiple injuries, patients with scapula fractures have been shown to have an association with which of the following, as compared to patients without scapula fractures?

    1. Increased length of hospital stay

    2. Increased mortality rate

    3. Increased rate of extremity fracture(s)

    4. Increased Injury Severity Scores

    5. Increased length of intensive care unit stay Corrent answer: 4

    According to the reference by Veysi et al, patients presenting to a trauma center with scapula fractures have an increased rate of pulmonary complications and increased Injury Severity Scores (ISS), but have no difference in mortality, length of ICU stay, or overall hospital stay. No differences were seen in abdominal or head injury rates either. A lower rate of extremity fractures was seen as compared to non-scapular fracture patients in their series.

    According to the referenced study by Brown et al, rib fx (44%) are the most common associated injury with scapula fractures.

     

     

     

    OrthoCash 2020

     

  98. All of the following techniques can help to prevent apex-anterior angulation during intramedullary nailing of proximal one-third tibia fractures EXCEPT:

    1. Posterior blocking screw

    2. Posterior starting hole

    3. Interlocking the nail in a semi-extended knee position

    4. Anteriorly directing the nail

    5. Anterior blocking screw Corrent answer: 5

    Sagittal malalignment is commonly seen after nailing proximal tibia fractures. The start point as well as the direction of the nail can lead to sagittal deformity. Freedman found in nailing tibia fractures that malalignment was seen in 58% of proximal third fractures, 7% of middle third fractures, and 8% of distal third fractures. Of the malaligned fractures, 83% were either segmental or comminuted.

     

    Lang found that a medialized nail entry point and a posteriorly and laterally directed nail insertion angle contributed to malalignment. It is logical that a fracture that is reamed and then nailed in the posterior direction will lead to a gap anteriorly, and that posterior comminution will lead to anterior angulation as the fracture hinges on the intact cortex anteriorly. An anterior starting hole will tend to lead to more of a posterior nail direction.

     

    Tornetta found that using only 15 degrees knee flexion (semi extended) eliminated the extension force of the quadriceps on the proximal fragment, which otherwise would have tended to cause anterior angulation at the fracture site; therefore interlocking in flexion leads to anterior angulation.

     

    Krettek found that a posteriorly placed blocking screw is meant to prevent posterior placement of the nail and therefore encourages decreased anterior angulation of the fracture.

     

    Henley found that if the fracture is high and the nail bend is within the distal fracture fragment, as the nail is inserted, the nail will drive the distal fragment posteriorly.

     

     

    OrthoCash 2020

     

  99. Which of the following factors has been shown to be the strongest predictor of screw cutout of a dynamic compression hip screw used for an intertrochanteric femur fracture?

    1. Age of the patient

    2. Intrinsic stability of the fracture

    3. Tip-apex distance

    4. Quality of reduction

    5. Angle of the sideplate Corrent answer: 3

    Illustration A found below depicts the method to calculate Tip-apex distance (TAD). The tip-apex distance is the sum of the distances from the tip of the lag screw to the apex of the femoral head as seen on the AP and lateral radiographs.

     

    Baumgaertner et al reported that he had no screws cut out if the tip-apex distance was less than 25mm. Tip-apex distance was the strongest predictor of cutout. Increasing age of the patient, poor reduction, use of a high angle sideplate, and unstable fracture were weaker predictors of cutout.

     

    Kyle et al demonstrated that obtaining an anatomic reduction when using a sliding hip screw with intertrochanteric fractures leads to the best radiographic and clinical outcomes.

     

     

     

     

     

    OrthoCash 2020

     

  100. A 66-year-old male sustains an open crush injury to his right lower leg with significant skin loss. His history is significant for COPD, diabetes controlled with an insulin pump, and testicular cancer treated with bleomycin twenty years ago. A radiograph of the chest shows a small pneumothorax which is being observed and does not require a thoracostomy tube. Which of the following is not a contraindication to hyperbaric oxygen treatment for this patient?

    1. Presence of an acute open fracture and crush injury

    2. History of COPD

    3. History of bleomycin treatment

    4. Presence of a pneumothorax

    5. Presence of an insulin pump Corrent answer: 1

    The presence of a crush injury to an extremity is an indication for hyperbaric oxygen (HBO) therapy. The remainder of the options listed are contraindications to hyperbaric oxygen treatment.

     

    Hyperbaric oxygen therapy potentially can provide enhanced oxygen delivery to peripheral tissues affected by vascular disruption, cytogenic and vasogenic edema, and cellular hypoxia caused by extremity trauma. The idea behind HBO is to provide enhanced oxygen delivery to peripheral tissues affected by vascular disruption, cytogenic and vasogenic edema, and cellular hypoxia caused by extremity trauma.

     

    Greensmith et al provide a review of HBO therapy and discuss the relative and absolute contraindications and indications for this treatment. They report in patients with crush injury or early compartment syndrome, hyperbaric oxygen therapy may reduce the penumbra of cells at risk for delayed necrosis and secondary ischemia. They report that both animal studies and prospective human clinical trials suggest the benefits of such therapy.

     

    Buettner et al found that based on clinical evidence and cost analysis, medical institutions that treat open fractures and crush injuries are justified in incorporating HBO theray as a standard of care.

    Illustration A shows an example of a hyperbaric oxygen(HBO) chamber. Incorrect Answers:

    Answer 2: COPD is a contraindication to hyperbaric oxygen treatment due to

    the possible presence of air trapping or bleb formation, which could place the patient at risk of pneumothorax.

    Answer 3: History of bleomycin treatment is a contraindication to hyperbaric oxygen treatment because supplemental oxygen may cause life threatening pneumonitis.

    Answer 4: Presence of a pneumothorax is a contraindication as hyperbaric oxygen treatment may cause a gas embolism, tension pneumothorax, or pneumomediastinum.

    Answer 5: Presence of an insulin pump is a contraindication to hyperbaric oxygen treatment because malfunction or deformation of the device may occur under pressure.

     

     

     

     

     

     

    OrthoCash 2020

     

  101. A 34-year-old male falls off of a ladder and sustains the ankle injury shown in Figure. Which of the following is unique with this particular ankle fracture pattern and must be recognized by the operating surgeon to optimize outcomes?

     

     

     

    1. Marginal impaction of the anteromedial tibial plafond

    2. Syndesmosis diastasis

    3. Deltoid ligament tear

    4. Posterolateral osteochondral lesion of the talus

    5. Fibular overlengthening

     

    Corrent answer: 1

     

    The radiograph demonstrates a Lauge-Hansen supination-adduction fracture-dislocation. There is a transverse fibula fracture and a vertical medial malleolus fracture.

     

    McConnell and Tornetta performed a Level 4 review and found that nearly 50% of these injuries have marginal impaction of the anteromedial tibial plafond and they found that anatomic reduction of that aspect of the injury led to good to excellent outcomes.

     

     

     

    OrthoCash 2020

     

  102. A 42-year-old female undergoes a subtalar bone block distraction arthrodesis as sequelae of a nonoperatively treated calcaneus fracture ten years prior. This procedure improves which of the following issues?

    1. Subtalar joint stiffness

    2. Midfoot supination

    3. Sinus tarsi impingement

    4. Anterior ankle impingement

    5. Hammertoe deformity

     

    Corrent answer: 4

     

    The subtalar fusion technique involves distraction of the subtalar joint, insertion of a bone block, fusion, and rigid screw fixation. The distraction allows correction of the talocalcaneal relationship.

     

    In Carr’s series, pre- and postoperative radiographic analysis for tibiotalar impingement, lateral talocalcaneal angle, and talonavicular alignment was performed, with improvement to a normal range seen in the cases analyzed.

     

    Bednarz did a radiographic analysis and showed an average increase of 8 mm in hindfoot height, 9 degrees in lateral talocalcaneal angle, and 11 degrees in lateral talar declination angle.

     

    Rammelt found that compared with the unaffected side, the talocalcaneal height was corrected by 61.8%, the talus-first metatarsal axis by 46.5%, the talar declination angle by 38.5% and the talocalcaneal angle by 35.4%. Based on these three references, this procedure ultimately addresses the lost hindfoot height, subtalar arthritis (joint is fused), ankle impingement (improvement of the talus 1st MT axis), and peroneal impingement. It does not address hindfoot valgus. However, the deformity after a calcaneus fracture is usually from lateral wall blowout and hindfoot varus.

     

    Illustration A shows a lateral radiograph of a distraction/bone block subtalar arthrodesis.

     

     

     

     

     

    OrthoCash 2020

     

  103. A 29-year-old male sustained a mid-shaft femur fracture in a motorcycle accident. Which of the following is associated with approximately 5% of patients sustaining this injury?

    1. Heterotopic ossification

    2. Ipsilateral femoral neck fracture

    3. Ipsilateral posterolateral corner injury

    4. Pudendal nerve injury

    5. Ipsilateral superficial femoral artery injury Corrent answer: 2

    Ipsilateral femoral neck fractures are seen in 1-9% of femoral shaft fractures and the femoral neck must be properly imaged either preoperatively or intraoperatively in any patient with a femoral shaft fracture. Dedicated hip films, possibly including an internal rotation AP, should be obtained before entering the OR.

     

    Daffner et al reported that in 11 of 20 cases of combined femoral shaft and neck fractures, the initial preoperative radiographs did not demonstrate the femoral neck fracture. Intraoperative fluoroscopy should also be used to evaluate for a femoral neck fracture both before (to evaluate for unrecognized fx) and after (to evaluate for iatrogenic fx) IM nailing.

     

    Tornetta et al also describe using preoperative CT scans to evaluate for a

    femoral neck fracture and found that they were able to reduce the number of missed ipsilateral femoral neck fractures.

     

     

     

    OrthoCash 2020

     

  104. A 51-year-old right-hand-dominant male fell onto his left arm and sustained the isolated injury shown in Figures A and B approximately 6 months prior to presentation. Examination of the wrist is notable for a stable DRUJ and no tenderness. The elbow shows no ligamentous laxity, and the patient reports isolated elbow pain during attempted pronation/supination Current radiographs reveal a malunited radial head fracture. Treatment should now consist of?

     

     

     

     

     

     

    1. Radial head resection

    2. Radial head replacement

    3. ORIF of the malunited fracture

    4. Arthroscopic debridement

    5. Total elbow replacement Corrent answer: 1

    The injury films represent a highly comminuted radial head fracture, which when treated with ORIF, is likely to have a poor outcome especially in the delayed setting. In the absence of DRUJ and elbow instability, and no wrist tenderness, radial head resection is the best treatment option. While a radial

    head prosthesis may theoretically prevent proximal radial head migration that can occur with radial head excision, this is unlikely in an isolated radial head fracture without other ligamentous injury (elbow ligaments, interosseous membrane or DRUJ).

     

    Herbertsson et al (March, 2004) reported on 100 patients with Mason II and III radial head fractures. Nine of those initially treated nonoperatively had continued pain and underwent late radial head excision with good results.

     

    In another report by Herbertsson et al (September, 2004) the authors report on radial head excision in both an acute and delayed setting and found fair-good results for most patients in both subgroups.

     

    Jackson et al present a review article on radial head fractures where management and operative technique are discussed.

     

    Finally, Antuna et al in their study conclude "Radial head resection in young patients with isolated fractures without instability yields long-term satisfactory results in >90% of cases. Osteoarthritic changes are uniformly present but typically are not associated with functional impairment".

     

     

     

    OrthoCash 2020

     

  105. What risk factor leads to the highest rate of postoperative loss of reduction in unstable posterior pelvic ring injuries?

    1. Type of anterior fixation

    2. Male sex

    3. Usage of a transiliac bar

    4. Vertical sacral fracture

    5. Sacroiliac joint fracture-dislocation Corrent answer: 4

    According to the referenced article by Griffin et al, the risk of postoperative loss of reduction is greatest with a vertical sacral fracture pattern (13%, all within 3 weeks). There was no significant association between failure and anterior fixation method, iliosacral screw arrangement or length, or any demographic or injury variable.

     

    Their conclusion: "Percutaneous iliosacral screw fixation is a useful technique in the management of vertically unstable pelvic fractures, but a vertical sacral fracture should make the surgeon more wary of fixation failure and loss of reduction."

     

     

    OrthoCash 2020

     

  106. A 22-year-old male sustains the closed injury seen in figure A. The injury is best treated with which of the following methods?

     

     

     

     

    1. External fixation

    2. Flexible intramedullary nailing

    3. Open reduction and internal fixation with acute bone grafting

    4. Open reduction and internal fixation

    5. Closed reduction and functional bracing Corrent answer: 4

    The xray shows a comminuted, high energy both bone forearm fracture. Open reduction and internal fixation without bone grafting is the most appropriate treatment. Acute bone grafting is only indicated if a large bony void, such as segmental bone loss of the radius, is present to allow the displaced bone ends to heal together despite their diastasis.

     

    In Moed et al's review, primary internal fixation in open forearm fractures led to nearly a 90% union rate (44/50) and good/excellent results in 85%.

    Autogenous cancellous grafting was anecdotally recommended if interfragmental compression could not be obtained.

     

    In Wright et al's review, there was no significant difference between union rates in comminuted both bone forearm fractures whether or not autograft was used (97% v. 98%). The fractures in this series were all comminuted without open injuries.

     

    Ring et al. reported on their series of comminuted forearm fractures, assessing risks of nonunion. They reported that the largest risk of nonunion was associated with polytrauma patients, followed by open injuries. They found no improvement in union with acute grafting any of their forearm injuries in this series.

     

    Incorrect Answers:

    1. An isolated closed forearm fracture such as this does not typically require external fixation unless soft tissue conditions prevent definitive fixation or splinting.

    2. Flexible nailing of length-unstable forearm fractures is contraindicated.

    3. Acute grafting is not associated with improved union rates in this fracture.

    5) Closed reduction and functional bracing would lead to significant upper extremity disability due to malunion.

     

     

     

    OrthoCash 2020

     

  107. In which of the following radiographs of different types of ankle fractures should the medial malleolus be treated with screw fixation directed parallel to the ankle joint?

     

     

     

     

     

     

     

     

     

     

     

     

     

     

    1. Figure A

    2. Figure B

    3. Figure C

    4. Figure D

    5. Figure E

     

    Corrent answer: 1

    Figure A show a classic SAD (supination adduction) fracture according to the Lauge-Hansen Classfication. This is evident by the vertical medial malleolar fracture and supinated position of the foot. The vertical medial malleolar fracture is best treated by screw fixation parallel to the joint (perpendicular to the fracture line). Careful attention must be paid to the presence of any medial plafond impaction from the talar displacement; if this is present, disimpaction and stabilization must be performed in order to optimize outcomes.

     

    The referenced review article by Michelson covers rotational ankle fractures, with a review of the diagnosis, treatment options, and patient outcomes. He notes that unstable fractures (bimalleolar, bimalleolar equivalent, etc.) usually are managed with open reduction and internal fixation for optimal outcomes.

     

    Incorrect answers:

    Figure B shows a Weber C (high fibular) ankle fracture, PER, without any evidence of a medial malleolar fracture.

    Figures C (SER IV), D (PER IV), and E (isloated medial malleolar fracture) all show fractures not suitable for screw fixation of the medial malleolus parallel to the joint since their fracture lines are not vertical.

     

     

     

    OrthoCash 2020

     

  108. Which of the following is an advantage of using blocking screws for tibial nailing?

    1. Decrease risk of nail breakage

    2. Eliminate use of interlocking screws

    3. Allow for larger nail use

    4. Enhance construct stiffness

    5. Decrease torsional rigidity Corrent answer: 4

    Blocking screws can be used to help obtain and maintain reductions, increase construct stiffness, and neutralize translational forces. There are no studies as of yet that find a blocking screw to decrease nail failure.

     

    Krettek found that medial and lateral blocking screws can increase the primary stability of distal and proximal metaphyseal fractures after nailing and can be an effective tool for selected cases that exhibit malalignment and/or instability by decreasing mechanically measured deformation.

     

    In a later clinical study, Krettek found that after using blocking screws, tibial healing was evident radiologically at a mean of 5.4 months with a decreased

    rate of malunions.

     

    Ricci also found that blocking screws are effective to help obtain and maintain alignment of fractures of the proximal third of the tibial shaft treated with intramedullary nails.

     

     

     

    OrthoCash 2020

     

  109. Following operative repair of lower extremity long bone and periarticular fractures, what is the time frame for patients to return to normal automobile braking time?

    1. 6 weeks after initiation of weight bearing

    2. 4 weeks postoperatively

    3. 8 weeks from the date of injury

    4. Once full range of motion of the ankle and knee exist

    5. At the time of bony union Corrent answer: 1

    According to the first referenced study by Egol et al, appropriate braking time returns at a point 6 weeks after initiation of weightbearing after treatment of lower extremity long bone and periarticular fractures, as examined with a driving simulator. No differences were seen in return of braking time between periarticular fractures and long bone injuries.

     

    The second reference by Egol studied only operatively treated ankle fractures and found that time to appropriate braking returns at 9 weeks postoperatively. Interestingly, no significant association was found between the functional scores and normalization of total braking time.

     

     

     

    OrthoCash 2020

     

  110. A 42-year-old male sustains the injury seen in figure A. What negative sequelae would occur with displacement of this fracture in the characteristic fashion?

     

     

     

    1. Post-traumatic subtalar arthrosis

    2. Stress fracture of the fibula

    3. Reflex sympathetic dystrophy

    4. Achilles tendon rupture

    5. Posterior skin necrosis Corrent answer: 5

    The radiograph shows a tongue-type calcaneus fracture, with major displacement of the posterior calcaneal body/tuberosity. The Achilles tendon insertion here causes characteristic proximal and posterior displacement, and with increasing displacement, posterior skin necrosis can be caused in a short period. Per the references, this should be treated urgently to prevent this sequelae. Lag screw fixation is appropriate for this fracture pattern, placed perpendicular to the fracture.

     

     

     

    OrthoCash 2020

     

  111. A 34-year-old male falls 10 feet from a balcony and is brought to the emergency room with the deformity seen in Figure A. Radiographs shown are shown in Figure B and C. Which of the following structures can block closed reduction of this injury pattern?

     

     

     

     

     

     

     

     

    1. Flexor hallucis longus tendon

    2. Extensor digitorum brevis muscle

    3. Posterior tibial tendon

    4. Tibialis anterior tendon

    5. Plantar fascia

     

    Corrent answer: 2

     

    Figures A through C show a medial subtalar dislocation. Irreducible dislocations are typically the result of either inadequate sedation or interposed soft tissue structures.

     

    In medial dislocations, the extensor digitorum brevis, the deep peroneal neurovascular bundle, or the joint capsule may block a closed reduction. In lateral dislocations, the most common structure implicated as a block to reduction is the posterior tibial tendon, although the flexor digitorum longs, posterior tibial neurovascular bundle or flexor hallucis may also block reduction.

     

    Bibbo et al found that subtalar dislocations were irreducible 32% of the time and that 88% had ipsilateral foot and ankle injuries. At follow up, 89% of patients demonstrated radiographic changes of the subtalar joint, and had worse function on the side of the subtalar dislocation as demonstrated by lower

    AOFAS scores.

     

    Incorrect Answers:

    Answer 1. The flexor hallucis longus tendon may be a block to closed reduction for lateral subtalar dislocations

    Answer 3. The posterior tibial tendon may be a block to reduction for lateral subtalar dislocations

    Answer 4. The tibialis anterior tendon does not commonly preclude closed reduction of a subtalar dislocation

    Answer 5. The plantar fascia does not block closed reduction of the subtalar joint

     

     

     

    OrthoCash 2020

     

  112. Spontaneous rupture of the extensor pollicis longus tendon is most frequently associated with which of the following scenarios?

    1. Non-displaced distal radius fracture

    2. Non-displaced Rolando fracture

    3. Second metacarpal base fracture

    4. Boxer's fracture

    5. Non-displaced radial styloid fracture Corrent answer: 1

    Rupture of the extensor pollicis longus (EPL) tendon after non operative treatment for a distal radius fracture occurs with a 0.3-5% incidence. The causes of EPL rupture include mechanical irritation, attrition, and vascular impairment leading to delayed rupture. Synovitis of the extensor carpi radialis due to repetitive use may invade the EPL tendon and lead to rupture.

    Recommended treatment in the pre-rupture setting includes a third dorsal compartment release with or without an extensor retinacular patch graft. Palmaris longus graft or a transfer from the extensor indicis proprius to the EPL tendon are reasonable treatment options. Results of all treatments seem to be clinically satisfactory.

     

    The referenced article by Gelb is a review of the etiology and treatment of this injury. He reviews the above discussion and findings.

     

     

     

    OrthoCash 2020

     

  113. A 79-year-old cyclist is involved in an accident and sustains a displaced femoral neck fracture as seen in Figure A. What is the

    optimal treatment?

     

     

     

     

    1. Open reduction internal fixation

    2. Bipolar hemiarthroplasty

    3. Unipolar hemiarthroplasty

    4. Total hip arthoplasty

    5. Nonoperative treatment

     

    Corrent answer: 4

     

    An AP pelvis radiographs with a displaced femoral neck fracture is seen in Figure A. It important to note that degenerative changes are seen on this image. Both references suggest that elderly active individuals should be treated with a primary total hip after displaced femoral neck fractures.

     

    In the first study by Blomfeldt et al, the group reviewed a series of patients who underwent either an acute primary total hip arthroplasty for a femoral neck fracture or a delayed primary hip after an attempt at ORIF. They found that the group treated with an acute primary total hip arthroplasty had better Harris hip and quality of life scores.

     

    The second reference from Blomfeldt et al, studies a population of active elderly patients randomized to either a total hip arthroplasty or bipolar for femoral neck fractures. The group found no mortality or dislocation difference between the groups, but higher Harris hip scores at 1 year in patients treated with a total hip

    arthroplasty.

     

     

     

     

    OrthoCash 2020

     

  114. A 53-year-old man sustains the injury seen in figure A and later undergoes open reduction and internal fixation. What variable will

    most significantly increase his rate of degenerative arthritis in the long-term?

     

     

     

     

    1. Postoperative joint stepoff

    2. Alteration of limb mechanical axis

    3. Fracture type

    4. Male sex

    5. Age greater than 50

     

    Corrent answer: 2

     

    Maintenance of mechanical axis correlates most with a satisfactory clinical outcome when managing an intra-articular fracture of the proximal tibia.

     

    According to the study of plateau fractures with up to 27 year follow-up by Rademakers et al, malalignment of the limb by greater than 5 degrees tripled the rate of degenerative osteoarthritis (27% v. 9%). Age at time of injury had no effect on outcome; 31% had joint space narrowing but 64% of those knees were well tolerated.

     

    Weigel and Marsh's study looked at high energy plateau fractures treated with staged external fixation followed by internal fixation, and noted a low rate of severe arthrosis even with mild to moderate joint incongruity.

    Stevens et al noted a worse outcome with increasing age at presentation with these injuries; fracture type had a small influence and adequacy of reduction had no significant influence on outcome.

     

    Figure A is a coronal CT image showing a lateral tibial plateau fracture with significant joint depression.

     

     

     

    OrthoCash 2020

     

  115. A 69-year-old male sustained a proximal humerus fracture that underwent open reduction and internal fixation nine months ago. He complains of constant pain and weakness; repeat radiographs are shown in Figures A and B. What is the most appropriate surgical treatment at this time?

     

     

     

     

     

     

    1. Revision open reduction and internal fixation

    2. Valgus corrective osteotomy of proximal humerus

    3. Shoulder arthroplasty

    4. Shoulder arthrodesis

    5. Humeral head resection Corrent answer: 3

    Figures A and B show loss of fixation of a proximal humerus fracture. The most appropriate treatment for this scenario is a humeral arthroplasty, as the tenuous blood supply of the proximal humerus is likely chronically disrupted, leading to osteonecrosis and poor healing potential of the proximal humerus.

    Traditionally, hemiarthroplasty was performed for these presentations, but reverse total shoulder arthroplasty has emerged as a potentially better

    treatment method, especially if the rotator cuff function/status is unknown or poor.

     

    According to the referenced article by Norris et al, delayed shoulder hemiarthroplasty decreased shoulder pain in 95% of patients but warned of technical difficulties and limited postoperative range of motion. A total shoulder arthroplasty is needed if glenoid erosion from the screw(s) or bone occurs.

     

     

     

    OrthoCash 2020

     

  116. During the ilioinguinal approach to the pelvis, the corona mortis artery must be identified and ligated if present. The corona mortis artery joins the external illiac artery with which other major artery?

    1. Pudendal

    2. Deep illiac circumflex

    3. Hypogastric

    4. Obturator

    5. Testicular

     

    Corrent answer: 4

     

    The "corona mortis" (translated as “crown of death”) artery is a vascular variant that joins the external illiac and the obturator artery as it crosses the superior pubic ramus. Tornetta et al did a study where "fifty cadaver halves were dissected to determine the occurrence and location of the corona mortis. Anastomoses between the obturator and external iliac systems occurred in 84% of the specimens. Thirty-four percent had an arterial connection, 70% had a venous connection, and 20% had both. The distance from the symphysis to the anastomotic vessels averaged 6.2 cm (range, 3-9 cm)." The corona mortis can be injured in superior ramus fractures and iatrogenically while plating pelvic ring injuries using the ilioinguinal approach.

     

     

     

     

     

    OrthoCash 2020

     

  117. A 26-year-old male sustains a femoral shaft fracture treated with the implant shown in Figure A. Postoperatively, what muscular deficits can be expected at medium and long-term follow-up?

     

     

     

    1. Weakness with hip abduction and knee flexion

    2. Weakness with hip abduction and knee extension

    3. Weakness with knee flexion and knee extension

    4. Weakness with hip external rotation and hip abduction

    5. Weakness with hip external rotation and hip flexion Corrent answer: 2

    Figure A shows a femoral shaft fracture treated with an antegrade femoral nail. Long term deficits are weakness with knee extension (quadriceps) and hip abduction (glutei muscles).

     

    The referenced study by Kapp et al noted long term quadriceps weakness as well as decreased bone mineral density in the femur (femoral neck by 9%, the lateral cortex by 20% and the medial cortex by 13%). It is unclear whether this is due to the injury, treatment, or a combination of both.

     

    The second referenced study by Archdeacon et al also noted weakness in hip abduction, which showed time dependent improvement. He reports that increased early ipsilateral trunk lean is associated with worse recovery of abduction strength.

     

     

    OrthoCash 2020

     

  118. A 33-year-old male sustains the injury seen in Figure A as a result of a high-speed motor vehicle collision. Based on this image, what is the most likely acetabular fracture pattern?

     

     

     

     

    1. Both column

    2. Anterior column

    3. Anterior column posterior hemitransverse

    4. Transverse

    5. T-type

     

    Corrent answer: 4

     

    The radiograph in Figure A shows a transverse acetabulum fracture. The iliopectineal (anterior column) and ilioischial lines (posterior column) are interrupted, revealing bicolumnar involvement; however, this is different than the both column fracture, as a transverse pattern has articular surface still in continuity with the axial skeleton via the sacroiliac joint.

     

    The referenced article by Patel et al showed a wide variation of inter and intra-observer agreement in interpreting radiographs of acetabular fractures, with high agreement for basic radiographic classification and only slight to moderate agreement for other radiologic variables such as impaction.

     

    The other referenced article by Letournel is a great review article regarding the initial classification of these fractures as well as a quick summary of his outcomes.

     

     

     

    OrthoCash 2020

  119. A 56-year-old carpenter sustains the closed injury seen in Figures A, B, and C. After temporary spanning external fixation is performed and soft tissue conditions improve, what strategy provides the optimal fixation for this fracture pattern?

     

     

     

     

     

     

     

     

     

    1. Anatomic lateral locking plate

    2. Posteromedial and lateral plates

    3. Anatomic medial locking plate

    4. Conversion of the spanning external fixator to a hinged external fixator

    5. Posterior buttress plate Corrent answer: 2

    Figures A and B show a bicondylar tibial plateau fracture, with a typical appearing lateral fracture line and a posteromedial fracture line. The posteromedial sheared fracture piece is difficult, and/or sometimes impossible, to achieve appropriate stable fixation with a single lateral locking plate, as there will be limited screw purchase and fixation into the posteromedial fragment.

     

    The referenced article by Georgiadis notes that a dual incision approach is safe and is associated with improved outcomes over their historical comparisons.

    They describe the dual incisions and approaches in length, and review risks/issues with each approach.

     

    The other referenced study by Bhattacharyya et al notes that these fractures have a typical appearance of the posteromedial fracture piece and that articular reduction quality is correlated with short-term results. They recommended buttress-type fixation of these fracture pieces.

     

     

    OrthoCash 2020

     

  120. A 31-year-old male sustains an irreducible ankle fracture-dislocation with the foot maintained in an externally rotated position. An AP and lateral radiograph are shown in figures A and B respectively. The attempted post reduction AP and lateral are shown in C and D. What structure is most likely preventing reduction?

     

     

     

     

     

     

    1. Anterior-inferior tibiofibular ligament

    2. Posterior-inferior tibiofibular ligament

    3. Peroneus brevis tendon

    4. Posterolateral ridge of the tibia

    5. Flexor hallucis longus tendon Corrent answer: 4

    As described by Hoblitzell et al, the so-called "Bosworth fracture-dislocation" is a rare fracture-dislocation of the ankle where the fibula becomes entrapped behind the tibia and becomes irreducible. It can cause compartment syndrome, as reported by Beekman and Watson.

     

    Hoblitzell et al stress the importance and difficulty of recognizing these injuries. Standard radiographs are difficult to interpret due to the often severe external rotation of the foot. Prompt treatment, though can lead to good results in patients. The posterolateral ridge of the distal tibia hinders reduction and reduction often requires an open technique

     

    Mayer and Evarts stated AP and mortise radiographs can be hard to interpret due to the external rotation posture of the foot. In their series a closed reduction consisting of traction and medial rotation applied to the foot while the fibular shaft is pushed laterally was successful in 3/4 patients.

     

     

     

    OrthoCash 2020

     

  121. A 37-year-old female sustains the injury seen in Figures A and B. At long-term follow up, degeneration of which of the following joints has been shown to have the highest rate of patient symptoms?

     

     

     

     

     

     

    1. Tibiotalar joint

    2. Talonavicular joint

    3. Calcaneocuboid joint

    4. Lisfranc joint

    5. Subtalar joint

     

    Corrent answer: 5

     

    Figures A and B show a medial subtalar dislocation, which is more common than a lateral dislocation (65% vs. 35%).

     

    The referenced article by Bibbo et al looked at long-term follow up of these patients, and noted that radiographic degeneration of the ankle and subtalar joints were 89%, although 31% of ankle joints were symptomatic and 68% of subtalar joints were symptomatic. Midfoot degeneration was seen radiographically in 72% (15% symptomatic).

     

     

     

    OrthoCash 2020

     

  122. In treating a lateral split-depression type tibial plateau fracture, which of the following adjuncts has been shown to have the least articular surface subsidence when used to fill the bony void?

    1. Crushed cancellous allograft

    2. Hydroxyapatite

    3. Calcium phosphate cement

    4. Autogenous iliac crest

    5. Bisected diaphyseal humeral allograft Corrent answer: 3

    In treating tibial plateau fractures, calcium phosphate has been shown to have the least amount of articular subsidence on follow-up examinations.

     

    The referenced study by Russell et al noted a significantly increased rate of subsidence at 12 months with autograft as compared to calcium phosphate cement (in types I-VI).

     

    The other referenced study by Lobenhoffer et al noted improved radiographic outcomes and earlier weightbearing with usage of calcium phosphate cement.

     

     

     

    OrthoCash 2020

     

  123. The pelvic spur sign on plain radiography is indicative of the following injuries?

    1. Transtectal transverse acetabular fracture

    2. Vertical shear pelvic ring injury

    3. Displaced H-type sacral fracture

    4. Both column acetabular fracture

    5. Anterior-posterior type III pelvic ring injury Corrent answer: 4

    The pelvic spur sign is indicative of a both column acetabular fracture. It is best seen on an AP or obturator oblique x-ray. The spur is the intact portion of the ilium, still attached to the axial skeleton and seen posterosuperior to the displaced acetabulum (typically medially displaced).

     

    Illustration A shows the spur sign (arrows) on a CT image, while illustration B shows an obturator oblique of the pelvis and the spur sign is shown with the long tailed arrow (on the left of the image).

     

     

     

     

     

     

     

     

    OrthoCash 2020

     

  124. A 33-year-old male sustains a distal humerus fracture and is treated with open reduction and internal fixation of the distal humerus with olecranon osteotomy. A postoperative radiograph is shown in Figure A. A new deficit of the anterior interosseous nerve is now noted in the recovery room. What physical exam finding would be expected with this nerve injury?

     

     

     

    1. Inability to flex radiocarpal joint

    2. Loss of sensation over palmar aspect of thumb

    3. Loss of sensation over dorsal hand first webspace

    4. Inability to abduct index finger

    5. Inability to flex thumb interphalangeal joint Corrent answer: 5

    A deficit in the anterior interosseous nerve (AIN) would result in an inability to flex the interphalangeal joint (IPJ) of the thumb.

     

    Injury to the AIN can be seen with K-wires that penetrate through the anterior cortex of the proximal ulna, such as mentioned above. The AIN is a branch of the median nerve that provides motor function to forearm/hand. It branches off from the median nerve 4 cm distal to the medial epicondyle, passes between the 2 heads of the pronator teres, travels through the forearm anterior to the interosseous membrane between the flexor pollicis longs (FPL) and flexor digitorum profundus (FDP), and then terminates in the pronator quadratus (PQ). The nerve gives of branches to the FDP, FPL, and PQ enabling for flexion of the distal phalangeal joint of the index and middle fingers, flexion of the IPJ of the thumb, and aids with pronation of the forearm, respectively.

    Injury to the nerve will result in weakness in motor function to these muscles.

     

    Mekail et al. reviewed the anterior approach to the proximal radius in order to describe and identify important neurovascular and musculoskeletal structures in the area. They were specifically aiming to determine the safest anatomic orientation for plate and screw fixation in regards to the posterior interosseous nerve. The authors, however, did discuss that medial plating was especially dangerous to the AIN, and significantly increased the risk of iatrogenic injury to the branch sent to the FPL.

    Parker et al. reported a case report in a patient who experienced an AIN deficit postoperatively after tension banding of an olecranon fracture.

    Intraoperatively, there were multiple passes of the K-wires in an attempt to find purchase in the anterior cortex of the ulna. The authors believed that during these passes, the nerve was injured and concluded that placing K-wires should not occur without radiologic visualization.

     

    Figure A is a postoperative lateral radiograph after tension banding of the olecranon. Perforation of the anterior ulnar cortex can be seen by the K-wire which can cause damage to the AIN nerve. Illustration A is a schematic of the path of the AIN, its branches, and its function.

     

    Incorrect Answers:

    Answer 1: Both ulnar and median nerves provide innervation muscles that flex the radoiocarpal joint.

    Answer 2: AIN has no cutaneous sensory fibers. Median nerve disruption would result in this deficit.

    Answer 3: Disruption of the superficial radial serve would result in this deficit.. Answer 4: Disruption of the deep branch of the ulnar nerve would result in this deficit.

     

     

     

     

     

    OrthoCash 2020

     

  125. A computed tomography (CT) scan has been shown to be indicated for evaluation of all of the following aspects of acetabular fractures, EXCEPT:

    1. Determination of surgical planning

    2. Intra-articular loose bodies

    3. Marginal impaction

    4. Fracture piece size and position

    5. Determination of pre-existing degenerative changes Corrent answer: 5

    CT scanning is indicated in acetabular fractures for determination of surgical approach and techniques, evaluation of marginal impaction and presence of intra-articular loose bodies (especially after hip dislocation), and evaluation of fracture piece sizes and relative positions.

     

    Kellam et al reviewed their initial experience with CT scanning and acetabular fractures, and noted a 25% change in surgical planning when CT was utilized versus plain radiographs; they also noted the ability to detect marginal impaction and fracture size/position was improved with CT.

     

     

     

    OrthoCash 2020

     

  126. A 69-year-old woman falls while getting out of her car and lands on her right shoulder sustaining a 4-part proximal humerus fracture. She subsequently undergoes surgery to treat the fracture, with immediate postoperative radiographs shown in Figure A. Six months following surgery, she denies shoulder pain, but she is unable to actively raise her hand above her shoulder. Which of the following is the most likely cause of this limitation?

     

     

     

    1. Joint infection

    2. Retroversion of the prosthesis

    3. Glenoid arthritis

    4. Axillary nerve injury

    5. Greater tuberosity malunion Corrent answer: 5

    The radiograph demonstrates a humeral hemiarthroplasty. Malunion of the greater tuberosity is a known complication of this procedure, and the most likely cause for loss of shoulder elevation.

     

    Frankle et al in 2004 reported a 25% rate of greater tuberosity malunion. They discuss surgical techniques to improve fixation of the tuberosities following hemiarthroplasty for proximal humerus fractures.

     

    Frankle et al in 2002 evaluated 5 different techniques to reattach the tuberosities following shoulder hemiarthroplasty in human cadavers. Findings suggested that a circumferential medial cerclage should be placed around the tuberosities to enhance the stability of the tuberosity repair.

     

    Bosch et al reviewed 39 consecutive 3 or 4 part proximal humerus fractures that were treated with either primary hemiarthroplasty or secondary hemiarthroplasty following a primary ORIF. Patients who underwent primary

    hemiarthroplasty reported better clinical outcomes. The authors concluded that elderly patients with 3 or 4 part humerus fractures are best treated with early arthroplasty.

     

     

     

    OrthoCash 2020

     

  127. An acetabular fracture with all segments of the articular surface detached from the intact posterior ilium is defined as what fracture pattern?

    1. Transverse

    2. Both column

    3. Anterior column posterior hemitransverse

    4. Posterior column with posterior wall

    5. Anterior column with anterior wall Corrent answer: 2

    A both column acetabular fracture is defined as an acetabular fracture with no articular surface in continuity with the remaining posterior ilium (and therefore, axial skeleton). The spur sign is a radiological sign seen with these fractures, and is the posterio-inferior aspect of the intact posterior ilium. The spur sign and other radiographic findings consistent with a both column acetabular fracture can be seen in Illustration A (AP), Illustration B (obturator oblique), and Illustration C (iliac oblique).

     

     

     

     

     

     

     

     

     

     

    OrthoCash 2020

     

  128. A large posteromedial tibial plateau fracture pattern, as seen with the bicondylar tibial plateau fracture shown in Figures A and B, is important to recognize because of which of the following factors?

     

     

     

     

     

    1. Association with posteromedial corner of the knee injury

    2. Association with anterior tibial artery injury

    3. Possible need for dual plate fixation

    4. Possible need for single extensile anterior approach to the knee

    5. Increased risk of deep venous thrombosis Corrent answer: 3

    Figures A and B show a bicondylar tibial plateau fracture with a large posteromedial fracture piece. This has clinical importance, as currently available plate/screw constructs often have poor fixation of this fracture segment, and this pattern often requires a second, posteromedial, approach and placement of a second plate/screw construct.

     

    The referenced article by Barei et al notes a prevalence of posteromedial fracture pieces of nearly 33% of all bicondylar tibial plateau fractures. They also recommend supplementary or alternative fixation techniques when this pattern is recognized.

     

    The referenced article by Higgins et al notes a 59% incidence of this fracture pattern (consisting of nearly 25% of the total joint surface) in bicondylar tibial plateau fractures, and recommends appropriate fixation to combat the vertical shear instability through a separate approach.

     

    The last referenced study by Higgings et al notes a significantly increased rate of late fracture displacement in a biomechanical model with a single lateral locking plate as compared to a dual plate construct.

     

     

     

    OrthoCash 2020

     

  129. At the elbow, the anterior bundle of the medial collateral ligament inserts at which site?

    1. Radial tuberosity

    2. 3mm distal to the tip of the coronoid

    3. Anteromedial process of the coronoid

    4. Medial border of the olecranon fossa

    5. Radial side of ulna at origin of annular ligament Corrent answer: 3

    The anterior bundle of the medial collateral ligament of the elbow inserts at the anteromedial process of the coronoid, also known as the sublime tubercle.

    Fractures at this site have been shown to have worse results with nonoperative

    treatment, due to increased rates of instability and post-traumatic arthrosis.

     

    The referenced articles by Ring and Steinmann are great reviews of the topic of coronoid fractures. They review the diagnosis, treatment options, rehabilitation, and outcomes of these injuries. They focus on the importance of the coronoid in elbow stability, especially with base fractures, or ones that involve the sublime tubercle.

     

    Illustration A depicts the anterior bundle of the MCL inserting at the sublime tubercle.

     

     

     

     

     

     

    OrthoCash 2020

     

  130. In the Lauge-Hansen classification system, a pronation-abduction ankle fracture has what characteristic fibular fracture pattern?

    1. Transverse fracture below the level of the syndesmosis

    2. Short oblique fracture running from anteroinferior to posteriosuperior

    3. Short oblique fracture running from posteroinferior to anteriosuperior

    4. Comminuted fracture at or above the level of the syndesmosis

    5. Wagstaff fracture

     

    Corrent answer: 4

     

    In the Lauge-Hansen classification, the characteristic fibular fracture pattern in a pronation-abduction injury is a comminuted fibular fracture above the level of the syndesmosis. In the first stage of this injury pattern, the deltoid fails in tension, or an avulsion fracture of the medial malleolus occurs. In the second stage, the anterior inferior tibiofibular ligament ruptures, or a small bony avulsion of this ligament's insertion/origin occurs. The final stage includes the creation of a comminuted fibular fracture above the level of the syndesmosis.

    The referenced article by Siegel et al noted that extraperiosteal bridge plating of these ankle injuries was safe and had excellent radiographic and clinical outcomes at final follow-up.

     

     

     

    OrthoCash 2020

     

  131. A 38-year-old male sustains the closed injury shown in Figures A and B. When treating this injury with an intramedullary nail, addition of blocking screws into which of the following positions can prevent the characteristic malunion deformity?

     

     

     

     

     

     

    1. Anterior to the nail in the proximal segment; medial to the nail in the proximal segment

    2. Anterior to the nail in the proximal segment; lateral to the nail in the proximal segment

    3. Posterior to the nail in the proximal segment; lateral to the nail in the proximal segment

    4. Anterior to the nail in the distal segment; lateral to the nail in the distal segment

    5. Posterior to the nail in the distal segment; medial to the nail in the proximal segment

    Corrent answer: 3

     

    Figures A and B show a proximal tibia fracture, which is prone to malreduction/malunion into a characteristic valgus and procurvatum (apex anterior) deformity. Placement of screws in this instance posterior to the nail (medial to lateral) and lateral to the nail (anterior to posterior) in the proximal segment will prevent iatrogenic malalignment.

     

    Intramedullary nails will not effect a reduction in metaphyseal proximal tibia fractures. Valgus and apex anterior deformities in these injuries may be caused by deforming muscular forces, limb positioning in hyper flexion, as well as iatrogenic deformity created by improper nail insertion technique. Blocking (Poller) screws are utilized to redirect intramedullary nails by creating an artificial cortex to guide the nail into appropriate position.

     

    The referenced biomechanical study by Krettek et al noted that addition of blocking screws added increased stability to metaphyseal fractures.

     

    Ricci et al noted no malalignment intraoperatively or at final follow-up of proximal tibia fractures treated with intramedullary nails if blocking screws were used.

     

     

     

    OrthoCash 2020

     

  132. In an uninjured proximal tibia which statement best describes the shape and position of the medial tibial plateau relative to the lateral tibial plateau?

    1. More concave and more proximal

    2. More convex and more proximal

    3. More concave and more distal

    4. More convex and more distal

    5. Symetric in conture and more distal

    Corrent answer: 3

     

    The medial tibial plateau is more concave and more distal relative to the lateral tibial plateau.

     

    Watson et al report "the medial tibial plateau has a more concave shape and is larger in both length and width than the lateral tibial plateau, which has a slightly convex shape. The lateral tibial plateau lies proximal to the medial plateau. The convexity of the lateral plateau helps differentiate it from the medial plateau on a lateral radiograph of the proximal tibia."

     

    Illustration A shows the relative concavity of the medial and lateral proximal tibia.

     

     

     

     

     

     

    OrthoCash 2020

     

  133. On average, the radial nerve travels from the posterior compartment of the arm and enters the anterior compartment at which of the following sites?

    1. Spiral groove of the humerus

    2. At the arcuate ligament of Osborne

    3. 10 cm distal to the lateral acromion

    4. 10 cm proximal to radiocapitellar joint

    5. At the origin of the deep head of the triceps Corrent answer: 4

    The radial nerve enters the anterior compartment through the intercompartmental fascia on average 10 cm proximal to the radiocapitellar joint. It has never been found to remain in the posterior compartment within

    7.5cm of this joint, leading to this area being named the "safe zone". During the posterior approach to the humerus, the radial nerve is found in the spiral groove in the middle third of the posterior humerus, medial to the lateral head and proximal to the deep head of the triceps. When performing an ORIF of a

    humerus fracture from a posterior approach it should be identified and protected.

     

    Illustration A shows the radial nerve as seen during the posterior approach to the humerus. Illustration B shows the radial nerve along with a ruler showing the transition at 10cm proximal to the radiocapitellar joint.

     

     

     

     

     

     

     

     

     

    OrthoCash 2020

     

  134. A 45-year-old female pedestrian is hit by an automobile. A clinical photo and radiograph are shown in Figure A and B. What is the most

    important factor in a surgeon's decision of determining between limb salvage and amputation?

     

     

     

     

     

     

     

    1. Level of education

    2. Lack of plantar sensation

    3. Contralateral lower extremity open fracture(s)

    4. Severity of soft tissue injury

    5. Amount of tibial bone loss

    Corrent answer: 4

     

    The clinical photo and radiograph are consistent with a Grade III open tibia fracture.

     

    The referenced study by the LEAP group reviews 527 patients with severe lower extremity fractures and found that the most important factor in determining the ability to salvage the extremity remains the severity of the soft tissue injury of that extremity. Bone loss has been shown to have no effect on the eventual outcome (amputation versus salvage). Similarly, plantar sensation at presentation has no bearing on final outcome, and in the LEAP study, often either partially or fully returned.

     

     

     

    OrthoCash 2020

     

  135. During an open reduction internal fixation of a humerus fracture using the posterior approach, a surgeon can identify the posterior antebrachial cutaneous nerve and trace it proximally to which of the following nerves?

    1. Ulnar

    2. Musculocutaneous

    3. Radial

    4. Median

    5. Axillary

     

    Corrent answer: 3

     

    The posterior antebrachial cutaneous nerve (PABCN) branches from the radial nerve in the axilla.

     

    The posterior antebrachial cutaneous nerve branches from the radial nerve just distal to the posterior brachial cutaneous nerve (PBCN) in the axilla and they course through the arm in closely to each other. In the proximal forearm, the posterior antebrachial cutaneous nerve is found on the lateral border of the brachioradialis muscle. The terminal branches innervate the posterior aspect of the forearm distally.

     

    Gerwin et al recommended identifying the lower lateral brachial cutaneous nerve first when approaching the humerus posteriorly. It can be traced proximally to safely identify the radial nerve before any proximal exposure of the shaft is done. Gerwin et al in their review also found that the radial nerve crossed the posterior aspect of the humerus an average of 20.7 +/- 1.2 centimeters proximal to the medial epicondyle to 14.2 +/- 0.6 centimeters

    proximal to the lateral epicondyle.

     

    In their review, Zlotolow et al. review the multiple surgical approaches to the humerus.

     

    Illustration A depicts the course of the PABCN and its relation to the PBCN and the radial nerve

     

     

     

     

     

     

    OrthoCash 2020

     

  136. A 37-year-old male sustains the closed injury seen in figure A. What technique can be utilized to avoid the characteristic deformity seen in this fracture pattern if an intramedullary nail is used for treatment?

     

     

     

    1. Medial starting point

    2. Lateral starting point

    3. Aiming the nail posteriorly in the proximal segment

    4. Anterior blocking screw in the proximal segment

    5. Medial blocking screw in the proximal segment Corrent answer: 2

    Figure A shows a proximal metaphyseal tibia fracture, which characteristically is malreduced into valgus and apex anterior (procurvatum) deformity. Some techniques to avoid these deformities are: provisional reduction with unicortical plates/clamps, semi-extended nailing, suprapatellar nailing, usage of a more lateral starting point, usage of an external fixator or femoral distractor, and usage of blocking screws - posterior screw and/or a lateral screw in the proximal segment.

     

    The two referenced studies draw attention to the high rate of malalignment with nailing of this fracture pattern; the first study reported a 58% malalignment rate, and the second reported an 84% rate (>5 degrees in either coronal or sagittal planes).

     

     

     

    OrthoCash 2020

  137. A 19-year-old male sustains the isolated, closed injury seen in Figure A. He is subsequently treated as shown in Figure B. When utilizing this technique, what forces are generated at the articular surface?

     

     

     

     

     

     

     

    1. Neutralization

    2. Torque

    3. Two-point bending

    4. Shear

    5. Compression

     

    Corrent answer: 5

     

    Figure A and B show a simple transverse olecranon fracture appropriately treated with a tension-band construct. This construct converts distraction forces at the joint generated by the pull of the triceps into compression forces. The change of force into compression requires active motion of the elbow extensor mechanism.

     

     

     

    OrthoCash 2020

     

  138. Antegrade femoral nailing has an increased rate of which of the following when compared to retrograde femoral nailing?

    1. Varus malalignment

    2. Union rate

    3. Operative time

    4. Subsequent operative procedures

    5. Hip pain

     

    Corrent answer: 5

     

    In the referenced study by Ricci et al, antegrade femoral nailing was shown to have an increased rate of hip pain as compared to retrograde femoral nailing, while having a similar rate of union, time to union, rate of malalignment, and operative time. Hip pain was signficantly higher in the antegrade nailing group, while knee pain was significantly greater in the retrograde group.

     

    The referenced study by Winquist et al noted a 99.1% union rate with intramedullary nailing.

     

    The referenced study by Moed et al noted a 6% nonunion rate in non-reamed retrograde femoral nailing with nail dynamization at 6-12 weeks and early weightbearing.

     

     

     

    OrthoCash 2020

     

  139. A 25-year-old male is involved in a motor vehicle accident and presents with the injury shown in Figure A. Early fixation of this fracture pattern is associated with all of the following EXCEPT?

     

     

     

    1. Decreased length of hospital stay

    2. Improved functional outcome

    3. Greater organ dysfunction

    4. Higher likelihood of being discharged to home as opposed to a rehab facility

    5. Improved fracture reduction Corrent answer: 3

    Early fixation of acetabular fractures is associated with lesser organ dysfunction, so therefore answer three is not true.

     

    Plaisier et al showed the timing of acetabular and pelvic ring fracture fixation greatly impacted patient outcome. Patients who had fixation within 24 hours of injury showed shorter length of stay in the hospital and ICU (decreased number of ventilator days), improved functional outcomes including a highly likelihood of being discharged to home as opposed to a rehabilitation facility, and lesser organ dysfunction.

     

    The reference by Matta et al is a classic article that shows that patients fixed within 3 weeks of injury showed both a higher rate of anatomical reduction and lower overall complication rate than patients with similar fracture patterns treated after 3 weeks.

     

     

     

    OrthoCash 2020

     

  140. A 33-year-old man requires a transfemoral amputation because of a mangling injury to his leg. Six months after the amputation he has persistent difficulty with ambulation because his distal femur moves into a subcutaneous position in his lateral thigh. It persists despite a

    well-fitted prosthesis. What technical error is the most likely cause of his dysfunction?

    1. Inadequate posterior skin flap

    2. Inadequate anterior skin flap

    3. Failure to bevel the distal femur

    4. Lack of abductor myodesis to femur

    5. Lack of adductor myodesis to femur Corrent answer: 5

    Adductor myodesis is a critical part of a transfemoral amputation. If it is not performed, then the abductors and hip flexors can cause the femur to abduct, leading to severe problems with gait. The gait disturbance persists despite proper prosthetic fitting. A transfemoral amputation is usually performed with equal anterior and posterior flaps.

     

    Pinzur et al highlight the fact that amputations are reconstructive procedures and should leave the patient with a functional residual limb.

     

     

     

    OrthoCash 2020

     

  141. An 18-year-old football player presents to the emergency department after sustaining an ankle injury. His radiograph is shown in figure A. What is the most appropriate definitive treatment?

     

     

     

    1. Open reduction and internal fixation of the medial malleolus with syndesmosis reduction and suture-button repair

    2. Repair of the anterior talo-fibular ligament

    3. Open reduction internal fixation of the fibula with syndesmosis reduction and suture-button repair

    4. Open reduction internal fixation of the medial malleolus and fibula

    5. Open reduction internal fixation of the fibula and medial malleolus with syndesmosis reduction and suture-button repair

    Corrent answer: 5

     

    The radiograph demonstrates an ankle fracture-dislocation. There is diastasis of the distal tibia and fibula, indicating a syndesmosis injury.

     

    Zalavras et al stated failure to recognize and treat the syndesomsis injury leads to inferior outcomes, and should be assessed after fibula and medial malleolar fixation. Treatment of choice is reduction of the syndesmosis and fixation.

     

     

     

    OrthoCash 2020

  142. A patient sustains a severe lower extremity injury. What can be said about his outcome at 2 years if he chooses reconstruction over amputation?

    1. He has a higher risk of rehospitalization

    2. He has a higher chance of returning to work

    3. He will have a higher overall SIP (Sickness Impact Profile) score

    4. His psychosocial SIP score will improve with time

    5. He will have a better SIP score if he did not complete high school Corrent answer: 1

    Severe lower extremity injury patients undergoing reconstruction have a higher rate of rehospitalization at 2 years. This question is based on data published by the LEAP study group, a multi-centered study of severe extremity injuries treated with either amputation or reconstruction.

     

    Bosse et al found that at 2 years the SIP score and return to work were not statistically signficantly different between amputation and reconstruction groups. Reconstruction patients had a higher risk of rehospitalization. The psychosocial subscale of SIP did not improve with time. Risk factors for poorer SIP score were: rehospitalization for a major complication, a low educational level, nonwhite race, poverty, lack of private health insurance, poor social-support network, low self-efficacy (the patient's confidence in being able to resume life activities), smoking, and involvement in disability-compensation litigation.

     

    MacKenize et al evaluated factors influential in returning to work (RTW) after severe lower extremity injury. Characteristics that correlated with higher rates of RTW included younger age, higher education, higher income, the presence of strong social support, and employment in a white-collar job that was not physically demanding. Receipt of disability compensation had a strong negative effect on RTW.

     

     

     

    OrthoCash 2020

     

  143. Lateral malleolus fractures can be treated with a variety of techniques, including posterior antiglide plating or lateral neutralization plating. What is an advantage of using lateral neutralization plating instead of posterior antiglide plating?

    1. Decreased joint penetration of distal screws

    2. Increased rigidity

    3. Decreased need for delayed hardware removal

    4. Decreased peroneal irritation

    5. Improved distal fixation Corrent answer: 4

    Posterior antiglide plating is a technique that involves placement of a plate on the posterior aspect of the distal fibula, using the plate as a reduction tool and direct buttress against distal fracture fragment displacement.

     

    Schaffer et al showed from a biomechanical standpoint that posterior antiglide plating was superior to lateral neutralization plating for distal fibula fracture fixation.

     

    Weber et al reported a (30/70) 43% rate of plate removal secondary to peroneal discomfort. In addition, peroneal tendon lesions were found in 9 of the 30 patients.

     

     

     

    OrthoCash 2020

     

  144. Varus malalignment after a talar neck fracture with medial comminution causes a decrease in what motion?

    1. Tibiotalar dorsiflexion

    2. Tibiotalar plantarflexion

    3. Subtalar eversion

    4. Subtalar inversion

    5. Internal rotation

     

    Corrent answer: 3

     

    Varus alignment at the talar neck results in a decrease in subtalar eversion before impingement occurs. Varus talar neck alignment can cause a fixed internal rotation position of the midfoot as the navicular follows the talar head. This can lead to a more rigid hindfoot which is specifically manifested as a decreased eversion range of motion.

     

    Illustration A is a diagram of the hindfoot that shows how malalignment can affect the hindfoot.

     

    Herscovici et al review the appropriate management of complex ankle and hindfoot injuries in this instructional course lecture.

     

    Daniels et al performed a cadaveric study where they osteotomized the talar neck and then studied ankle motion with and without removal of a medially

    based wedge of bone. They found that subtalar eversion was specifically decreased.

     

    Sanders et al found that secondary reconstructive procedures following talar neck fractures were most commonly performed to treat subtalar arthritis or misalignment.

     

     

     

     

     

     

    OrthoCash 2020

     

  145. Which of the following nonunions is appropriately treated with exchange reamed nailing without bone graft augmentation?

    1. Infected tibial shaft nonunion 6 months status post intramedullary nail fixation

    2. Oligotrophic humeral shaft nonunion 7 months status post non-operative management

    3. Hypertrophic tibial shaft nonunion 7 months status post intramedullary nail fixation

    4. Comminuted open tibial shaft nonunion with segmental bone loss 8 months status post intramedullary nail fixation

    5. Supracondylar femoral shaft nonunion 6 months status post intramedullary nail fixation with 4 distal locking screws

    Corrent answer: 3

     

    Exchange nailing is indicated for nonunions of diaphyseal femoral and tibia fractures in the absence of infection, comminution, or segmental bone loss. Hypertrophic nonunions need better stability (increased nail diameter) to

    achieve union. Where as atrophic nonunions often need better biology (bone graft, flap coverage, etc.)

     

    The referenced article by Brinker et al reviews the indications for exchange nailing. They argue, on the basis of the available literature, that exchange nailing is an excellent choice for aseptic nonunions of noncomminuted diaphyseal femoral and tibia fractures.

     

    Zelle et al. demonstrated 95% success with reamed exchange nailing for the treatment of aseptic tibial shaft nonunions that were initially treated with nonreamed intramedullary nailing.

     

     

     

    OrthoCash 2020

     

  146. A 32-year-old man presents to the emergency department with a humeral shaft fracture. He has wrist drop as well as impaired finger and thumb extension. Which motor function would be expected to recover last?

    1. Elbow extension

    2. Forearm supination

    3. Wrist extension in radial deviation

    4. Middle finger MCP extension

    5. Index finger MCP hyperextension Corrent answer: 5

    The patient is presenting with radial nerve palsy secondary to his humerus fracture. Motor recovery proceeds in a proximal to distal direction.

     

    Abrams et al detailed the order of innervation of the radial nerve and found the following order (proximal to distal): brachioradialis, extensor carpi radialis longus, supinator, extensor carpi radialis brevis, extensor digitorum communis, extensor carpi ulnaris, extensor digiti quinti, abductor pollicis longus, extensor policis longus, extensor policis brevis, and extensor indicis proprius.

     

    Branovacki et al found a slightly different pattern: brachioradialis, extensor carpi radialis longus, superficial sensory, extensor carpi radialis brevis, supinator, extensor digitorum/extensor carpi ulnaris, extensor digiti minimi, abductor pollicis longus, extensor pollicis brevis, extensor pollicis longus and extensor indicis proprius.

     

    While both extensor digitorum and extensor indicis proprius extend the index

    finger MCP joint, only extensor indicis proprius hyperextends the index finger past neutral.

     

     

     

    OrthoCash 2020

     

  147. A 22-year-old female is involved in a motor vehicle collision and sustains the injury shown in Figures A through D. According to these images, what is the acetabular fracture classification?

     

     

     

     

     

     

     

     

     

     

     

     

     

    1. Anterior column posterior hemitransverse

    2. Both column

    3. Transverse

    4. Transverse with posterior wall

    5. Anterior column

     

    Corrent answer: 2

     

    Figures A through D show a comminuted both column acetabular fracture. In this injury, both columns are involved, with the acetabulum losing all connection to the axial skeleton (sacrum). This differentiates it from all other patterns, where at least part of the acetabular cartilage maintains connection to the sacrum.

     

    Figure C shows the ischial spur, which is classically known as the spur sign and most easily seen on the obturator oblique radiograph.

     

    Incorrect Answers:

    Answer 1: This injury has axial skeleton attachment to the acetabular cartilage through the posterior column.

    Answer 3: This injury has axial skeleton attachment to the acetabular cartilage through the anterior and posterior columns.

    Answer 4: This injury has axial skeleton attachment to the acetabular cartilage through the anterior column as well as the posterior column, depending on fracture pattern.

    Answer 5: This injury has no posterior column involvement, and therefore the posterior column maintains the axial skeleton attachment to the acetabulum.

     

     

     

    OrthoCash 2020

     

  148. A 27-year-old man sustains a displaced femoral neck fracture and undergoes urgent open reduction internal fixation. What is the most prevalent complication after this injury?

    1. Flexion contracture

    2. Hip instability

    3. Nonunion

    4. Abductor lurch

    5. Osteonecrosis

     

    Corrent answer: 5

     

    Femoral neck fractures in young patients are difficult to treat, and AVN is a significant concern. Despite advances in both imaging and implants, this injury often leads to functional impairment.

     

    Haidukewych et al followed treatment of femoral neck fractures in young

    patients. They found almost 10% of displaced fractures were associated with the development of nonunion, where as 27% were associated with the development of osteonecrosis. Their results were influenced by fracture displacement and the quality of reduction. Varus malreduction most closely correlates with failure of fixation after reduction and cannulated screw fixation.

     

    Swiontkowski reviews both the treatment and post operative complications in intracapsular hip fractures. In this Current Concept Review, the rate of AVN was discussed as being related to the pre-operative degree of displacement seen on radiographs.

     

    Incorrect Responses:

    Answers 1 & 4: While each of these complications do occur, they are less common and are related to the approach and degree of surgical dissection. Answer 2: Hip instability is relatively uncommon.

    Answer 3: Nonunion rate is significant but lower than the AVN rate. It is has been associated with the degree of initial displacement and varus malreduction.

     

     

     

    OrthoCash 2020

     

  149. A patient with an intertrochanteric hip fracture undergoes reduction and dynamic hip screw application. The post-operative radiographs demonstrate that the lag screw is superior in the femoral head with a tip-apex distance of 40 millimeters. This patient is at increased risk of what complication?

    1. lag screw cutout

    2. osteonecrosis

    3. osteoarthritis

    4. peri-prosthetic fracture

    5. lag screw breakage Corrent answer: 1

    Baumgaertner et al in their classic study in 1995 determined that the position of the lag screw in the femoral head influenced the risk of cutout of a dynamic hip screw construct in treatment of intertrochanteric fractures. They had no cutouts if the tip-apex distance on the combined AP and lateral radiographs was less than 25 millimeters. Subsequent studies demonstrated a decreased cutout rate once people were aware of the tip-apex distance importance.

     

     

    OrthoCash 2020

     

  150. A 37-year-old male sustains the injury shown in Figure A following a motorcycle crash. During the approach, what limb position minimizes tension placed on the sciatic nerve?

     

     

     

     

    1. Hip at 45 degrees, knee flexed to 90 degrees

    2. Hip at 60 degrees, knee flexed to 90 degrees

    3. Hip at 90 degrees, knee extended

    4. Hip at 0 degrees, knee flexed to 90 degrees

    5. Hip at 90 degrees, knee flexed to 90 degrees Corrent answer: 4

    During the Kocher-Langenbeck approach, the sciatic nerve is at the least amount of tension with the hip extended and the knee flexed to 90 degrees.

     

    The CT exhibits a posterior wall acetabular fracture, which is fixed via a Kocher-Langenbeck approach. The sciatic nerve, which comes out of the greater sciatic notch, is at the least amount of tension with the hip extended and knee flexed to 90 degrees.

     

    Borrelli et al examined the intraneural pressure of the sciatic nerve with the hip and knee in various different positions. The authors noted that the highest tension was placed on the sciatic nerve when the hip was flexed to 90 degrees and the knee was fully extended. As a result, the leg is typically position with the hip in extension (or minimal flexion) and the knee in about 90 degrees of flexion when performing acetabular surgery via a posterior approach.

    Figure A shows an axial pelvic CT cut with a posterior wall acetabular fracture. Incorrect answers:

    Answer 1,2,3,5: With any degree of hip flexion, it places tension on the sciatic

    nerve, answer 4 (hip flex to 90 and knee fully extended), specifically places the highest amount of intraneural pressure on the nerve.

     

     

     

    OrthoCash 2020

  151. A 21-year-old male sustains the open injury shown in Figure A, which is associated with a 12 centimeter laceration over the fracture site. This laceration is able to be closed during initial surgery. What adjunct treatment has been shown to improve outcomes when using an intramedullary nail?

     

     

     

     

    1. rhBMP-7

    2. Adjunctive fracture plating

    3. Calcium phosphate

    4. Antibiotic impregnated cement beads

    5. rhBMP-2

     

    Corrent answer: 5

     

    rhBMP-2 has been shown in two randomized controlled studies to have improved clinical outcomes in grade III open tibial fractures.

     

    Swiontkowski et al and Govender et al have shown in two separate clinical studies that use of this product has: significantly fewer invasive interventions (e.g., bone-grafting and nail exchange), significantly faster fracture-healing than did the control patients, increased healing (union) rates, fewer hardware failures, fewer infections, and faster wound-healing (83% compared with 65%

    had wound-healing at six weeks).

     

     

     

     

    OrthoCash 2020

     

  152. Which of the following statements is true regarding treatment of intertrochanteric hip fractures with an intramedullary nail versus a sliding hip screw?

    1. The use of intramedullary nail has increased in the last ten years

    2. The use of sliding hip screws has increased in the last ten years

    3. Medicare reimbursement is more for a sliding hip screw

    4. Intramedullary nails have demonstrated superior outcomes in randomized-controlled studies

    5. Sliding hip screw is superior for treatment of reverse obliquity intertrochanteric fractures

    Corrent answer: 1

     

    The use of intramedullary (cephalomedullary) devices has increased in the last ten years despite a lack of evidence to support superiority over extramedullary implants (sliding hip screws)

     

    Intertrochanteric hip fractures remain one of the most common injuries managed by Orthopaedic surgeons. The optimal form of surgical stabilization for these injuries has been a topic of debate, however several recent studies have demonstrated equivalent outcomes with long cephalomedullary nails and sliding hip screws.

     

    Anglen et al. reviewed the database of orthopaedic surgeons taking their oral board examination. The authors found that the use of intramedullary nails for intertrochanteric hip fractures dramatically increased from 3% in 1999 to 67% in 2006. The authors calls attention to the fact that reimbursement was higher until 2010 for intramedullary nails despite a lack of evidence demonstrating superiority.

     

    Forte et al. evaluated geographic variation in the use of intramedullary nails to treat intertrochanteric hip fractures. The authors found significant regional variation in the use of these devices despite similarities in the treatment populations.

     

    Barton et al. conducted a Level 1 prospective randomized controlled study comparing long cephalomedullary nails with sliding hips screws in the treatment of unstable intertrochanteric fractures (AO/OTA 31-A2). The authors

    found no significant difference in any of the measured variables when comparing the two devices.

     

    Incorrect Answers:

    Answer 2: The use of the sliding hip screw has decreased despite equivalence with cephalomedullary nails

    Answer 3: Until 2010 Medicare reimbursement was more for cephalomedullary nails.

    Answer 4: Intramedullary nails have not been shown to have superior outcomes in multiple studies

    Answer 5: Sliding screws have been shown to have worse outcomes for reverse obliquity fractures

     

     

     

    OrthoCash 2020

     

  153. A 34-year-old man is brought to the trauma bay following a motorcycle collision with a left femoral shaft fracture and an open right tibial plateau fracture. Radiographs are provided in figures A and

    B. He is proceeding to the operating room for an emergent splenectomy. The mean arterial pressure is 51 mmHg following 6 units of packed red blood cells as well as crystalloid replacement. Base deficit is 10 mmol/L. Neurosurgery is concerned for evolving subdural hematoma and is recommending serial head CT scans. Which of the following is the best immediate treatment option to address his fractures?

     

     

     

     

     

     

    1. Irrigation and debridement of open tibia plateau fracture and traction stabilization of femur and tibia plateau fractures

    2. Irrigation and debridement with open reduction internal fixation of tibial plateau fracture and intramedullary nail fixation of femur fracture

    3. Irrigation and debridement with open reduction internal fixation of tibial plateau fracture and plate fixation of femur fracture

    4. Irrigation and debridement with external fixation of tibia plateau fracture and reamed intramedullary nail fixation of femur fracture

    5. Irrigation and debridement with external fixation of tibia plateau fracture and external fixation of femur fracture

    Corrent answer: 5

     

    Radiographs demonstrate a femoral shaft and high-energy tibia plateau fracture. The patient is medically unstable and the best treatment is expeditious debridement of the open fracture and stabilization of the fractures with definitive fixation at a later date. Early stabilization reduces the risk of cardiopulmonary complications including fat embolism syndrome.

     

    Roberts et al recommends damage control orthopaedics emphasizing fracture stabilization without definitive surgical treatment in the unstable trauma patient. They note that this treatment method adds little physiological stress to the traumatized patient.

     

    Turen et al discusses the importance of early fixation of long bone fractures to mobilize the multiple extremity trauma patient and mitigate cardiopulmonary complications. They note, however, that understanding of the complexities of the multiply injured patient is necessary to avoid intensive surgical treatments that are likely to adversely affect outcome.

     

     

     

    OrthoCash 2020

     

  154. A 79-year-old woman with osteoporosis presents with a displaced, severely comminuted olecranon fracture involving the proximal 40%. Which of the following represents the most appropriate surgical treatment?

    1. Intramedullary screw

    2. Kirschner wire tension band

    3. Total elbow arthroplasty

    4. Fragment excision and triceps advancement

    5. Dorsal bridge plating Corrent answer: 4

    Multiple treatments exist for olecranon fractures. Tension band construct (Illustration A) and intramedullary screw or k-wire placement are typically reserved for non-comminuted olecranon fractures, whereas plate and screw

    fixation (Illustration B) is used for comminuted fractures.

     

    Hak et al review olecranon fracture treatment and state that fragment excision and triceps advancement is most appropriate in elderly, osteoporotic patients with severely comminuted fractures involving the proximal 30-40% of the olecranon.

     

    Veillette et al state that when performing a triceps advancement for treatment of an olecranon fracture, between 50% and 70% of the olecranon articular surface can be excised without compromising elbow stability provided the coronoid and distal trochlea are preserved.

     

    When excision and triceps advancement is performed, the triceps should be attached adjacent to the articular surface.

     

     

     

     

     

     

     

     

    OrthoCash 2020

     

  155. Buttress plating is most appropriate in which of the following clinical situations?

 

 

 

 

 

 

 

 

 

Dr. Mohammed Hutaif

About the Author: Prof. Dr. Mohammed Hutaif

Vice Dean of the Faculty of Medicine at Sana'a University and a leading consultant in orthopedic and spinal surgery. Learn more about my expertise and achievements.

Share this article